Você está na página 1de 197

SUMÁRIO

MA-14 - Aula 01 . . . . . . . . . . . . . . . . . . . . . . . . . . . . . . . . . . . . . . . . . . . . . 1
1.2 Divisibilidade: Problemas . . . . . . . . . . . . . . . . . . . . . . . . . . . 1
1.2 Divisão Euclidiana: Problemas . . . . . . . . . . . . . . . . . . . . . . . . . 7
2.3 Unidades 1 e 2 . . . . . . . . . . . . . . . . . . . . . . . . . . . . . . . . . 10
MA-14 - Aula 02 . . . . . . . . . . . . . . . . . . . . . . . . . . . . . . . . . . . . . . . . . . . . . 23
3.1 Sistemas de Numeração: Problemas . . . . . . . . . . . . . . . . . . . . . . 23
4.2 Jogo de Nim: Problemas . . . . . . . . . . . . . . . . . . . . . . . . . . . . 28
MA-14 - Aula 03 . . . . . . . . . . . . . . . . . . . . . . . . . . . . . . . . . . . . . . . . . . . . . 29
5.2 Máximo Divisor Comum: Problemas . . . . . . . . . . . . . . . . . . . . . 29
6.2 Propriedades do mdc: Problemas . . . . . . . . . . . . . . . . . . . . . . . 35
MA-14 - Aula 04 . . . . . . . . . . . . . . . . . . . . . . . . . . . . . . . . . . . . . . . . . . . . . 45
7.2 Mínimo Múltiplo Comum: Problemas . . . . . . . . . . . . . . . . . . . . . 45
8.2 Equações Diofantinas Lineares: Problemas . . . . . . . . . . . . . . . . . . 51
8.3 Exercícios Suplementares . . . . . . . . . . . . . . . . . . . . . . . . . . . . 57
MA-14 - Aula Revisão . . . . . . . . . . . . . . . . . . . . . . . . . . . . . . . . . . . . . . . . 59
9.2 Revisão . . . . . . . . . . . . . . . . . . . . . . . . . . . . . . . . . . . . . 59
MA-14 - Aula 05 . . . . . . . . . . . . . . . . . . . . . . . . . . . . . . . . . . . . . . . . . . . . . 73
10.2 Expressões Binômias . . . . . . . . . . . . . . . . . . . . . . . . . . . . . . 73
11.1 Números de Fibonacci . . . . . . . . . . . . . . . . . . . . . . . . . . . . . 78
MA-14 - Aula 06 . . . . . . . . . . . . . . . . . . . . . . . . . . . . . . . . . . . . . . . . . . . . . 81
12.2 Teorema Fundamental Da Aritmética . . . . . . . . . . . . . . . . . . . . . 81
12.3 Problemas . . . . . . . . . . . . . . . . . . . . . . . . . . . . . . . . . . . . 83
13.1 Pequeno Teorema de Fermat . . . . . . . . . . . . . . . . . . . . . . . . . . 90
13.2 Problemas . . . . . . . . . . . . . . . . . . . . . . . . . . . . . . . . . . . . 90
13.3 Exercícios suplementares . . . . . . . . . . . . . . . . . . . . . . . . . . . . 93
13.4 O Renascimento da Aritmética . . . . . . . . . . . . . . . . . . . . . . . . 96
MA-14 - Aula 07 . . . . . . . . . . . . . . . . . . . . . . . . . . . . . . . . . . . . . . . . . . . . . 99
15.2 Primos de Fermat e de Mersenne . . . . . . . . . . . . . . . . . . . . . . . 99
MA-14 - Aula 08 . . . . . . . . . . . . . . . . . . . . . . . . . . . . . . . . . . . . . . . . . . . . . 109
Primos de Fermat e de Mersenne . . . . . . . . . . . . . . . . . . . . . . . . . . . . . . . . 109

i
ii

17.1 Primos de Fermat . . . . . . . . . . . . . . . . . . . . . . . . . . . . . . . . 109


17.2 Primos de Mersenne . . . . . . . . . . . . . . . . . . . . . . . . . . . . . . 109
17.3 Teorema da Dirichlet . . . . . . . . . . . . . . . . . . . . . . . . . . . . . . 110
17.4 Problemas . . . . . . . . . . . . . . . . . . . . . . . . . . . . . . . . . . . . 110
Números Perfeitos . . . . . . . . . . . . . . . . . . . . . . . . . . . . . . . . . . . . . . . . . . . 114
18.1 Números Perfeitos . . . . . . . . . . . . . . . . . . . . . . . . . . . . . . . . 114
18.2 Problemas . . . . . . . . . . . . . . . . . . . . . . . . . . . . . . . . . . . . 115
MA-14 - Aula 08 . . . . . . . . . . . . . . . . . . . . . . . . . . . . . . . . . . . . . . . . . . . . . 119
Fatoração do Fatorial em Primos . . . . . . . . . . . . . . . . . . . . . . . . . . . . . . . . 119
17.1 O Teorema de Legendre . . . . . . . . . . . . . . . . . . . . . . . . . . . . 119
17.2 Problemas . . . . . . . . . . . . . . . . . . . . . . . . . . . . . . . . . . . . 120
17.3 Exercícios suplementares . . . . . . . . . . . . . . . . . . . . . . . . . . . . 124
Congruências . . . . . . . . . . . . . . . . . . . . . . . . . . . . . . . . . . . . . . . . . . . . . . . . 125
18.1 Congruências . . . . . . . . . . . . . . . . . . . . . . . . . . . . . . . . . . 125
18.1.1 O Pequeno Teorema de Fermat . . . . . . . . . . . . . . . . . . . . 126
18.2 Problemas . . . . . . . . . . . . . . . . . . . . . . . . . . . . . . . . . . . . 126
18.3 Exercícios suplementares . . . . . . . . . . . . . . . . . . . . . . . . . . . . 133
MA-14 - Aula 09 . . . . . . . . . . . . . . . . . . . . . . . . . . . . . . . . . . . . . . . . . . . . . 141
Aplicações de Congruências . . . . . . . . . . . . . . . . . . . . . . . . . . . . . . . . . . . . . 141
19.1 Aplicações de Congruências . . . . . . . . . . . . . . . . . . . . . . . . . . 141
19.1.1 Regra dos nove fora . . . . . . . . . . . . . . . . . . . . . . . . . . . 142
19.1.2 Representação decimal de número perfeito par . . . . . . . . . . . . 144
19.2 Problemas . . . . . . . . . . . . . . . . . . . . . . . . . . . . . . . . . . . . 146
Os Teoremas de Euler e Wilson . . . . . . . . . . . . . . . . . . . . . . . . . . . . . . . . . . 151
20.1 A Função φ Euler . . . . . . . . . . . . . . . . . . . . . . . . . . . . . . . . 151
20.1.1 O Cálculo de φ(m) . . . . . . . . . . . . . . . . . . . . . . . . . . . 151
20.1.2 Teorema de Euler . . . . . . . . . . . . . . . . . . . . . . . . . . . . 151
20.1.3 Teorema de Wilson . . . . . . . . . . . . . . . . . . . . . . . . . . . 152
20.2 Problemas . . . . . . . . . . . . . . . . . . . . . . . . . . . . . . . . . . . . 152
20.3 Teorema de Wilson . . . . . . . . . . . . . . . . . . . . . . . . . . . . . . . 159
20.4 Problemas suplementares . . . . . . . . . . . . . . . . . . . . . . . . . . . . 165
MA-14 - Aula 10 . . . . . . . . . . . . . . . . . . . . . . . . . . . . . . . . . . . . . . . . . . . . . 123
Congruências Lineares . . . . . . . . . . . . . . . . . . . . . . . . . . . . . . . . . . . . . . . . 123
21.1 Congruências Lineares . . . . . . . . . . . . . . . . . . . . . . . . . . . . . 123
21.1.1 Redução de Congruências . . . . . . . . . . . . . . . . . . . . . . . 124
21.2 Problemas . . . . . . . . . . . . . . . . . . . . . . . . . . . . . . . . . . . . 124
21.3 Teorema Chinês dos Restos . . . . . . . . . . . . . . . . . . . . . . . . . . 128
21.3.1 O Problema de Sun-Tsu . . . . . . . . . . . . . . . . . . . . . . . . 128
iii

21.4 Problemas . . . . . . . . . . . . . . . . . . . . . . . . . . . . . . . . . . . . 129


21.5 Exercícios complementares . . . . . . . . . . . . . . . . . . . . . . . . . . . 136
Aritmética das Classes Residuais . . . . . . . . . . . . . . . . . . . . . . . . . . . . . . . . 138
22.1 Classes Residuais . . . . . . . . . . . . . . . . . . . . . . . . . . . . . . . . 138
22.1.1 Exemplos . . . . . . . . . . . . . . . . . . . . . . . . . . . . . . . . 138
22.1.2 Propriedades da adição . . . . . . . . . . . . . . . . . . . . . . . . . 139
22.1.3 Propriedades da multiplicação . . . . . . . . . . . . . . . . . . . . . 139
22.1.4 Exemplo . . . . . . . . . . . . . . . . . . . . . . . . . . . . . . . . . 141
22.2 Problemas . . . . . . . . . . . . . . . . . . . . . . . . . . . . . . . . . . . . 141
22.3 Exercícios complementares . . . . . . . . . . . . . . . . . . . . . . . . . . . 144
MA-14 - Aula 13 . . . . . . . . . . . . . . . . . . . . . . . . . . . . . . . . . . . . . . . . . . . . . 147
Introdução à Criptografia I . . . . . . . . . . . . . . . . . . . . . . . . . . . . . . . . . . . . . 147
23.1 Criptografia . . . . . . . . . . . . . . . . . . . . . . . . . . . . . . . . . . . 147
Introdução à Criptografia II. . . . . . . . . . . . . . . . . . . . . . . . . . . . . . . . . . . . . 149
24.1 Criptografia . . . . . . . . . . . . . . . . . . . . . . . . . . . . . . . . . . . 149
MA-14 - Aula 01

Semana 05/08 a 11/08

Unidade 1
Divisibilidade

1.2 Divisibilidade: Problemas


Exercício 1.2.1.
Mostre, por indução matemática, que, para todo n ∈ N,

a) 8|32n + 7 b) 9|10n + 3 × 4n+2 + 5

c) 9|n4n+1 − (n + 1)4n + 1 d) 169|33n+3 − 26n − 27

Demonstração.

a) Aplicar indução matemática.

b) Aplicar indução matemática.


Para n = 0 e n = 1 é imediato, a propriedade é verdadeira.
Suponhamos que, para n ≤ h seja verdade que 9|10h + 3 × 4h+2 + 5, logo existe
α ∈ N tal que 10h + 3 × 4h+2 + 5 = 9α
Para h + 1 temos 10h+1 + 3 × 4h+3 + 5 = 10h (9 + 1) + 3 × 4 × 4h+2 + 5 =

= 9 × 10h + [10h + 3 × 4h+2 + 5] + 3 × 3 × 4h+2 = 9β

para algum β ∈ N
Portanto, 9|10n + 3 × 4n+2 + 5 para todo n ∈ N

1
2

c) Aplicando indução sobre n

d) Aplicando indução sobre n

Exercício 1.2.2.
Mostre que, para todo n ∈ N:

a) 9|10n − 1 d) 3|10n − 7n g) 19|32n+1 + 44n+2


b) 8|32n − 1 e) 13|92n − 24n h) 17|102n+1 + 72n+1
c) 53|74n − 24n f ) 6|52n+1 + 1 i) 14|34n+2 + 52n+1

Solução.

a) Temos para todo n ∈ N

10n − 1 = (m(9) + 1)n − 1 = m(9) + 110 − 1 = m(9)

b) Temos para todo n ∈ N

c) Temos para todo n ∈ N que

74n = 492n = (m(53) − 4)2n = m(53) + (−4)2n = m(53) + 24n

Logo 74n − 24n = m(53). Portanto, 53|74n − 24n

d) Temos para todo n ∈ N que

10n − 7n = (m(3) + 1)n − (m(3) + 1)n = m(3) + (1)n − (1)n = m(3)

Portanto, 53|74n − 24n .

e)

f)

g)
3

h)

1)

Exercício 1.2.3.
Sejam a, b ∈ Z.

a) Se a ̸= b, mostre que, para todo n ∈ N, n ≥ 2,

an − bn
= an−1 + an−2 b + an−3 b2 + · · · + abn−2 + bn−1
a−b

b) Se a + b ̸= 0, mostre que, para todo n ∈ N∗ ,

a2n+1 + b2n+1
= a2n − a2n−1 b + a2n−2 b2 + · · · − ab2n−1 + b2n
a+b

c) Mostre que, para todo n ∈ N,

a2n − b2n
= a2n−1 − a2n−2 b + a2n−3 b2 + · · · + ab2n−2 − b2n−1
a+b

Demonstração. a)

Por indução sobre n ≥ 2 quando b ̸= a


a 2 − b2
Se n = 2 temos a2 − b2 = (a − b)(a + b) ⇒ = a + b é verdadeira
a−b
Suponhamos para h ∈ N seja verdade que

ah − bh
= ah−1 + ah−2 b + ah−3 b2 + · · · + abh−2 + bh−1
a−b

Para h + 1 e aplicando a hipótese auxiliar

ah+1 − bh+1 = a(ah − bh ) + bh (a − b) =

ah+1 − bh+1 = a[(a − b)(ah−1 + ah−2 b + ah−3 b2 + · · · + abh−2 + bh−1 )] + bh (a − b) =

ah+1 − bh+1 = (a − b)[a(ah−1 + ah−2 b + ah−3 b2 + · · · + abh−2 + bh−1 ) + bh ] =


ah+1 − bh+1
= ah + ah−1 + ah−2 b + ah−3 b2 + · · · + abh−2 + bh−1 + bh
a−b
4

Portanto, a igualdade é verdadeira para todo n ∈ N, n≥2


Demonstração. b)

Exercício 1.2.4.
Para quais valores de a ∈ N:

a) (a − 2)|a3 + 4 ? b) (a + 3)|a3 − 3 ?
c) (a + 2)|a4 + 2 ? d) (a + 2)|a4 + 2a3 + a2 + 1 ?

Demonstração.

a) Suponhamos que (a − 2)|a3 + 4 então existe β ∈ N tal que a3 + 4 = β(a − 2) isto é


a3 − 8 + 12 = β(a − 2), assim

12
(a − 2)[β − (a2 + 2a + 4)] = 12 ⇒ β − (a2 + 2a + 4) =
a−2

Como, β − (a2 + 2a + 4) ∈ N, temos a = 8, 6, 5, 4, 3


Portanto, a ∈ N que satisfaz (a+2)|a4 +2a3 +a2 +1 são os números a = 8, 6, 5, 4, 3.

b)

c)

d) Suponhamos que (a+2)|a4 +2a3 +a2 +1 então existe β ∈ N tal que a4 +2a3 +a2 +1 =
β(a + 2) isto é a3 (a + 2) + (a2 − 4) + 5 = β(a + 2), assim

5
(a + 2)[β − (a3 + a − 2)] = 5 ⇒ β − (a3 + a − 2) =
a+2

Como, β − (a3 + a − 2) ∈ N, temos a = 3, −1, −3, −7


Portanto, o único a ∈ N que satisfaz (a + 2)|a4 + 2a3 + a2 + 1 é a = 3.

Exercício 1.2.5.
Mostre que, para todos a, m, n ∈ N, m > n > 0 ⇒ (a2 + 1)|(a2 − 1)
n m

Demonstração.

Se m > n, então existe p ∈ N tal que m = n + p, assim a2 = a2


m n+p
observe que 2n+p
é par
m n+p n+p n+p−1 n+p−1 n+p−1 n+p−1
a2 − 1 = a2 − 12 = (a2 + 12 )(a2 − 12 )
m n+p−1 n+p−1 n+p−2 n+p−2 n+p−2 n+p−2
a2 − 1 = β1 × (a2 − 12 ) = β1 × (a2 + 12 )(a2 − 12 )
m n+p−2 n+p−2 n+p−3 n+p−3 n+p−3 n+p−3
a2 − 1 = β2 × (a2 − 12 ) = β2 × (a2 + 12 )(a2 − 12 )
5

.. .. ..
. . .
m n+2 n+2 n+1 n+1 n+1 n+1
a2 − 1 = βp−2 × (a2 − 12 ) = βp−2 × (a2 + 12 )(a2 − 12 )
m n+1 n+1 n n n n
a2 − 1 = βp−1 × (a2 − 12 ) = βp−1 × (a2 + 12 )(a2 − 12 )

Logo, a2 − 1 = βp−1 × (a2 − 12 ) = K × (a2 + 1) para algum K ∈ N.


m n+1 n+1 n

Portanto, para todos a, m, n ∈ N, m > n ⇒ (a2 + 1)|(a2 − 1.


n m

Exercício 1.2.6.
Mostre, para todo n ∈ N, que n2 |(n + 1)n − 1.
Demonstração.

Para todo n ∈ N sabemos pelo binômio de Newton que


n ∑
n−1
(n + 1) = n
Cnn−k nn−k ×1 =n +n×n
k n n−1
+ Cnn−k nn−k × 1k + 1
k=0 k=2


n−1
2 n−2 n−2 2
= n [n +n +n Cnn−k nn−k−2 ] + 1 = m(n2 ) + 1
k=2

Portanto, para todo n ∈ N, temos que n2 |(n + 1)n − 1.

Exercício 1.2.7.
Mostre, para todo a ∈ N, que:
a) 2|a2 − a b) 3|a3 − a c) 5|a5 − a d) 7|a7 − a
Demonstração.

a) Seja N = a2 − a = a(a − 1), Se a ∈ N é par, logo a = 2k, então N = (2k)(2k − 1) =


2(2k 2 − k), assim, 2|a2 − a.

Se a = 2α + 1, então N = (2α + 1)[(2α + 1) − 1] = 2α(2α + 1), logo 2|a2 − a.

Assim, o produto de dois números inteiros consecutivos é múltiplo de 2.

b) Se a|3 o resultado é imediato. Suponhamos que a - 3 e seja o conjunto M = {a, 2a}


então cada um dos elementos de M , diferença con elementos do conjunto P = {1, 2}
em alguma ordem, são divisíveis por 3.

Suponhamos a = 1 + m(3) e 2a = 2 + m(3) então multiplicando estas igualdades


temos 2!a2 = 2! + m(3).

Logo, como 2! não é múltiplo de 3 segue

2!a2 − 2! = m(3) ⇒ a(a2 − 1) = a × m(3) ⇒ a3 − a = m(3)


6

Portanto, 3|a3 − a
Assim, o produto de três números inteiros consecutivos é múltiplo de 3.

c) Se a|5 o resultado é imediato. Suponhamos que a - 5 e seja o conjunto M =


{a, 2a, 3a 4a} então cada um dos elementos de M , diferença con elementos do
conjunto P = {1, 2, 3, 4} em alguma ordem, são divisíveis por 5.
Suponhamos a − 1 = m(5) ⇒ a = 1 + m(5), 2a − 2 = m(5) ⇒ 2a =
2 + m(5), 3a − 3 = m(5) ⇒ 3a = 3 + m(5), 4a − 4 = m(5) ⇒ 4a = 4 + m(5)
de onde 4!a4 = 4! + m(5). Logo, como 4! não é múltiplo de 5 segue

4!a4 − 4! = m(5) ⇒ a(a4 − 1) = a × m(5) ⇒ a5 − a = m(5)

Portanto, 5|a5 − a

d) Suponhamos que a - 7 e seja o conjunto M = {a, 2a, 3a 4a, 5a, 6a} então cada um
dos elementos de M , diferença con elementos do conjunto P = {1, 2, 3, 4, 5, 6} em
alguma ordem, são divisíveis por 7. Logo

6!a6 − 6! = m(7) ⇒ a7 − a = m(7)

Portanto, 7|a7 − a
7

Unidade 2

Divisão Euclidiana

1.2 Divisão Euclidiana: Problemas


Exercício 1.2.1.
Ache .
Solução.

Exercício 1.2.2.
Quais são os números que, quando divididos por 5, deixam resto igual a) à metade do
quociente? b) ao quociente? c) ao dobro do quociente? d) ao triplo do quociente?
Demonstração.

a) Seja D o número procurado, das condições do problema temos

q
D = 5q + ⇒ 2D = 11q ⇒ D = 11β, q = 2β β∈N
2
Os números são: 11, 22, 33, 44

b) Em geral temos D = 5q + r, 0≤r<q


Supor r = q está errado pela definição do algoritmo da divisão. Quando r = 0
temos 0 = 5 × 0 + 0.
Portanto, o zero é o único número.

c)

Exercício 1.2.3.
Seja n um número natural. Mostre que um, e apenas um, número de cada terna abaixo
é divisível por 3. a) n, n + 1, n + 2 b) n, n + 2, n + 4 c) n, n + 10, n + 23 d)
n, n + 1, 2n + 1.
Demonstração.

O conjunto de todos números naturais podemos representar mediante o conjunto


A = { 3k, 3k + 1, 3k + 2, k ∈ N }. Se n = 3k então para todos os 4 exercícios
um, e apenas um, número de cada terna é divisível por 3
8

a) Se n = 3k + 1 então a terna dada podemos escrever na forma 3k + 1, 3k + 2, 3k + 3


logo um, e apenas um, número da terna é divisível por 3.

Se n = 3k + 2 então a terna dada podemos escrever na forma 3k + 2, 3k + 3, 3k + 4


logo um, e apenas um, número da terna é divisível por 3.

Com qualquer das três hipóteses na terna um, e apenas um, número da é divisível
por 3.

b) Se n = 3k + 1 então a terna dada podemos escrever na forma 3k + 1, 3k + 3, 3k + 5


logo um, e apenas um, número da terna é divisível por 3.

Se n = 3k + 2 então a terna dada podemos escrever na forma 3k + 2, 3k + 4, 3k + 7


logo um, e apenas um, número da terna é divisível por 3.

Com qualquer das três hipóteses na terna um, e apenas um, número da é divisível
por 3.

c) Se n = 3k + 1 então a terna dada podemos escrever na forma 3k + 1, 3k + 11, 3k + 24


logo um, e apenas um, número da terna é divisível por 3.

Se n = 3k + 2 então a terna dada podemos escrever na forma 3k + 2, 3k + 12, 3k + 25


logo um, e apenas um, número da terna é divisível por 3.

Com qualquer das três hipóteses na terna um, e apenas um, número da é divisível
por 3.

d) Se n = 3k + 1 então a terna dada podemos escrever na forma 3k + 1, 3k + 2, 6k + 3


logo um, e apenas um, número da terna é divisível por 3.

Se n = 3k + 2 então a terna dada podemos escrever na forma 3k + 2, 3k + 3, 6k + 5


logo um, e apenas um, número da terna é divisível por 3.

Com qualquer das três hipóteses na terna um, e apenas um, número da é divisível
por 3.

Exercício 1.2.4.
a) Mostre que, se um número a não é divisível por 3, então a2 deixa resto 1 na divisão
por 3. b) A partir desse fato, prove que, se a e b são inteiros tais que 3 divide a2 + b2 ,
então a e b são divisíveis por 3.
Demonstração. a)

Se o número não é divisível por três então é da forma a = 3k+1 ou a = 3k+2, k ∈ Z,


logo
a2 = 3k(3k + 2) + 1 ou a2 = 3k(3k + 4) + 4 = 3[3k(3k + 4) + 1] + 1
9

Exercício 1.2.5.
O resto da divisão do inteiro N por 20 é 8. Qual é o resto da divisão de N por 5?
Solução.

Temos N = 20q + 8, isto é N = 5(4q) + 5 + 3 = 5(4q + 1) + 3.


O resto, é 3.

Exercício 1.2.6.
Ache o menor múltiplo de 5 que deixa resto 2 quando dividido por 3 e por 4.
Demonstração.

Seja x o número pedido, então x = 3a + 2 ou x = 4b + 2 para algum a, b ∈ N∗ , logo


3a + 2 = 4b + 2 de onde 3a = 4b assim, a = 4 + 4t e b = 3 + 3t para todo t ∈ N∗

t 0 1 2 3 4 5 6 7 8 9
a 4 8 12 16 20 24 28 32 36 40
b 3 6 9 12 15 18 21 24 27 30
x 14 26 38 50 62 74 86 98 110 122

Logo, o menor número é 50.


10

Problemas Suplementares

2.3 Unidades 1 e 2
Exercício 2.3.1.
Sejam a, b, c ∈ Z e c ̸= 0. Mostre que: ac|bc ⇔ a|b.
Demonstração.

(⇒) Condição necessária.


Seja ac|bc então existe m ∈ N tal que bc = m · ac, logo bc − m · ac = 0 assim,
c(b − ma) = 0.
Se c = 0 nada a concluir. Suponhamos que b − ma = 0 então b = ma e portanto a|b.
(⇐) Condição suficiente.
Suponhamos que a|b então existe β ∈ Z tal que b = βa. Para c ∈ Z temos que bc = βca
de onde ac|bc.
Portanto, ac|bc ⇔ a|b.

Exercício 2.3.2.
(ENC-98)1 A soma de todos os múltiplos de 6 que se escrevem (no sistema decimal)
com dois algarismos é:
(a) 612 (b) 648 (c) 756 (d) 810 (e) 864
Solução.

Os múltiplos de 6 com dois algarismos são: 12, 18, . . . , 96. A soma pedida é

12 + 18 + . . . + 96 = 6(1 + 2 + 3 + 4 + . . . + 16 − 1) =
[ ]
16 × 17
=6 − 1 = 810
2
Resposta d) 810.

Exercício 2.3.3.
Com quanto zeros termina o número 100!?
Solução.: (Primeira)

Da definição de fatorial temos:

100! = 1 × 2 × 3 × 4 × . . . × 98 × 99 × 100

100! = 250 [1 × 3 × 5 × 7 × . . . 97 × 99][1 × 2 × 3 × 4 × . . . × 50]


1
Exame Nacional de Cursos, MEC/INEP.
11

100! = 250 [1×3×5×7×. . . 97×99][225 (1×2×3×4×. . .×25)(1×3×5×7×. . .×47×49)]

100! = 275 [5 × 15 × 25 × 35 × . . . × 85 × 95 × α1 ][(25!)(5 × 15 × 25 × 35 × 45 × β1 ]

100! = 275 [510 (1 × 3 × 5 × 7 × . . . 17 × 19)α2 ][(25!)][55 (1 × 3 × 5 × 7 × 9)β2 ]

100! = 275 · 515 [(1 × 3 × 5 × 7 × . . . 17 × 19)α2 ][(25!)][(1 × 3 × 5 × 7 × 9)β2 ]

100! = 275 · 515 [(5 × 15α3 ][(25!)][(5 × β3 ] = 275 · 518 γ1 × 25! (2.1)

Por outro lado:

25! = 1 × 2 × 3 × 4 × . . . × 24 × 25 = [1 × 3 × 5 × 7 × . . . 23 × 25][212 (1 × 2 × 3 × 4 × . . . × 12)]

25! = 212 [5 × 15 × 25 × α3 ][(2 × 4 × 5 × 6 × 8 × 10 × 12)β3 ]

25! = 212 [54 × α4 ][(210 × 52 )β4 ] = 222 × 56 × γ2

En (2.1)

100! = 275 · 515 [(5 × 15α3 ][(25!)][(5 × β3 ] = 275 · 518 γ1 × (222 × 56 × γ2 )]

Portanto 100! = 297 × 524 × γ, termina em 24 zeros.


Recomendo o site http : //2000clicks.com/M athHelp/BasicF actorialT able.aspx
Solução.: (Segunda)
Seja N = (1)(2) · · · (9)(10)(11) · · · (20) · · · (80) · · · (90)(91) · · · (99)(100) = 100!
Como 10 = 2 × 5 e temos muito mais potências de 2, nossa preocupação será obter ao
máximo as potências de 5 então

N = (1) · · · (5) · · · (10) · · · (15) · · · (20) · · · (80) · · · (85) · · · (90) · · · (95) · · · (100) = 100!

N = · · · (5) · · · [2(5)] · · · [3(5)] · · · [4(5)] · · · [16(5)] · · · [17(5)] · · · (90) · · · (95) · · · [20(5)] = 100!

N = 520 [(1) · · · (5) · · · (10) · · · (15) · (16) · (17) · · · (19) · · · (20)]α = 100!, α∈N

N = 520 [(1) · · · (5) · · · [2(5)] · · · [3(5)] · (16) · (17) · · · (19) · · · [4(5)]]α = 100!, α∈N

N = 524 [(1) · · · (1) · · · [2] · · · [3] · (16) · (17) · · · (19) · · · [4]α = 100!, α∈N

N = 524 224 2k β = 100!, β, k ∈ N

Portanto 100! = 524 224 2k β, termina em 24 zeros.


Exercício 2.3.4.
a) Mostre que o produto de i números naturais consecutivos é divisível por i!.

b) Mostre que 6|n(n + 1)(2n + 1), para todo n ∈ N.


12

Demonstração.

a) Seja n ∈ N tal que n > i e os números consecutivos n, n − 1, n − 2, · · · , n − (i − 1)


na forma decrescente. Seu produto é dado por

N = n(n − 1)(n − 2) · · · (n − i + 1) (2.2)

Sabemos que

n! n(n − 1)(n − 2) · · · (n − i + 1) N
Cni = = = =α∈N
i!(n − i)! i! i!

logo em (2.2) segue N = α × i!


Portanto, o produto de i números naturais consecutivos é divisível por i!

b) Primeira solução: Seja N = n(n + 1)(2n + 1) então temos

N = n(n + 1)[(n − 1) + (n + 2)] = (n − 1)n(n + 1) + n(n + 1)(n + 2)

Temos três números n − 1, n, n + 1 consecutivos, logo (n − 1)n(n + 1) = 6α, α ∈ N.


Também os três números n, n + 1, n + 2 são consecutivos, logo n(n + 1)(n + 2) =
6β, β ∈ N. Isto implica que N = 6(α + β), α, β ∈ N.
Portanto, 6|n(n + 1)(2n + 1), para todo n ∈ N.

b) Segunda solução: Aplicando indução sobre n


Se n = 1, temos (1)(2)(3) = 6 é imediato que 6|n(n + 1)(2n + 1).
Suponhamos que para algum h ∈ N onde h ≤ n cumpra que 6|h(h + 1)(2h + 1), isto
é existe α ∈ N tal que h(h + 1)(2h + 1) = 6α.
Para h + 1 temos

(h + 1)[(h + 1) + 1][2(h + 1) + 1] = (h + 1)(h + 2)[(2h + 1) + 2] ⇔

h(h + 1)(2h + 3) + 2(h + 1)(2h + 3) = h(h + 1)(2h + 1) + 2h(h + 1) + 2(h + 1)(2h + 3)

aplicando a hipótese auxiliar

= 6α + 2(h + 1)[h + (2h + 3)] = 6α + 6(h + 1)2 = 6β

onde β ∈ N.
Portanto, 6|n(n + 1)(2n + 1), para todo n ∈ N.
13

Exercício 2.3.5.
Mostre que 13|270 + 370 .
Demonstração.
Temos: 24 = 13 + 3, 25 = m(13) + 6, 26 = m(13) + 4 = m(13) − 1. Logo,

270 = 24 × (26 )11 = 24 [m(13) − 1]11 = m(13) − 24 = m(13) − 3

Por outro lado, 32 = 13 − 4, 33 = m(13) + 1. então

370 = 3 · (33 )23 = 3 · (m(13) + 1)23 = 3(m(13) + 123 ) = m(13) + 3

Assim, 270 + 370 = [m(13) − 3] + m(13) + 3 = m(13).


Portanto, 13|270 + 370 .
Exercício 2.3.6.
Mostre que existem infinitos valores de n em N para os quais 8n2 + 5 é divisível por 7
e por 11.
Demonstração.
Se o número 8n2 + 5 é divisível por 7 e por 11, logo ele é divisível por 77 (7 e 11 são
coprimos). Suponhamos que 8n2 + 5 é divisível por 77, logo existe β ∈ N tal que

8n2 + 5 = 77β ⇒ 8n2 + 5 − 77 = 77(β − 1) ⇒ 8(n2 − 9) = 77(β − 1)

Como 8 - 77, segue que 8|(β − 1) e 77|(n2 − 9).


Assim, para todo α ∈ N temos β − 1 = 8α e n2 − 9 = 77α, α ∈ N, logo
β = 1 + 8α.
Portanto, 8n2 + 5 = 77(1 + 8α) para todo α ∈ N, assim existem infinitos valores de
n em N para os quais 8n2 + 5 é divisível por 7 e por 11
Exercício 2.3.7.
Ache o quociente e o resto da divisão a) de 27 por 5. b) de 38 por 7.
Solução.

a) 27 = 5(5) + 2 quociente q = 5 e o resto r = 2.

b) 38 = 5(7) + 3 quociente q = 5 e o resto r = 3.

Exercício 2.3.8.
Mostre como, usando uma calculadora que só realiza as quatro operações, pode-se
efetuar a divisão euclidiana de dois números naturais em apenas três passos. Aplique o
seu método para calcular o quociente e o resto da divisão de 3721056 por 18735.
Demonstração.
14

Exercício 2.3.9.
Discuta a paridade a) da soma de dois números. b) da diferença de dois números.
c) do produto de dois números. d) da potência de um número. e) da soma de n números
ímpares.
Solução.

a) A soma de dois números pares ou ímpares sempre é par. A soma de um número par
com outro ímpar sempre é ímpar.

b) A diferença de dois números pares ou ímpares sempre é par. A diferença de um número


par com outro ímpar sempre é ímpar.

c) O produto de dois números sendo um deles par, sempre é par. O produto de dois
números ímpares sempre é ímpar.

d) A potência de um número par é par. A potência de um número ímpar sempre é ímpar.

e) A soma de n números ímpares será par se n é par, e será ímpar se n é ímpar.

Exercício 2.3.10.
a) Mostre que um número natural a é par se, e somente se, an é par, qualquer que
seja n ∈ N∗ . b) Mostre que an ± am é sempre par, quaisquer que sejam n, m ∈ N∗ . c)
Mostre que, se a e b são ímpares, então a2 + b2 é divisível por 2 mas não divisível por 4.
Demonstração.

a) Suponhamos a seja par, então a = 2k, k ∈ Z logo an = (2k)n = 2(2n−1 k n ) é par.


Inversamente
A mostrar que, se an é par, então a é par.
Por contradição
Suponhamos a não seja par, logo a = 2k + 1, k ∈ Z de onde


n ∑
n−1
an = (2k + 1)n = Cnj (2k)n−j 1j = Cnj (2k)n−j + 1 = 2α + 1, α∈Z
j=0 j=0

logo an é ímpar.
Isto mostra que sean é par, então a é par.
Portanto, um número natural a é par se, e somente se, an é par, qualquer que seja
n ∈ N∗

b)
15

c)

Exercício 2.3.11.
Mostre que a) se n é ímpar, então n2 − 1 é divisível por 8. b) se n não é divisível por
2, nem por 3, então n2 − 1 é divisível por 24. c) ∀ n ∈ N, 4 - n2 + 2.
Demonstração.

a) Se n é ímpar, então é da forma 2k +1, k ∈ N. Logo, como o produto de dois números


naturais consecutivos sempre é par, temos

n2 − 1 = (2k + 1)2 − 1 = 4k 2 + 4k = 4k(k + 1) = 8β, β∈N

Portanto, n2 − 1 é divisível por 8

b)

c) Todo natural n ∈ N podemos escrever em alguma das formas dos elementos do conjunto
{ 5k, 5k + 1, 5k + 2, 5k + 3, 5k + 4 } onde k ∈ N. Logo

• Se n = 5k, então n2 = (5k)2 = m(5)


• Se n = 5k + 1, então n2 = (5k + 1)2 = m(5) + 1
• Se n = 5k + 2, então n2 = (5k + 2)2 = m(5) + 4
• Se n = 5k + 3, então n2 = (5k + 3)2 = m(5) + 32 = m(5) + 4
• Se n = 5k + 4, então n2 = (5k + 4)2 = m(5) + 42 = m(5) + 1

Assim o quadrado de qualquer número natural é da forma 5k ou 5k + 1 ou 5k + 4


De onde n2 +4 = 5h+4 ou n2 +4 = (5k +1)+4 = 5(k +1) ou n2 +4 = (5k +4)+4 =
5(k + 1) + 3
Portanto, ∀ n ∈ N, 4 - n2 + 2

Exercício 2.3.12.
Sejam dados os números naturais a, m e n tais que 1 < a < m < n. a) Quantos
múltiplos de a existem entre m e n? b) Quantos múltiplos de 7 existem entre 123 e
2551? c) Quantos múltiplos de 7 existem entre 343 e 2551?
Demonstração.
16

Exercício 2.3.13.
(ENC-2000) Mostre que, se um inteiro é, ao mesmo tempo, um cubo e um quadrado,
então ele é da forma 5n, 5n + 1, ou 5n + 4.
Demonstração.

Seja n ∈ N, consideremos os números n2 e n3 , pelos dados do problema temos N =


(n2 )3 = (n3 )2 = n6 , isto é o número procurado é potência sexta de n.
Pela parte c) do exercício (1.2.7) sabemos que 5|n5 − n ⇒ n5 − n = m(5), de onde
n(n5 − n) = m(5) ⇒ n6 = n2 + m(5)
Mostramos na parte c) do exercicio (12.3.7) que o quadrado de qualquer número
natural é da forma 5k ou 5k + 1 ou 5k + 4.
Assim temos que n6 = n2 + m(5), logo N = n6 = 5k + m(5) = m(5) ou

N = n6 = 5k + 1 + m(5) = m(5) + 1, ou N = n6 = (5k + 4) + m(5) = m(5) + 4

Portanto, se um inteiro é, ao mesmo tempo, um cubo e um quadrado, então ele é da


forma 5n, 5n + 1, ou 5n + 4.

Exercício 2.3.14.
(ENC-2001) Seja N um número natural; prove que a divisão de N 2 por 6 nunca deixa
resto 2.
Demonstração.

O conjunto dos números naturais podemos identificar com elementos do conjunto

A = { 6m, 6m + 1, 6m + 2, 6m + 3, 6m + 4, 6m + 5, m∈N}≡N

logo qualquer número natural tem uma dessas formasm assim,

• Se N = 6m ⇒ N 3 = 6(6m2 ) + 0 o resto r = 0.

• Se N = 6m + 1 ⇒ N 2 = 6(6m2 + 2m) + 1 o resto r = 1.

• Se N = 6m + 2 ⇒ N 2 = 6(6m2 + 4m) + 4 o resto r = 4.

• Se N = 6m + 3 ⇒ N 2 = 6(6m2 + 6m + 1) + 3 o resto r = 3.

• Se N = 6m + 4 ⇒ N 2 = 6(6m2 + 8m + 2) + 4 o resto r = 4.

• Se N = 6m + 5 ⇒ N 2 = 6(6m2 + 10m + 4) + 1 o resto r = 1.

Portanto, a divisão de N 2 por 6 nunca deixa resto 2.


17

Exercício 2.3.15.
Mostre que, se n é ímpar, então a soma de n termos consecutivos de uma P A é sempre
divisível por n.
Solução.

Exercício 2.3.16.
Mostre, para todo n ∈ N, que

a) 6|n3 + 11n b) 9|4n + 15n − 1 c) 3n+2 |103n − 1


d) 7|23n − 1 e) 8|32n + 7 f ) 7|32n+1 + 2n+2

g) a2 − a + 1|a2n+1 + (a − 1)n+2 ∀a∈N


Solução.

a) Por indução sobre n.


Se n = 1 temos 13 + 11(1) = m(6)
Suponhamos para h ∈ N cumpra h3 + 11h = m(6).
Para h + 1 ∈ N e pela hipótese indutiva

(h + 1)3 + 11(h + 1) = (h3 + 11h) + 3h2 + 3h + 12 = m(6) + 3h(h + 1) (2.3)

Se h = 2k-par, em 3h(h + 1) = 6k(2k + 1) = m(6)


Se h = 2k + 1-ímpar, em 3h(h + 1) = 6(2k + 1)(k + 1) = m(6)
Logo (h + 1)3 + 11(h + 1) = m(6)
Portanto, 6|n3 + 11n.

b)

c)

d) Para todo n ∈ N temos

23n − 1 = (23 )n − 1 = (m(7) + 1)n − 1 = m(7) + 1n − 1 = m(7)

Portanto, 7|23n − 1.
18

e) Para todo n ∈ N temos

32n + 7 = (32 )n + 7 = (m(8) + 1)n + 7 = m(8) + 1n + 7 = m(8)

Portanto, 8|32n + 7.

f ) Para todo n ∈ N temos

32n+1 + 2n+2 = 3(32 )n + 4 · 2n = 3(m(7) + 2)n + 4 · 2n =

= 3[m(7) + 2n ] + 4 · 2n = m(7) + (3 + 4)2n = m(7)

Portanto, 7|32n+1 + 2n+2 .

g)

Exercício 2.3.17.
Mostre que, se um inteiro é um quadrado e um cubo, então é da forma 7k ou 7k + 1.
Solução.

Seja a ∈ Z, e N um inteiro que é um quadrado e um cubo, logo podemos escrever na


forma N = (a3 )2 ou N = (a2 )3 assim temos que N = a6 .

• Se a = m(7) então N = a6 = m(7)

• Suponhamos que a - 7 e seja o conjunto M = {a, 2a, 3a 4a, 5a, 6a} então cada um
dos elementos de M , diferença con elementos do conjunto P = {1, 2, 3, 4, 5, 6} em
alguma ordem, são divisíveis por 7. Logo

6!a6 − 6! = m(7) ⇒ a6 − 1 = m(7) ⇒ N = a6 = 1 + m(7)

Portanto, se um inteiro é um quadrado e um cubo, então é da forma 7k ou 7k + 1.

Exercício 2.3.18.

a) Mostre que um quadrado perfeito ímpar é da forma 4n + 1.

b) Mostre que nenhum elemento da sequência 11, 111, 1111, . . . , é um quadrado perfeito.

Demonstração.

a) Só iremos considerar números ímpares, para o caso dos pares, seu quadrado sempre
será número par,
Seja 2k + 1, k ∈ Z qualquer número ímpar, seu quadrado é da forma N = 4k 2 +
4k + 1 = 4n + 1, isto é um quadrado perfeito ímpar é da forma 4n + 1.
19

b) Seja N = 1111 . . . 11 número com n > 1 algarismos, e suponhamos que N seja um


quadrado perfeito, isto é, existe K ∈ N tal que N = K 2 . Temos

N = 1111 . . . 11 = 10n−1 + 10n−2 + 10n−3 + · · · + 102 + 10 + 1 ⇒

10n − 1
N= = K2 ⇒ 10n − 1 = 9K 2 ⇒
10 − 1
10n − 1 = 9K 2 ⇒ 10n − 1 = (10 − 1)K 2 ⇒ 10(10n−1 − K 2 ) = 1 − K 2

Como K ∈ N, temos que 1 − K 2 é múltiplo de 10, isto acontece somente se K = 1


e conse1uentemente n = 1 sendo um absurdo! Isto do fato supor que N seja um
quadrado perfeito.
Portanto, nenhum elemento da sequência 11, 111, 1111, . . . , é um quadrado perfeito.

Exercício 2.3.19.

a) Mostre que todo quadrado perfeito é da forma 5k ou 5k ± 1.

b) Com que algarismo pode terminar um quadrado perfeito?

c) Se três inteiros positivos verificam a2 = b2 + c2 , então entre eles há um múltiplo de 2


e um múltiplo de 5.

d) A soma dos quadrados de dois inteiros ímpares não pode ser um quadrado perfeito.

Demonstração.

a) Seja n = . . . a2 a1 a0 , isto podemos escrever na forma n = 10(. . . + 10a2 + a1 ) + a0 o


quadrado deste número é

n2 = 102 (. . . + 10a2 + a1 )2 + 20a0 (. . . + 10a2 + a1 ) + a20 = m(5) + a20

onde m(5) indica múltiplo de 5

n2 = m(5) + a20 (2.4)

Na igualdade (2.4) se a0 = 0 ou a0 = 5 então a20 = m(5) logo n2 = m(5).


Na igualdade (2.4) se a0 = 1, 4, 6 ou a0 = 9 então a20 = m(5)+1, logo n2 = m(5)+1.
Na igualdade (2.4) se a0 = 2, 3, 7 ou a0 = 8 então a20 = m(5) + 4 = m(5) − 1, logo
n2 = m(5) − 1.
Portanto, todo quadrado perfeito é da forma 5k ou 5k ± 1.
20

b) Pela parte a) um quadrado perfeito pode terminar em 0, 1, 4. 6 ou 9.

c) Se três inteiros positivos verificam a2 = b2 + c2 , então para b ∈ Z, b > 0 temos


c = b − 1 e a = b + 1, logo

a2 = b2 + c2 ⇔ (b + 1)2 = b2 + (b − 1)2 ⇔ b(b − 4) = 0

logo, b = 4, c = 3 e a = 5.
Portanto, entre eles há um múltiplo de 2 e um múltiplo de 5.

d) Sejam α, β ∈ Z, α ̸= β e consideremos os números 2α + 1 e 2β + 1 então

N = (2α + 1)2 + (2β + 1)2 = 4(α2 + α + β 2 + β) + 2

N = (5 − 1)(α2 + α + β 2 + β) + 2 = m(5) + 2 − (α2 + α + β 2 + β)

pela parte a) do Exercício (1.2.7) temos α2 = m(2) + α e β 2 = m(2) + β

N = m(5) + 2 − [m(2) + 2α + m(2) + 2β] = m(5) + m(2)

Pela parte a) deste Exercício N não pode ser um quadrado perfeito.


Portanto, a soma dos quadrados de dois inteiros ímpares não pode ser um quadrado
perfeito.

Exercício 2.3.20.
Mostre que, de n inteiros consecutivos, um, e apenas um, deles é divisível por n.
Demonstração.

Exercício 2.3.21.
Um número é dito livre de quadrados se não for divisível pelo quadrado de nenhum
número diferente de 1.

a) Determine qual é o maior número de números naturais consecutivos livres de quadra-


dos.

b) Defina números livres de cubos e resolva o problema correspondente.


21

Solução.

Exercício 2.3.22.
Seja m ∈ N. Pode o número m(m + 1) ser a sétima potência de um número natural?
(generalize).
Solução.

Exercício 2.3.23.
Dados a; b ∈ N, quantos números naturais divisíveis por b existem na sequência
a; 2a; . . . ba?
Solução.

Exercício 2.3.24.
Sejam a; d ∈ N. Mostre que, na sequência a + 0d; a + d; a + 2d; a + 3d; . . . ou não existe
nenhum quadrado ou existem infinitos quadrados.
Solução.
22
MA-14 - Aula 02

Semana 12/08 a 18/08

Unidade 3

3.1 Sistemas de Numeração: Problemas


Exercício 3.1.1.
Um certo número de três algarismos na base 10 aumenta de 36 se permutarmos os dois
algarismos da direita, e diminui de 270 se permutarmos os dois algarismos da esquerda.
O que acontece ao número se permutarmos os dois algarismos extremos?
Demonstração.

Seja N = abc o número, pelas hipóteses abc = acb − 36 e abc = bac + 270, logo

102 a + 10b + c = 102 a + 10c + b − 36 ⇒ c−b=4 (3.5)


102 a + 10b + c = 102 b + 10a + c + 270 ⇒ a−b=3 (3.6)

Das igualdades (3.5) e (3.6) segue que c = a + 1.


Ao permutar os dois algarismos extremos

cba = 100c + 10b + a = 100(a + 1) + 10b + (c − 1) = 102 a + 10b + c + 99

Resulta cba = abc + 99, o número aumenta em 99 unidades.

Exercício 3.1.2. Critério de divisibilidade por uma potência de 2


Seja dado um número a, representado na base 10 por a = an an−1 . . . a0 . Usando o fato
de que 2k |10k , mostre que 2k divide a se, e somente se, o número ak−1 . . . a1 a0 é divisível
por 2k . Em particular, a é divisível por 2 se, e somente se, a0 é 0, 2, 4, ∥ : 6 ou 8; também,
a é divisível por 4 se, e somente se, a1 a0 é divisível por 4.
Demonstração.

23
24

Como 2k |10k então existe β ∈ N tal que 10k = β · 2k , e de a = an an−1 . . . a0 , então

a = 10n an + 10n−1 an−1 + . . . + 102 a2 + 10a1 + 100 a0

Suponhamos que o número ak−1 . . . a1 a0 seja divisível por 2k , logo

10k−1 ak−1 + . . . + 102 a2 + 10a1 + a0 = γ2k

então

10k ak + (10k−1 ak−1 + . . . + 102 a2 + 10a1 + a0 ) = γ2k + 10k ak = 2k (γ + 5k ak )

Portanto, 2k divide a.
Inversamente
Se 2k divide a, então a = α2k , assim

10k ak + 10k−1 ak−1 + . . . + 102 a2 + 10a1 + a0 = α · 2k

então
10k−1 ak−1 + . . . + 102 a2 + 10a1 + a0 = α · 2k − 10k ak = 2k (α − 5k ak )

Portanto, o número ak−1 . . . a1 a0 é divisível por 2k


Em particular, quando k = 1 temos que a é divisível por 21 , isto é a0 = γ · 2, logo a0
é 0, 2, 4, 6 ou 8.
Em particular, quando k = 2 temos que a = a1 a0 é divisível por 22 , isto é a1 a0 =
γ · 22 = 4γ, logo o número a1 a0 tem que ser divisível por 4.

Exercício 3.1.3.
Escolha um número abc de três algarismos no sistema decimal, de modo que os algaris-
mos das centenas a e o das unidades c difiram de, pelo menos, duas unidades. Considere
os números abc e cba e subtraia o menor do maior, obtendo o número xyz. A soma de
xyz com zyx vale 1089. Justifique este fato.
Solução.

Seja N = abc o número e suponhamos que N seja maior do que M = cba, logo a > c.
Assim, a = c + K onde k = 2, 3, 4, 5, 6 ou 7. Logo, P = N − M = xyz

xyz = N −M = abc−cba = (102 a+10b+c)−(102 c+10b+a) = 102 (a−c)+0×10+(c−a)

assim, xyz = 102 · K − K, de onde xyz = [102 (K − 1) + 100 − K = 102 (K − 1) + 10 ×


9 + (10 − K) consequentemente zyx = 102 (10 − K) + 10 × 9 + (K − 1)
25

Somando, xyz + zyx = [102 (K − 1) + 10 × 9 + (10 − K)] + [102 (10 − K) + 10 × 9 +


(K − 1)] = 9 × 102 + 180 + 9 = 1089.
Este fato acontece, porque o resultado é independente do valor dado para k = 2, 3, 4, 5, 6
ou 7.
Exercício 3.1.4.
Seja dado o número 4783 na base 10; escreva-o nas seguintes bases: 2, 3, 4, 7, 12 e
15.
Solução.

Exercício 3.1.5.
O número 3416 está na base 7; escreva-o nas bases 5 e 12.
Solução.

34167 = 3 × 73 + 4 × 72 + 1 × 7 + 6 = 123810 = 144235 = 87212

Exercício 3.1.6.
Um número na base 10 escreve-se 37; em que base escrever-se-á 52?
Solução.
Suponhamos se escreva na base a, então

3710 = 52a ⇒ 37 = 5a + 2 ⇒ a=7

Na base sete.
Exercício 3.1.7.
Considere 73 na base 10; em que base ele se escreverá 243?
Solução.
Suponhamos se escreva na base a, então

7310 = 243a ⇒ 73 = 2a2 + 4a + 3 ⇒ a=5

Na base cinco.
26

Exercício 3.1.8.
Escreva a tabuada na base 5. Use-a para calcular 132 + 413 e 23 × 342.
Solução.

1×1 = 1 2×1 = 2 3×1 = 3 4×1 = 4


1×2 = 2 2×2 = 4 3×2 = 11 4×2 = 13
1×3 = 3 2×3 = 11 3×3 = 14 4×3 = 22
1×4 = 4 2×4 = 13 3×4 = 22 4×4 = 31

Assim temos
3 4 2 ×
1 3 2 + 2 3
4 1 1 2 1 3 1
1 0 4 3 1 2 3 4
2 0 0 2 1
Exercício 3.1.9.
Utilize o método dos antigos egípcios para calcular 527 × 72.
Solução.

1 −→ 527
2 −→ 1054
4 −→ 1581
8 −→ 2108
16 −→ 2635
32 −→ 3164
64 −→ 3689
64 + 8 = 72 −→ 3689 + 2108 = 5797
Assim, 527 × 72 = 5797

Exercício 3.1.10.
Escreva:
a) O número 2n − 1 na base 2.
bn − 1
b) O número O número na base b.
b−1
Demonstração.

Exercício 3.1.11.
Sendo a = [an . . . a1 a0 ]b, mostre que o número a − (a0 + . . . + an ) é divisível por b − 1.
Demonstração.
27

Exercício 3.1.12.
Mostre que, na base 10, o algarismo das unidades de um quadrado perfeito só pode ser
0, 1, 4, 5, 6 ou 9.
Demonstração.

Seja n ∈ N um número na base 10, e denotemos m(5) o múltiplo de 5, então

n = an . . . a1 a0 = 10n an + · · · + 10a1 + a0 = m(5) + a0

ao quadrado n2 = [m(5) + a0 ]2 = m(5) + a20 como a0 é o algarismo das unidades, temos

a0 = 1 ou 4 ou 6 ou 9 ⇒ a20 = m(5) + 1; a0 = 5 ⇒ a20 = m(5)

a0 = 2 ou 3 ou 7 ou 8 ⇒ a20 = m(5) + 4

Como a20 = m(5), m(5) + 1, m(5) + 4 segue que o algarismo das unidades de um
quadrado perfeito só pode ser 0, 1, 4, 5, 6 ou 9.
28

Unidade 4
Jogo de Nim

4.2 Jogo de Nim: Problemas


Exercício 4.1.1.
Determine, em cada caso apresentado abaixo, se a posição é segura ou insegura.
Demonstração.

a) | | | |

b) ||| | | | |

c) | | | |

d) | |

Exercício 4.1.2.
Determine qual das seguintes situações iniciais no Jogo de Nim permite ao primeiro
jogador traçar uma estratégia vencedora.
a) (12; 14; 15), b) (7; 9; 14), c) (7; 9; 15; 17).
Demonstração.

Exercício 4.1.3.
Demonstre que as afirmações feitas na variante 3 do jogo de Nim são verdadeiras.
Demonstração.
MA-14 - Aula 03

Semana 19/08 a 25/08

Unidade 5

5.2 Máximo Divisor Comum: Problemas


Exercício 5.2.1.
Para cada par de números naturais a e b dados abaixo, ache (a, b) e determine números
inteiros m e n tais que (a, b) = na + mb.
a) 637 e 3887 b) 648 e 1218 c) 551 e 874
d) 7325 e 8485 e) 987654321 e 123456789
Solução.

a) 3887 = 6 × 637 + 52, 637 = 9 × 65 + 53, 65 = 1 × 52 + 13, logo

13 = 65 − 1 × 52 = 65 − 1(637 − 9 × 65)

13 = 10 × 65 − 637 = 10(3887 − 6 × 637) − 635 = 10 × 3887 − 61 × 637

Portanto, (637, 3887) = 13, m = −61 e n = 10.

b) 1218 = 648 × 1 + 570


Logo, 648 × 47 − 1218 × 25 = 6
Portanto, (648, 1218) = 13, m = 47 e n = −25.

c) Aplicando o Lema 1 de Euclides temos

(874, 551) = (874 − 551, 551) = (323, 551) = (323, 551 − 323) = (323, 228) =

= (323, 228) = (323 − 228, 228) = (95, 228) = (95, 228 − 2 × 95) = (95, 38) =

= (95−38, 38) = (57, 38) = (57−38, 38) = (19, 38) = (19, 38−2×19) = (19, 0) = 19

29
30

assim, (874, 551) = 19

Isto é 19 = 19(46, 29) ⇒ 1 = (46, 29) ⇒ 1 = 29 × 8 − 46 × 5, logo


concluímos que 19 = 551 × 8 − 874 × 5

Portanto, (874, 551) = 19, m = 8 e n = −5.

d) 7325 e 8485

e) 987654321 e 123456789

Exercício 5.2.2.
Seja n ∈ N. Mostre que:
a) (n, 2n + 1) = 1 b) (n + 1, n2 + n + 1) = 1
c) (2n + 1, 9n + 4) = 1 d) (n! + 1, (n + 1)! + 1) = 1
Demonstração.

a) Primeira solução: Observe que 2n + 1 > 2n, pelo algoritmo de Euclides segue que
2n + 1 = 1 × 2n + 1, logo 1 = 1 × (2n + 1) − (1) × 2n. Então o mdc{2n + 1, 2n} = 1
são coprimos.

Portanto (n, 2n + 1) = 1

a) Segunda solução: Suponhamos que (n, 2n + 1) = d ⇒ d|n e d|2n + 1

Como d|n ⇒ d|2n, assim temos que 2n = αd e como também d|2n + 1 logo
2n + 1 = βd para α, β ∈ N.

Logo αd = βd + 1 ⇒ 1 = d(α − β) de onde d = 1.

Portanto (n, 2n + 1) = 1

b) Pelo Lema 1 (Lema de Euclides) segue

(n + 1, n2 + n + 1) = (n + 1, n(n + 1) + 1) = (n + 1, 1)

O máximo divisor comum de dois naturais consecutivos (n + 1, 1) = 1

Portanto (n, 2n + 1) = 1.

Portanto (n, 2n + 1) = 1.
31

c) Primeira solução: Observe que 9n + 4 > 2n + 1, pelo algoritmo de Euclides segue


que 9n + 4 = 4 × (2n + 1) + (n + 0) então 2n + 1 = 2 × n + 1, logo 1 = 1 ×
(2n + 1) + (−2) × n ⇒ 1 = 1 × (2n + 1) + (−2)[9n + 4 − 4 × (2n + 1)], isto é
1 = (−2) × (9n + 4) + 9 × (2N + 1).
Portanto, mdc{2n + 1, 9n + 4} = 1.

c) Segunda solução: Suponhamos (2n + 1, 9n + 4) = d então d|2n + 1 e d|9n + 4 logo


d|(9n + 4) − 4(2n + 1), assim d|n
Temos d|2n + 1 e d|n pela parte a) segue que d = 1.
Portanto, mdc{2n + 1, 9n + 4} = 1.

d) (n! + 1, (n + 1)! + 1) = 1
Pelo Lema 1 (Lema de Euclides) segue

(n! + 1, (n + 1)! + 1) = (n! + 1, n × n!)

A mostrar que (n! + 1, n × n!) = 1


Pelo Lema 1 (Lema de Euclides) segue

(n! + 1, n × n!) = (n! + 1, n(n! + 1) − n) = (n! + 1, −n) = 1

Exercício 5.2.3.
Mostre que (a, a2 + na + b)|b, quaisquer que sejam a, b, n ∈ N.
Demonstração.

Para qualquer a, b, n ∈ N, suponhamos que

d = (a, a2 + na + b) ⇒ d|a e d|a2 + na + b

além disso existem números inteiros x, y ∈ Z tais que d = a · x + y(a2 + na + b).


Como d|a então existe c ∈ N tal que a = d · c. Também d|a2 + na + b então existe
β ∈ N tal que a2 + na + b = β · d de onde

b = β · d − a2 − na = βd − d2 c2 − ndc = d(β − dc2 − nc) ⇒ d|b

isto é (a, a2 + na + b) = d|b


Portanto, (a, a + b)|b, quaisquer que sejam a, b ∈ N

Exercício 5.2.4.
Dados a ∈ Z r {−1}, mostre que
32
( )
a2m − 1
a) Se m ∈ N, mostre que: , a + 1 = (a + 1, 2m)
a+1
( 2m+1 )
a +1
b) Se m ∈ N ∪ {0}, mostre que: , a + 1 = (a + 1, 2m + 1)
a+1
Demonstração. a)

Como a + 1|a2m − 1 então a2m − 1 = (a + 1)[a2m−1 − a2m−2 + . . . − a2 + a − 1]

a2m − 1 = (a + 1)[(a2m−1 + 1) − (a2m−2 − 1) + . . . − (a2 − 1) + (a + 1) + 2m]

a2m − 1
= n(a + 1) + 2m para algúm n ∈ N
a+1
Assim, ( )
a2m − 1
, a+1 = (n(a + 1) + 2m, a + 1)
a+1
Pelo Lema 1 (Lema de Euclides)
( )
a2m − 1
, a + 1 = (2m, a + 1)
a+1

Demonstração. b)
Como a + 1|a2m+1 + 1 então a2m + 1 = (a + 1)[a2m − a2m−1 + . . . + a3 − a2 − a + 1]

a2m+1 + 1 = (a + 1)[(a2m − 1) − (a2m−1 + 1) + . . . − (a3 + 1) + (a2 − 1) − (a + 1) + 2m + 1]

a2m+1 − 1
= n(a + 1) + 2m + 1 para algúm n ∈ N
a+1
Assim, ( )
a2m+1 + 1
,a + 1 = (n(a + 1) + 2m + 1, a + 1)
a+1
Pelo Lema 1 (Lema de Euclides)
( )
a2m+1 + 1
,a + 1 = (2m + 1, a + 1)
a+1

Exercício 5.2.5.
( ) ( )
340 − 1 5 510 − 1
Calcule a) , 3 −1 b) ,6
35 − 1 6
( ) ( )
240 + 1 8 250 + 1 10
c) , 2 +1 d) , 2 +1
28 + 1 210 + 1

Solução. a)
33
( ) ( 5 8 )
340 − 1 5 (3 ) − 1 5
Temos, , 3 −1 = , 3 − 1 , por outro lado
35 − 1 35 − 1

(35 )8 − 1 = (35 − 1)((35 )7 + (35 )6 + (35 )5 + (35 )4 + (35 )3 + (35 )2 + (35 ) + 1)

(35 )8 −1 = (35 −1)((35 )7 −1+(35 )6 −1+(35 )5 −1+(35 )4 −1+(35 )3 −1+(35 )2 −1+(35 )−1+8)

(35 )8 − 1 = (35 − 1)[β(35 − 1) + 8] β∈N


( 40 )
3 −1 5 ( 5 ) ( )
, 3 − 1 = β(3 − 1) + 8, 35
− 1 = 8, 35
− 1
35 − 1
Solução. b)
( ) ( 10 )
510 − 1 5 −1
Temos, ,6 = , 5 + 1 , aplicando o Lema de Euclides segue que
6( 5+) 1
510 − 1
segue que m = 5 e , 5 + 1 = (2 × 5, 5 + 1) = 2
5+1
Solução. c)
( 40 ) ( 8 5 )
2 +1 8 (2 ) + 1 8
Temos, , 2 +1 = , 2 + 1 , aplicando a parte b) do exercício
28 + 1 28 + 1
anterior segue que m = 2

( ) ( 8 5 )
240 + 1 8 (2 ) + 1 8
, 2 +1 = , 2 + 1 = (2 × 2 + 1, 28 + 1) = (5, 257) = 1
28 + 1 28 + 1

Solução. d)
( ) ( 10 5 )
250 + 1 10 (2 ) + 1 10
Temos, ,2 + 1 = , 2 + 1 , aplicando a parte b) do exer-
210 + 1 210 + 1
cício anterior segue que m = 2

( ) ( 10 5 )
250 + 1 10 (2 ) + 1 10
, 2 +1 = , 2 + 1 = (2 × 2 + 1, 210 + 1) = (5, 1025) = 5
210 + 1 210 + 1

Exercício 5.2.6.
Sejam a e n números naturais com a ̸= 1. Mostre que.

(a − 1)2 |an − 1 ⇔ a − 1|n

Solução.

Temos (a − 1)2 |an − 1 então an − 1 = α(a − 1)2 , α ∈ N, por outro lado

an − 1 = (a − 1)(an−1 + · · · + a + 1) = (a − 1)[(a − 1)β + n], β∈N


34

Logo, α(a − 1)2 = (a − 1)[(a − 1)β + n] ⇒ (a − 1)2 [α − β) = (a − 1)n ⇒


(a − 1)[α − β) = n ⇒ (a − 1)|n
35

Unidade 6

Propriedades do mdc

6.2 Propriedades do mdc: Problemas


Exercício 6.2.1.
Sejam a; b; d ∈ Z com d ≥ 0. Mostre que se I(a; b) = dZ, então d = (a; b).
Solução.

Exercício 6.2.2.
Mostre que:

a) Se (a, b) = 1, a|c e b|c, então ab|c.

b) Mostre que, se (a, b) = 1, então (a · c, b) = (c, b).

c) Mostre que (a · c, b) = 1 se, e somente se, (a, b) = (c, b) = 1

d) (a, b) = (a, d) = (c, b) = (c, d) = 1 se e somente se (a · c, b · d) = 1

e) Se (c, d) = 1, então (an , bm ) = 1, ∀ n, m ∈ N.

Demonstração.

a) Como (a, b) = 1, a|c e b|c então existem x, y, α, β ∈ Z tais que

a · x + b · y = 1, c = β · a, c=α·b

Multiplicando a primeira igualdade por c e, logo substituindo os valores de c corres-


pondentes, segue

a·x·c+b·y·c=c ⇒ a · x · (α · b) + b · y · (β · a) = c

isto é ab(xα + yβ) = c.


Portanto, ab|c.

b) Da hipótese (a, b) = 1, logo existem x, y ∈ Z tais que 1 = ax + by.


Suponhamos d1 = (a · c, b) e d2 = (c, b)
Como d1 = (a · c, b) então existem α, β ∈ N tais que a · c = αd1 e b = βd1 .
Como d2 = (c, b) então existem γ, ρ ∈ N tais que c = γd2 e b = ρd2 .
36

Como 1 = ax + by, logo c = acx + bcy. Substituindo as relações acima, segue que
c = (αd1 )x − (βd1 )cy ⇒ d1 |c

Sendo que d1 |c e d1 |b então d1 |(c, b) = d2 .

Por outro lado,

Como d2 |c e d2 |b então d2 |ac e d2 |b, logo d2 |(ac, b) = d1

Assim, como d1 , d2 ∈ N e d1 |d2 e d2 |d1 ⇒ d1 = d2

Portanto, se (a, b) = 1, então (a · c, b) = (c, b)

c) Seja (a · c, b) = 1, então existem β, ρ ∈ Z tais que 1 = β(a · c) + µb

Suponhamos que d1 = (a, b) e d2 = (c, b), então d1 |a, d1 |b e d2 |c, d2 |b e existem


γ, ρ, η, ω ∈ N tais que a = γd1 , b = ρd1 e c = ηd2 , b = ωd2

Como 1 = β(a · c) + µb ⇒ 1 = β(γd1 · c) + µρd1 ⇒ d1 |1

Como 1 = β(a · c) + µb ⇒ 1 = β(a · ηd2 ) + µωd2 ⇒ d2 |1. De d1 |1, d2 |1 ⇒


d1 = d1 = 1.

Portanto, se (a · c, b) = 1, então (a, b) = (c, b) = 1

Inversamente.

Seja (a, b) = (c, b) = 1 e suponhamos que (a · c, b) = d, então d|(a · c) e d|b, logo


(a · c) = αd e b = βd, para algum α, β ∈ N.

Como (a, b) = (c, b) = 1 então existem x, y, m, n ∈ N tais que

1 = ax + by ⇒ c = acx + bcy ⇒ c = (αd)x + (βd)cy ⇒ d|c

1 = mc + nb ⇒ a = acm + ban ⇒ a = (αd)m + (βd)an ⇒ d|a

Como d|a, d|b e d|c então d|ax + by = 1 e d|mc + nb = 1 assim, d = 1 = (ac, b)

Portanto, (a · c, b) = 1 se, e somente se, (a, b) = (c, b) = 1

d) Pela parte c) deste Exercício temos de:

(a, b) = (c, b) = 1 ⇒ (a · c, b) = 1, e de (a, d) = (c, d) = 1 ⇒ (a · c, d) = 1

Como (b, a · c) = 1 e (d, a · c) = 1, então (b · d, a · c) = 1

Portanto, (a · c, b · d) = 1.

e) Aplicar indução, primeiro sobre n, logo sobre m, usando o resultado da parte a)

Portanto, (an , bm ) = 1, ∀ n, m ∈ N.
37

Exercício 6.2.3.
Para todos a; b ∈ Z e todo n ∈ N, mostre que.

(an , bn ) = (a, b)n

Solução.

Exercício 6.2.4.

a) Mostre que, se n é ímpar, n(n2 − 1) é divisível por 24

b) Mostre que 24 divide n(n2 − 1)(3n + 2) para todo n ∈ N.

Demonstração.

a) Primeira solução
Se n é ímpar então n = 2k + 1 para todo k ∈ Z, logo

n(n2 − 1) = (n − 1)n(n + 1) = (2k)(2k + 1)(2k + 2) = 4k(2k + 1)(k + 1) (6.7)

A igualdade em (6.7) vale para todo k ∈ Z. Todo número inteiro podemos escrever
como algum elemento do conjunto

A = { 6m, 6m + 1, 6m + 2, 6m + 3, 6m + 4, 6m + 5, ∀m∈Z}≡Z

Em (6.7), se k = 6m ⇒ n(n2 − 1) = 24m(12m + 1)(6m + 1)


Em (6.7), se k = 6m + 1 ⇒ n(n2 − 1) = 24(6m + 1)(4m + 1)(3m + 1)
Em (6.7), se k = 6m + 2 ⇒ n(n2 − 1) = 24(3m + 1)(12m + 5)(2m + 1)
Em (6.7), se k = 6m + 3 ⇒ n(n2 − 1) = 24(2m + 1)(12m + 7)(3m + 2)
Em (6.7), se k = 6m + 4 ⇒ n(n2 − 1) = 24(3m + 2)(4m + 3)(6m + 5)
Em (6.7), se k = 6m + 5 ⇒ n(n2 − 1) = 24(6m + 5)(12m + 11)(m + 1)
Portanto, n(n2 − 1) é divisível por 24.

a) Segunda solução
k(k + 1)(2k + 1)
Sabemos que a soma 12 + 22 + 32 + · · · + k 2 = ∈N
6
Na igualdade (6.7) temos

k(k + 1)(2k + 1)
n(n2 − 1) = 4k(2k + 1)(k + 1) = 24 · ∈N
6

Portanto, n(n2 − 1) é divisível por 24.


38

b) Seja P = n(n2 − 1)(3n + 2). Todo número natural podemos expressar como algum
elemento do conjunto

A = { 6m, 6m + 1, 6m + 2, 6m + 3, 6m + 4, 6m + 5, ∀m∈N}≡N

Como os números naturais da forma 6m + 1, 6m + 3, 6m + 5 ∀ m ∈ N são


ímpares, então pela parte a), é imediato que n(n2 − 1)(3n + 2) seja divisível por 24,
logo temos
P = n(n2 − 1)(3n + 2) (6.8)

Em (6.8), se k = 6m então

P = 6m(36m2 − 1)(18m + 2) = 12m(6m + 1)(6m − 1)(9m + 1)

Se m-par, o fator m é par, logo temos P = 24β para algum β ∈ N.

Se m-ímpar, o fator 9m + 1 é par, logo temos P = 24α para algum α ∈ N.

Em (6.8), se k = 6m + 2 então

P = 12(3m + 1)(2m + 1)(6m + 1)(9m + 4)

Se m-par, o fator 9m + 4 é par, logo temos P = 24β para algum β ∈ N.

Se m-ímpar, o fator 3m + 1 é par, logo temos P = 24α para algum α ∈ N.

Em (6.8), se k = 6m + 4 então

P = 12(3m + 2)(6m + 5)(2m + 1)(9m + 7)

Se m-par, o fator 3m + 2 é par, logo temos P = 24β para algum β ∈ N.

Se m-ímpar, o fator 3m + 1 é par, logo temos P = 24α para algum α ∈ N.

Portanto, 24 divide n(n2 − 1)(3n + 2) para todo n ∈ A ≡ N

Exercício 6.2.5.

a) Mostre que n5 − n é divisível por 30.

b) Mostre que n5 e n possuem o mesmo algarismo das unidades.

Demonstração.
39

a) Pelo exercício 12 da Unidade 1, sabemos que 5|n5 − n para todo n ∈ N.


Resta mostrar que 6|n5 − n.
Afirmo: O produto de três números consecutivos é múltiplo de 6.
Com efeito, sejam os três números consecutivos n − 1, n e n + 1.
Seja M = (n − 1)n(n + 1) = n(n2 − 1).
Sabemos que todo número natural podemos escrever como algum elemento do con-
junto A = { 3k, 3k + 1, 3k + 2 ∀ k ∈ N } ≡ N então:
Se n = 3k ⇒ M = (3k)(9k 2 − 1) = 6α ⇒ 6|M .
Pois, o fator k é par se k-par, e o fator 9k 2 − 1 é par se k-ímpar.
Se n = 3k + 1 ⇒ M = (3k)(3k + 1)(3k + 2) = 6β ⇒ 6|M .
Pois, o fator k é par se k-par, e o fator 3k + 1 é par se k-ímpar.
Se n = 3k + 2 ⇒ M = 3(3k + 1)(3k + 2)(k + 1) = 6γ ⇒ 6|M .
Pois, o fator 3k + 2 é par se k-par, e o fator k + 1 é par se k-ímpar.
Assim, 6|n(n2 − 1).
Voltando ao problema, como n5 −n = n(n4 −1) = n(n2 −1)(n2 +1) = 6ρ(n2 +1) ⇒
6|n5 − n e de 5|n5 − n, sendo (5, 6) = 1 temos que n5 − n é divisível por 30.

b) Primeira solução: Seja n ∈ N tal que n = . . . 102 a2 + 10a1 + a0 , então n = 10k + a0 ,


logo aplicando a parte a) deste exercício como a50 − a0 = 30β.


4
5
n = (10k + c) = 5
C5j (10k)5−j aj0 + a50 = 10α + a50 =
j=0

n5 = 10α + (30β + a0 ) = 10γ + a0

Portanto, n5 e n possuem o mesmo algarismo das unidades.

b) Segunda solução: Mostramos na parte a) que 30|n5 − n, logo 10|n5 − n.


Seja n = . . . a2 a1 a0 ⇒ n = . . . a2 a1 0 + a0
Também temos n5 = . . . b2 b1 b0 ⇒ n5 = . . . b2 b1 0 + b0
A diferença destes números é

n5 − n = (. . . b2 b1 0 + b0 ) − (. . . a2 a1 0 + a0 ) = 10(. . . c2 c1 ) + (b0 − a0 )

Como 10|n5 −n ⇒ n5 −n = 10β, β ∈ N e a0 e b0 são algarismos das unidades,


então b0 − a0 = 0 de onde b0 = a0 .
40

Portanto, n5 e n possuem o mesmo algarismo das unidades.

Exercício 6.2.6.
a
Mostre que a|bc se, e somente se, c.
(a, b)
Demonstração.

Suponhamos que a|bc, então bc = αa.


( a b ) ax by
Para todo a, b ∈ N temos que , = 1, então + = 1 para
(a, b) (a, b) (a, b) (a, b)
x, y ∈ Z, logo
axc αay a
+ =c ⇒ (xc + αy) = c
(a, b) (a, b) (a, b)
a
Portanto, se a|bc então |c.
(a, b)
Inversamente (⇐) [ ]
a a ab βb
Suponhamos que c, logo c = β , assim bc = β = a.
(a, b) (a, b) (a, b) (a, b)
a
Portanto, se c então a|bc.
(a, b)
Exercício 6.2.7.
Sejam a e b dois números inteiros e (a, b) = 1.

a) Mostre que (b + a, b − a) é 1 ou 2

b) Mostre que (a + b, a2 + b2 ) é 1 ou 2.

Demonstração.

a) Suponhamos que d = (b + a, b − a) então b + a = αd e b − a = βd


Resolvendo o sistema segue que 2b = (α + β)d e 2a = (α − β)d.
Sabe-se que (2a, 2b) = 2(a, b) = 2, logo ((α + β)d, (α − β)d) = 2 então d(α + β, α −
β) = 2.
Como (α + β, α − β) ∈ N então d = 1 ou d = 2.
Portanto, (b + a, b − a) é 1 ou 2

b) Primeira solução: Mostre que (a + b, a2 + b2 ) é 1 ou 2.


Deste exercício parte a) suponhamos que (b + a, b − a) = 1 e como (1, b) = 1. Pelo
Exercício (6.2.13) - a) segue do Lema 1 (Lema de Euclides)

1 = (b + a, b(b − a)) = (b + a, a(b + a) + b(b − a)) = (a + b, a2 + b2 )

Por outro lado, suponhamos que d = (a + b, a2 + b2 ) então d = (a + b, (a + b)2 − 2ab),


logo d = (a + b, −2ab) então d|a + b e d|2ab
41

Como (a, b) = 1 ⇒ (a + b, b) = 1 ou (a + b, a) = 1 isto é a + b - a e a + b - b,


logo d|2 ⇒ d = 1 ou d = 2
Portanto, (a + b, a2 + b2 ) = 1 ou 2.

b) Segunda solução: Mostre que (a + b, a2 + b2 ) é 1 ou 2.


Suponhamos d = (a + b, a2 + b2 ) então a + b = αd e a2 + b2 = βd, α, βinn.
Como a + b = αd ⇒ (a + b)(a − b) = αd(a − b) ⇒ d|a2 − b2 logo existe
γ ∈ N tal que a2 − b2 = γd.

a2 + b2 = βd, a2 − b2 = γd ⇒ 2a2 = d(β + γ), 2b2 = d(β − γ)

Como (a, b) = 1, pelo Exercício (6.2.6) segue que (a2 , b2 ) = 1 ⇒ a2 x + b2 y =


1 ⇒ 2a2 x + 2b2 y = 2 isto é

d(β + γ)x + d(β − γ)y = 2 ⇒ d|2 d = 1 ou d = 2

Portanto, (a + b, a2 + b2 ) = 1 ou 2.

Exercício 6.2.8.
Mostre que, se a, b, x, y ∈ Z, com ax + by = (a, b), então (x, y) = 1.
Demonstração.

Suponhamos (a, b) = d, então pela hipótese segue ax + by = d, além disso como como
(a, b) = d logo d|a e d|b, isto é existem α, β ∈ Z tais que a = αd e b = βd.
Substituindo em ax + by = d segue que x(αd) + y(βd) = d isto é d(xα + yβ) = d,
como d ̸= 0 segue que xα + yβ = 1.
Por outro lado, seja d1 = (x, y) então d1 |x e d1 |y, logo existem α1 , β1 ∈ Z tais que
x = d1 α1 e y = d1 β1 de onde 1 = xα + yβ = d1 (α1 α + .β1 β) então d1 |1, logo d1 = 1.
Portanto, (x, y) = 1.

Exercício 6.2.9.
a
Sejam a e b dois números naturais com (a; b) = 1. Mostre que se é par, então
(a, b)
b
é ímpar. Vale a recíproca?
(a, b)
Solução.

Exercício 6.2.10.
Um prédio possui duas escadarias, uma delas com 780 degraus e a outra com 700
degraus. Sabendo que os degraus das duas escadas só estão no mesmo nível quando con-
duzem a um andar, descubra quantos andares tem o prédio.
Solução.
42

Pelo algoritmo de Euclides, temos 20 = 9(780) − 10(700).


780
Pela primeira escada, para chegar ao segundo andar temos que subir
= 39 degraus.
20
700
Para segunda escada para chegar ao segundo andar andar temos que subir = 35
20
degraus.
O prédio tem 20 andares.

Exercício 6.2.11.
Calcule (1116, 984, 852).
Solução.

Partindo do princípio que o máximo divisor de dois números a e b não pode ultrapassar
ao máximo divisor comum de três números. a, b e c temos o seguinte.
Pelo algoritmo de Euclides obtemos 1116 = (1) × 852 + 264, de onde

12 = (−29)(1116) + (38)(852) = (1116, 852)

Logo, (1116, 984, 852) = ((1116, 852), 984) = (12, 984).


Observe que 984 = (82) × 12 + 0, então 12 = (12, 984).
Portanto, (1116, 984, 852) = 12.

Exercício 6.2.12.
Mostre que se três números inteiros são tais que dois deles são coprimos, então eles são
coprimos. Mostre que não vale a recíproca; isto é, exiba três números inteiros coprimos
mas que não são dois a dois coprimos.
Solução.

A demonstrar que: Se (a, b) = 1 , (a, c) = 1 e (b, c) = 1 então (a, b, c) = 1.


Suponhamos a, b, c ∈ N tais que (a, b) = 1, (a, c) = 1 e (b, c) = 1.

(a, b, c) = ((a, b) , c) = (1, c) = 1 ou (a, (b, c)) = (a, 1) = 1 ou = (b, (a, c)) = 1

Em qualquer caso (a, b, c) = 1.


Portanto, se (a, b) = 1 e (a, c) = 1 e (b, c) = 1 então (a, b, c) = 1.
A recíproca é
Se (a, b, c) = 1 então (a, b) = 1 e (a, c) = 1 e (b, c) = 1
Por contra-exemplo.
Temos (6, 14, 21) = 1 observe que (6, 14) = 2 ̸= 1 e (6, 21) = 3 ̸= 1 e
(14, 21) = 7 ̸= 1
Portanto, a recíproca nem sempre é verdadeira.
43

Exercícios suplementares
Exercício 6.2.13.
Suponha que (a, b) = (a, d) = (c, b) = (c, d) = 1.

a) Mostre que, se n ∈ N, então (a + b, bn ) = (a − b, bn ) = 1.

Demonstração.

a) Suponhams a ̸= b então a − b ̸= 0, como (a, b) = 1, então existem x, y ∈ Z tais que

1 = ax + by = (a − b)x + b(y + x) ⇒ 1 = (a − b, b)

Como (a − b, b) = 1, (1, bm−1 ) = 1 e pela parte a) deste exercício segue que


(a − b, bm ) = 1 para todo m ∈ N.
De modo análogo 1 = ax + by = (a + b)x + b(y − x) ⇒ 1 = (a + b, b),
(1, bm−1 ) = 1 e pela parte b) deste exercício segue que (a + b, bm ) = 1 para todo
m ∈ N.
Portanto, se a ̸= b e n ∈ N, então (a + b, bn ) = (a − b, bn ) = 1.
44
MA-14 - Aula 04

Semana 26/08 a 01/09

Unidade 7

7.2 Mínimo Múltiplo Comum: Problemas


Exercício 7.2.1.
Calcule o mmc dos pares de números.

a) 38 e 46 b) 35 e 75 c) 235 e 740
Solução.

Exercício 7.2.2.
Mostre o seguinte:

a) Sejam a, b ∈ Z não nulos e seja n ∈ N; mostre que [na, nb] = n[a, b].
m m
b) Se m é um múltiplo comum de a e b, mostre que m = [a, b] ⇔ ( , ) = 1.
a b
ra sb
c) Se r e s não são nulos e ra = sb, mostre que = = [a, b].
(r, s) (r, s)

Demonstração.

45
46

a) Pela Proposição 1 da Unidade 7, segue que [na, nb](na, nb) = na · nb então

ab
na · nb = n · [na, nb](a, b) ⇒ n· = [na, nb] ⇒ n[a, b] = [na, nb]
(a, b)

Portanto, se a, b ∈ N e n ∈ Z temos [na, nb] = n[a, b].

b) Se m é um múltiplo comum de a e b, então a|m e b|m, logo m = βa e m = αb


Como m = [a, b], então pela Proposição 1 da Unidade 7 segue que [a, b] · (a, b) = ab
de onde (ma, mb) = ab assim,

1 m m
(ma, mb) = 1 ⇒ ( , )=1
ab b a

m m
Portanto, se m = [a, b] ⇒ ( , ) = 1..
a b
Inversamente (⇐)
m m
Suponhamos que ( , ) = 1, como m é múltiplo de a e b, segue que m = βa e
a b
m = αb então (α, β) = 1.
De onde (m · α, m · β) = m, sendo m · α e m · β múltiplos de a e b, e como m|mα e
m|mβ, e m é o menor dos múltiplos de a e b, segue que m = [a, b].
m m
Portanto, ( , ) = 1. ⇒ m = [a, b].
a b
c) Sendo r e s não nulos, logo podemos dividir por esses números. Sabe-se que [a, b] ·
ab ra · b rab rab ra
(a, b) = ab então [a, b] = = = = = .
(a, b) r(a, b) (ra, rb) (sb, rb) (r, s)
ab sa · b sab sab sb
De modo análogo, temos [a, b] = = = = = .
(a, b) s(a, b) (sa, sb) (sa, ra) (r, s)
ra sb
Portanto, se ra = sb então = = [a, b].
(r, s) (r, s)

Exercício 7.2.3.
Sejam a, b, c três números naturais não nulos. Mostre que abc = [a, b, c](ab, ac, bc).
Demonstração.

Seja d = (a, b), como (ab, ac, bc) = (ab, (ac, bc)) então

(ab, ac, bc) = (ab, (ac, bc)) = (ab, c(a, b)) = (ab, cd) ⇒

abcd
(ab, ac, bc) = (ab, cd) = (7.9)
[ab, cd]
ab
Por outro lado, [a, b, c] = [[a, b], c] = [ , c] = [am, c] onde b = md.
d
47

Em (7.9)

abcd abcd abcd abc


(ab, ac, bc) = = = =
[ab, cd] [amd, cd] [amd, cd] [am, c]

abc abc
(ab, ac, bc) = = ⇔ (ab, ac, bc)[a, b, c] = abc
[am, c] [a, b, c]
Exercício 7.2.4.
Seja n ∈ N; calcule [n2 + 1, n + 1].
Demonstração.

Temos pela Proposição 9 unidade 1, que (n2 +1, n+1) = ((n2 −1)+2, n+1) = (2, n+1).
Supondo que 2|n2 +1 ou n+1|n2 +1 segue que n = 0 ou n = 1. Logo (n2 +1, n+1) = 1
ou (n2 + 1, n + 1) = 2.
Suponhamos que (n2 + 1, n + 1) = 1, então n-par e,

[n2 + 1, n + 1] · (n2 + 1, n + 1) = (n2 + 1) · (n + 1) ⇒ [n2 + 1, n + 1] = n3 + n2 + n + 1

Suponhamos que (n2 + 1, n + 1) = 2, então n-ímpar e,

1
[n2 + 1, n + 1] · (n2 + 1, n + 1) = (n2 + 1) · (n + 1) ⇒ [n2 + 1, n + 1] = (n3 + n2 + n + 1)
2

Exercício 7.2.5.

a) Mostre que (a, b) = [a, b] ⇔ a = b.

b) Mostre que, [an , bn ] = [a, b]n , a, b ∈ Z, n ∈ N.

c) Mostre que, se b = a2 , então, [a, b] = (a, b)2 .

Demonstração.

a) Seja (a, b) = [a, b], então pela Proposição 1 da Unidade 7 [a, b] · (a, b) = ab ⇒
[a, b]2 = ab.
Como m = [a, b] é tal que m = βa e m = αb segue

m2 = (βa)2 = ab ⇒ β 2 a = b ou m2 = (αb)2 = ab ⇒ α2 b = a

Substituindo um no outro α2 (β 2 a) = a ⇒ (αβ)2 = 1 ⇒ α = β = 1, logo


a = b.
Inversamente, suponhamos que a = b.
Temos [a, b] = [a, a] = a e (a, b) = (a, a) = a, assim [a, b] = (a, b).
48

b)

c) Mostre que, se b = a2 , então, (a, b) = (a, a2 ) = a. Por outro lado, [a, b] = [a, a2 ] = a2 .

Destas duas igualdades, segue que [a, b] = [a, a2 ] = a2 = (a, b)2

Portanto, se b = a2 , então, [a, b] = (a, b)2 .

Exercício 7.2.6.
Sejam a, b ∈ Z ambos não nulos. Considere o conjunto

M (a, b) = aZ ∩ bZ = { x ∈ Z; ∃ n, m ∈ Z tais que x = na e x = mb }

a) Mostre que [a, b] = min {M (a, b) ∩ N}.


b) Mostre que M (a, b) = [a, b]Z .

Demonstração.

a) Seja x ∈ M (a, b), então ∃ n, m ∈ Z tais que x = na e x = mb. Quando m = n = 1


temos que x = a = b ∈ Z, logo M (a, b) ̸= ∅ ainda mais, M (a, b) ⊂ N.

Pelo princípio da Boa Ordem, existe P ∈ M (a, b) tal que P = n0 a e P = m0 b


para algum m0 , n0 ∈ N além disso P ≤ x para todo x ∈ M (a, b). Portanto,
P = min M (a, b).

Suponhamos que [a, b] = c, então a|c e b|c, logo c = βa e c = αb onde α, β ∈ N.


Pela definição de M (a, b) segue que c ∈ M (a, b).

Como P é múltiplo comum de a e b então c|P , assim c ≤ P .

Por outro lado, sendo P o menor elemento de M (a, b), então P ≤ x para todo
x ∈ M (a, b), em particular para c, assim P ≤ c

Das duas últimas desigualdades P = c = [a, b] = min M (a, b).

Portanto, P = [a, b] = min M (a, b).

b) Pela parte (a) temos que qualquer elemento de x ∈ M (a, b) é múltiplo de P , isto é
x = γP , onde γ ∈ N

Portanto, todo elemento de M (a, b) é múltiplo de min M (a, b).

Exercício 7.2.7.
Sejam d, m ∈ N . Mostre que uma condição necessária e suficiente para que existam
a, b ∈ Z tais que (a, b) = d e [a, b] = m é que d|m.
Demonstração.
49

Condição necessária (⇒)


ab b a
Suponhamos que m = [a, b], então m = = a· = b· , logo a|m e
(a, b) (a, b) (a, b)
b|m
Por hipótese (a, b) = d logo a = αd e b = ηd isto é d|a e d|b.
Assim, a|m e d|a logo d|m, de modo análogo b|m e d|b logo d|m
Condição suficiente (⇐)
Suponhamos que d|m, então existe β ∈ N tal que m = βd. Sabemos que (β, 1) = 1
logo (βd, d) = d
Chamando a = βd e b = d temos que existe, a, b ∈ Z tal que (a, b) = d.
Por outro lado, pela definição de a e b temos que a|m e b|m, logo m é múltiplo
comum de a e b.
ab ad
Sabemos que [a, b] = = = a = m.
(a, b) d
Portanto se d|m, existem a, b ∈ Z tais que (a, b) = d e [a, b] = m é que .

Exercício 7.2.8.
Sejam a1 , · · · , an ∈ Z r {0}. Mostre que

(ai , aj ) = 1, i ̸= j ⇔ [a1 , · · · , an ] = a1 · · · an

Demonstração.

Indução sobre n ∈ N.
Condição necessária (⇒)
Se n = 2 temos (a1 , a2 ) = 1, ⇒ [a1 , a2 ](a1 , a2 ) = a1 a2 isto é [a1 , a2 ] = a1 a2 é
verdade.
Suponhamos para h ∈ N,sejam a1 , · · · , ah ∈ Z tais que

(ai , aj ) = 1, i ̸= j ⇒ [a1 , · · · , ah ] = a1 · · · ah

Seja h + 1 ∈ N e (ai , aj ) = 1, i ̸= j, ∀ i, j = 1, 2, . . . , h, h + 1, sabemos pela


Proposição 5.3.2. do livro 2 que

[a1 , · · · , ah , ah+1 ] = [a1 , a2 , a3 , · · · , [ah , ah+1 ]]

Da hipótese (ah , ah+1 ) = 1 ⇒ [ah , ah+1 ] = ah ah+1 , logo

[a1 , · · · , ah , ah+1 ] = [a1 , a2 , a3 , · · · , (ah ah+1 )] =

Como (ai , ah ) = 1 e (ai , ah+1 ) = 1 ⇒ (ai , ah ah+1 ) = 1, ∀ i = 1, 2, . . . , h − 1.


2
“Elementos de Aritmética” de A Hefez
50

Assim, temos da hipótese indutiva

[a1 , · · · , ah , ah+1 ] = a1 a2 a3 · · · (ah ah+1 ) = a1 a2 a3 · · · ah ah+1

Condição suficiente (⇐)


a1 a2
Suponhamos que [a1 , a2 ] = a1 a2 seja verdade, então [a1 , a2 ] = = a1 a2 de onde
(a1 , a2 )
(a1 , a2 ) = 1
Para qualquer h ∈ N suponhamos que

[a1 , · · · , ah ] = a1 · · · ah ⇒ (ai , aj ) = 1, i ̸= j ∀ i, j = 1, 2, . . . , h

Para h + 1 ∈ N temos

a1 · · · an an+1
[a1 , · · · , an , an+1 ] = a1 · · · an an+1 =
(a1 , · · · , an , an+1 )

logo (a1 , · · · , an , an+1 ) = 1. Pela Proposição 5.2.2. do livro 3


que

1 = (a1 , · · · , ah , ah+1 ) = (a1 , a2 , a3 , · · · , ah−1 , (ah , ah+1 )) ⇒

temos (ai , aj ) = 1, i ̸= j ∀ i, j = 1, 2, . . . , h − 1 sendo d = (ah , ah+1 ), também


(ai , d) = 1, ∀ i = 1, 2, . . . , h − 1. Para dois elementos temos

ah ah+1
[ah , ah+1 ] = = ah ah+1 ⇒ (ah , ah+1 ) = 1
(ah , ah+1 )

ai ah+1
[ai , ah+1 ] = = ai ah+1 ⇒ (ai , ah+1 ) = 1, ∀ i = 1, 2, . . . , h − 1
(ai , ah+1 )
Portanto, (ai , aj ) = 1, i ̸= j, ∀ i, j = 1, 2, . . . , h, h + 1.

Exercício 7.2.9.
Sejam a, b, c ∈ Z não nulos. Mostre que:
a) (a, [b, c]) = [(a, b), (a, c)]; b) [a, (b, c)] = ([a, b], [a, c]).
Demonstração.

3
“Elementos de Aritmética” de A Hefez
51

Unidade 8

Equações Diofantinas Lineares

8.2 Equações Diofantinas Lineares: Problemas


Exercício 8.2.1.
Resolva em Z as equações:
a) 90X + 28Y = 22 b) 50X + 56Y = 74
c) 40X + 65Y = 135 d) 8X + 13Y = 23
Solução.

a) 90X − 28X = 22 ⇒ 45X − 14Y = 11 como mdc{45, 14} = 1 e 45 = 3 × 14 +


3, 14 = 4 × 3 + 2, 3 = 1 × 2 + 1, logo

1 = 3−(1)(2) = 3−(1)(14−4×3) = 5×3−14 = 5(45−3(14))−14 = 5(45)−16(14)

Assim, 45(5) − 14(16) = 1 ⇒ 45(55 + 14t) − 14(176 + 45t) = 11

Portanto, X = 55 + 14t, Y = 176 + 45t, ∀ t ∈ Z.

b) Primeira solução: 50X − 56Y = 74 ⇒ 25X − 28Y = 37 como mdc{25, 28} = 1


e 28 = 1×25+3, 25 = 8×3+1, logo 1 = 25−8(3) = 25−8(28−25) = 9×25−8(28)

Assim, 25(9) − 28(8) = 1 ⇒ 25(333) − 28(296) = 37

Portanto, X = 333 + 28t, Y = 296 + 25t, ∀ t ∈ Z.

b) Segunda solução: 50X − 56Y = 74 ⇒ 25X − 28Y = 37 como (25, 28) = 1 e


28 = 1×25+3, 25 = 8×3+1, logo 1 = 25−8(3) = 25−8(28−25) = 9×25−8(28)

Assim, 25(9) − 28(8) = 1 ⇒ 25(333) − 28(296) = 37, como 333 = 11 × 28 + 25


e 296 = 11 × 25 + 21.

Podemos escrever 25(28t + 25) − 28(25t + 21) = 37

Portanto, X0 = 25, Y0 = 21 é a solução minimal.

c) 40X + 65Y = 135 ⇒ 8X + 13Y = 27 como (8, 13) = 1 e 13 = 1 × 8 + 5, 8 =


1 × 5 + 3, 5 = 1 × 3 + 2, 3 = 1 × 2 + 1, logo 1 = 8(5) + 13(−3) ⇒ 27 =
8(135) + 13(−81)

Assim, 8(135) + 13(−81) = 27 ⇒ 8(135 − 13t) + 13(8t − 81) = 27

Portanto, X = 135 − 13t, Y = 8t − 81, ∀ t ∈ Z.


52

d) 8X + 13Y = 23 como (8, 13) = 1 assim, 1 = 8(5) + 13(−3) ⇒ 23 = 8(130) +


13(−69)
Logo, 8(115) + 13(−69) = 23 ⇒ 8(115 − 13t) + 13(8t − 69) = 23
Portanto, X = 115 − 13t, Y = 8t − 69, ∀ t ∈ Z.

Exercício 8.2.2.
Para quais valores de c em N a equação 90X + 28Y = c não possui soluções em
N ∪ {0} ?
Solução.

Se c = 2k + 1 é ímpar então 2(45X + 14Y ) = 2k + 1, isto é absurdo!


Logo a equação não tem solução para os valores de c ∈ N onde c-ímpar.
Estudemos o caso c-par.
Sendo c = 2k, k ∈ N temos 2(45X + 14Y ) = 2k ⇒ 45X + 14Y = k.
Pelo algoritmo de Euclides obtemos 45(5) − 14(16) = 1 ⇒ 45(5k) − 14(16k) = k,
ainda podemos escrever na forma

45(5k + 14t) − 14(16k + 45t) = k t∈N

Observe que 5k + 14t ∈ N e 16k + 45t ∈ N quando k ≥ 14, de onde c ≥ 28 é par.

Exercício 8.2.3.
Resolva em N r {0} as equações:
a) 16X + 7Y = 601 b) 30X + 17Y = 201
c) 47X + 29Y = 1288 d) 8X + 13Y = 23
Solução.

a) 16X + 7Y = 601 como (16, 7) = 1 e 1 = 7 × 7 − 3 × 16, assim, 16(−3) + 7(7) = 1 ⇒


16(−1803) + 7(4207) = 601 ⇒ 16(7t − 1803) + 7(4207 − 16t) = 601
Assim, X = 7t − 1803, Y = 4207 − 16t, ∀ t ∈ Z, se s = (262 − t) temos
Portanto, X = 31 − 7, Y = 15 + 16s, ∀s∈Z

b) 30X +17Y = 201 como (30, 17) = 1, assim, 1 = 30×(−6)+17×11 ⇒ 30(−1206)+


17(2211) = 601 ⇒ 30(17t − 1206) + 17(2211 − 30t) = 601
Portanto, X = 17t − 1206, Y = 2211 − 30t, ∀t∈Z

c) 47X + 29Y = 1288 como (47, 29) = 1, assim, 1 = 47 × (−8) + 13 × 29 ⇒


47(−10304) + 29(16744) = 1288 ⇒ 47(29t − 10304) + 29(16744 − 47t) = 1288
Portanto, X = 29t − 10304, Y = 16744 − 47t, ∀t∈Z
53

Observe que X = 29(t − 356) + 20, Y = 12 − 47(t − 356), ∀ t ∈ Z, e


temos que X = 29s + 20, Y = 12 − 47s, ∀ s ∈ Z, resolve o problema disto
X0 = 20, Y0 = 12 é a solução minimal.

d) 8X + 13Y = 23 como (8, 13) = 1 assim, 1 = 8(5) + 13(−3) ⇒ 23 = 8(130) +


13(−69)
Logo, 8(115) + 13(−69) = 23 ⇒ 8(115 − 13t) + 13(8t − 69) = 23

X = 11 + 13(8 − t), Y = 8(t − 8) − 5, ∀t∈Z

Portanto, X = 11 + 13s, Y = 8s − 5, ∀ s ∈ Z.

Exercício 8.2.4.
Dispondo de 100 reais, quais são as quantias que se podem gastar comprando selos de
5 reais e de 7 reais?
Solução.

Suponhamos sejam X selos de 5 reais e Y selos de 7 reais, então 5X + 7Y = 100


como 5(3) + 7(−2) = 1, logo 5(300) + 7(−200) = 100 a solução geral da equação é
5(300 − 7t) + 7(5t − 200) = 100 para certos t ∈ N.
Logo 0 ≤ 300 − 7t e 0 ≤ 5t − 200 então t ≤ 42 e 40 ≤ t

t 40 41 42
X 20 13 6
Y 0 5 10
Podemos comprar 13 selos de R$5 e 5 selos de R$7, ou 6 selos de R$5 e 10 selos de
R$7 ou somente 20 selos de R$5.

Exercício 8.2.5.
Determine todos os múltiplos de 11 e de 9 cuja soma é igual a: a) 79 b) 80
c) 270
Solução.

a) 11a + 9b = 79 como (11, 9) = 1 e 1 = 5 × 9 − 4 × 11, assim, 11(−4) + 9(5) =


1 ⇒ 11(−316) + 9(395) = 79 ⇒ 11(9t − 316) + 9(395 − 11t) = 79
Logo, a = 9t − 316, b = 395 − 11t, t ∈ Z.
Se a ≥ 0 ⇒ 9t − 316 ≥ 0 ⇒ t ≥ 35, 11 ⇒ t = 36.
Se b ≥ 0 ⇒ 395 − 11t ≥ 0 ⇒ 35, 9 ≥ t ⇒ t = 35.
Logo não existe t ∈ Z que resolva o problema, consequentemente não existem múl-
tiplos de 11 e de 9 cuja soma é igual a 79.
54

b) 11a + 9b = 80 como (11, 9) = 1 e 1 = 5 × 9 − 4 × 11, assim, 11(−4) + 9(5) =


1 ⇒ 11(−320) + 9(400) = 80 ⇒ 11(9t − 320) + 9(400 − 11t) = 79

Logo, a = 9t − 320, b = 400 − 11t, t ∈ Z.

Se a ≥ 0 ⇒ 9t − 320 ≥ 0 ⇒ t ≥ 35, 55 ⇒ t = 36.

Se b ≥ 0 ⇒ 400 − 11t ≥ 0 ⇒ 36, 6 ≥ t ⇒ t = 36.

Logo existe t ∈ Z que resolve o problema, consequentemente existem múltiplos de


11 e de 9 cuja soma é igual a 79, eles são 44 e 36.

c) 11a + 9b = 270 como (11, 9) = 1 e 1 = 5 × 9 − 4 × 11, assim, 11(−4) + 9(5) =


1 ⇒ 11(−1080) + 9(1350) = 270 ⇒ 11(9t − 1080) + 9(1350 − 11t) = 79

Logo, a = 9t − 1080, b = 1350 − 11t, t ∈ Z.

Se a ≥ 0 ⇒ 9t − 1080 ≥ 0 ⇒ t ≥ 120.

Se b ≥ 0 ⇒ 1350 − 11t ≥ 0 ⇒ 122, 72 ≥ t ⇒ t = 120, 121. 122.

Logo existem t ∈ Z que resolvem o problema, consequentemente existem múltiplos


de 11 e de 9 cuja soma é igual a 270, são: a = 0, b = 30, a = 9, b = 19 e
a = 18, b = 8.

Exercício 8.2.6.
Determine o menor inteiro positivo que tem restos 11 e 35 quando dividido, respecti-
vamente, por 37 e 48.
Solução.( Primeira solução)

Seja N o inteiro positivo.


Temos que N = 37x + 11 = 48y + 35 ⇒ 37x − 48y = 24
Como o mdc{37, 48} = 1 e

48 = (1)(37) + 11, 37 = (3)(11) + 4, 11 = (2)(4) + 3, 4=3+1

1 = 4 − (11 − 2 × 4) = −11 + 3 × 4 = −11 + 3(37 − 3 × 11) = −10(11) + 3(37) =

1 = −10(48 − 37) + 337 = 48(−10) + 37(13)

logo, 37(13) + 48(−10) = 1 ⇒ 37(312) + 48(−240) = 24

48 37
x = 312 − t = 312 − 48t, y = −240 + t = −240 + 37t
1 1

O número é N = 37(312 − 48t) + 11 = 48(240 − 37t) + 35, t ∈ N. Como 240 − 37t ≥ 0


e 312 − 48t ≥ 0, então t ≤ 6
55

t 6 5 4 0
X 24 92 120 312
Y 18 55 92 -240
N 899 2675 4451 11555

O menor número é 899.


Solução.(Segunda solução)
Seja N o inteiro positivo, temos N = 37x + 11 = 48y + 35 ⇒ 37x − 48y = 24 logo
37x = 24(1 + 2y) ⇒ x = 24s, 2y = 37s − 1 s ∈ Z

s 1 3 5 7
x 24 72 120 168
y 18 55 92 129
N 899 2675 4451 6227

Exercício 8.2.7.
Numa criação de coelhos e galinhas, contaram-se 400 pés. Quantas são as galinhas
e quantos são os coelhos, sabendo que a diferença entre esses dois números é a menor
possível?
Solução.( Primeira solução)

Sejam g o número de galinhas e c o total de coelhos, temos 4c + 2g = 400 ⇒


2c + g = 200, como o mdc{2, 1} = 1 e 2(1) − 1(1) = 1 então 2(200) − 1(200) = 200,
isto é 2(200 − t) + 1(2t − 200) = 200
Logo c = 200 − t; 0 < t < 200 e g = 2t − 200; 100 < t

t 101 199 149 151 125 165 127 132 133 134
c 99 1 51 49 75 35 73 68 67 66
g 2 198 98 102 50 110 54 64 66 68
diferença 97 197 47 53 25 75 19 3 1 2

A menor diferença é 1, que se obtém com 67 coelhos e 66 galinhas.


Solução.(Segunda solução)
Sejam g o número de galinhas e c o total de coelhos, temos 4c + 2g = 400 isto é
g = 2(100 − c) ⇒ g = 2s, c = 100 − s, s ∈ Z,

s 1 2 3 4 5 10 20 32 33 34
c 99 98 97 96 95 90 80 68 67 66
g 2 4 6 8 10 20 40 64 66 68
diferença 97 94 91 88 85 70 40 4 1 2

A menor diferença é 1, que se obtém com 67 coelhos e 66 galinhas.


56

Exercício 8.2.8.
Subindo uma escada de dois em dois degraus, sobra um degrau. Subindo a mesma
escada de três em três degraus, sobram dois degraus. Determine quantos degraus possui a
escada, sabendo que o seu número é múltiplo de 7 e está compreendido entre 40 e 100.
Solução.

Seja N o número de degraus, então 40 ≤ N ≤ 100


Temos que N = 2x + 1 = 3y + 2 ⇒ N = 2(x + 1) − 1 = 3(y + 1) − 1, logo N + 1
é múltiplo de 6, assim como 40 ≤ N ≤ 100 segue que

N = 41, 47, 53, 59, 65, 71, 77, , 83, 89, 95

Também sendo múltiplo de 7, o número é 77.

Exercício 8.2.9.
(ENC 2002) Em certo país, as cédulas são de $4 e $7. Com elas, é possível pagar, sem
troco, qualquer quantia inteira a) a partir de $11, inclusive. b) a partir de $18, inclusive.
c) ímpar, a partir de $7, inclusive. d) que seja $1 maior do que um múltiplo de $3. e)
que seja $1 menor do que um múltiplo de $5.
Solução.

a) Suponha que 4X + 7Y = n para n ≥ 11, então como 4(2) + 7(−1) = 1 segue que
4(2n − 7t) + 7(4t − n) = n, t ∈ N.
Como 2n − 7t ≥ 0 e 4t − n ≥ 0 ⇒ 7n ≤ 28t ≤ 8n
Quando n = 11 ⇒ X = 1, Y = 1; quando n = 12 ⇒ X = 3, Y = 0,
quando n = 13 não existem cedulas que para pagar esse valor sem troco.
Portanto a) é falso.

b) Pela primeira parte, quando n ≥ 18 temos 4(2n − 7t) + 7(4t − n) = n, t ∈ N.


Como X = 2n − 7t ≥ 0 e Y = 4t − n ≥ 0 ⇒ 7n ≤ 28t ≤ 8n
Podemos representar na tabela

n 18 19 20 21 22 23 24 25 26 27 28 28 29 31
t 5 5 5 6 6 6 6 7 7 7 7 8 8 8
X 1 4 5 0 2 4 6 1 3 5 7 0 2 6
Y 2 1 0 3 2 1 0 3 2 1 0 4 3 1

Observe que 7n ≤ 28t ≤ 8n ⇒ 0, 250n ≤ t ≤ 0, 285n


57

c) Pela primeira parte, quando n ≥ 7 sendo n-ímpar temos 4(2n − 7t) + 7(4t − n) = n,
t ∈ N, então X = 2n − 7t ≥ 0 e Y = 4t − n ≥ 0 ⇒ 7n ≤ 28t ≤ 8n.
Se n = 9 ⇒ 2, 25 ≤ t ≤ 2, 57, logo não existe t ∈ Z.
Portanto, não é possível pagar 9 reais.

d) Que seja $1 maior do que um múltiplo de $3. Se queremos pagar $10 com essas notas
de $4 e $7 é impossível.

e) Que seja $1 menor do que um múltiplo de $5. Se queremos pagar $9 com essas notas
de $4 e $7 é impossível.

Portanto, a resposta correta a esta questão deve ser a letra c).

Exercício 8.2.10.
De quantas maneiras pode-se comprar selos de 3 reais e de 5 reais de modo que se
gaste 50 reais?
Solução.

Suponhamos podemos comprar X selos de 3 reais e Y selos de 5 reais, então 3X +


5Y = 50 como (3, 5) = 1 e 1 = 2 × 3 − 1 × 5, assim, 3(2) + 5(−1) = 1 ⇒
3(100) + 5(−50) = 50 ⇒ 3(100 − 5t) + 5(3t − 50) = 50
Logo, X = 100 − 5t, Y = 3t − 50, t ∈ Z.
Se a ≥ 0 ⇒ 100 − 5t ≥ 0 ⇒ 20 ≥ t.
Se b ≥ 0 ⇒ 3t − 50 ≥ 0 ⇒ t ≥ 16, 6 ⇒ t = 17, 18, 19, 20.
Logo existem t ∈ Z que resolvem o problema, consequentemente existem múltiplos de
3 e de 5 cuja soma é igual a 50, são: X = 15, Y = 1, X = 10, Y = 4, X = 5, Y = 7.
Podemos comprar de três modos diferentes.

Exercício 8.2.11.
Sejam a1 ; a2 ; · · · ; an ; ∈ Z. Mostre que a equação

a1 X1 + a2 X2 + · · · + an Xn = c

possui soluções inteiras se, e somente se, (a1 ; a2 ; . . . an )|c.


Solução.

8.3 Exercícios Suplementares


Exercício 8.3.1.
58

Calcule o mmc dos pares de números do Problema 1, Unidade 5.


Solução.

Os números do Problema 1, Unidade 5 são:


a) 637 e 3887 b) 648 e 1218 c) 551 e 874
d) 7325 e 8485 e) 987654321 e 123456789

a) Temos que (3887, 637) = 13 e como 3887 × 637 = 244881, logo [3887, 637] =
2476019
= 190463
13
Portanto, [3887, 637] = 190463.
789264
b) Temos que (648, 1218) = 6 e como 1218×648 = 789264, logo [648, 1218] = =
6
131544
Portanto, [648, 1218] = 131544.
481574
c) Temos que (874, 551) = 19 e como 874 × 551 = 481574, logo [874, 551] = =
19
25346
Portanto, [874, 551] = 25346.

d)

e)
MA-14 - Aula Revisão

Semana 02/09 a 08/09

Unidade 9

9.2 Revisão
Exercício 9.2.1.

a) Quantos múltiplos de 5 existem no intervalo [1, 120]? e no intervalo [1, 174]?

b) Quantos múltiplos de 7 existem em cada um dos intervalos [70, 342] e [72, 342]?

Solução.

a) Primeira solução: Seja n a quantidade de múltiplos de 5, para o intervalo [1, 120]


temos que 5n ∈ [1, 120] ⇒ 1 < 5 ≤ 5n ≤ 120 ⇒ 1 ≤ n ≤ 24, de onde para
n = 24 se obtém 120. Assim, no intervalo [1, 120] temos 24 múltiplos de 5.
Para o intervalo [1, 174] temos que 5n ∈ [1, 174] ⇒ 5 ≤ 5n ≤ 170 < 174 ⇒
1 ≤ n ≤ 34, de onde para n = 34 se obtém 174. Assim, no intervalo [1, 120] temos
34 múltiplos de 5.

a) Segunda solução: Seja n a quantidade de múltiplos de 5, para o intervalo [1, 120]


120 − 5
temos que n = + 1 ⇒ n = 24. Assim, no intervalo [1, 120] temos 24
5
múltiplos de 5.
Para o intervalo [1, 174] temos, seja m a quantidade de múltiplos de 7, para o
170 − 5
intervalo [1, 174] temos que m = + 1 ⇒ m = 34.
5
b) Temos 70 = 7 × 10, 77 = 7 × 11, . . . , 342 = 7 × 48 + 6. Em [70, 342], existem
48 − 9 = 39 múltiplos de 7.
Temos 72 < 77 = 7 × 11, 84 = 7 × 12, . . . , 342 = 7 × 48 + 6. Em [72, 342], existem
48 − 10 = 38 múltiplos de 7.

59
60

De outro modo, seja n a quantidade de múltiplos de 7, para o intervalo [70, 342]


336 − 70
temos que n = + 1 ⇒ n = 39. Para o intervalo [72, 342] temos,
7
seja m a quantidade de múltiplos de 7, para o intervalo [72, 342] temos que m =
336 − 77
+ 1 ⇒ m = 38.
7
Exercício 9.2.2.
Dados 0 < a ≤ n < m. a) Mostre que no intervalo [1, n] existem q múltiplos de
a, onde q é o quociente da divisão de n por q. b) Quantos são os múltiplos de a no
intervalo [n, m]? (Na última situação, divida a análise em dois casos: n múltiplo de a e
o contrário.)
Demonstração. a)

Como 0 < a ≤ n então existem q, r ∈ N tais que n = aq + r onde 0 ≤ r < a


Assim temos relação 0 < a ≤ aq + r, sendo a > 0 e r < a temos que todos os múltiplos
de a se encontram no intervalo [a, aq] múltiplos de a. Os múltiplos são

a, 2a, 3a, 4a, 5a, . . . , a(q − 1), aq

Portanto, são q a quantidade de múltiplos de a.


Demonstração. b)
Como 0 < a ≤ n < m então existem q1 , r1 ∈ N tais que m = aq1 + r1 onde 0 ≤ r1 < a
Temos duas situações:
• Suponhamos n = aq, então todos os múltiplos de a que se encontram no intervalo
[n, m] são

aq, a(q + 1), a(q + 2), a(q + 3), a(q + 4), . . . , a(q1 − 1), aq1

Portanto, são q1 − (q − 1) a quantidade de múltiplos de a no intervalo [n, m].


• Suponhamos n = aq + r, então todos os múltiplos de a se encontram no intervalo
[n, m] múltiplos de a são

a(q + 1), a(q + 2), a(q + 3), a(q + 4), . . . , a(q1 − 1), aq1

Portanto, são q1 − q a quantidade de múltiplos de a no intervalo [n, m].

Exercício 9.2.3.
Mostre que dados m inteiros consecutivos um, e apenas um, deles é múltiplo de m.
Demonstração.

Seja k o primeiro número inteiro de uma sequência, então temos m inteiros consecutivos

k, k + 1, k + 2, k + 3, . . . , k + (m − 2), k + (m − 1)
61

o último número é k + (m − 1).


Se k = 1 existe somente um número múltiplo de m, é o último número.
Se k = 2 existe somente um número múltiplo de m, é o penúltimo número.
Se k = 3 existe somente um número múltiplo de m, é o antes do penúltimo número.
E assim sucessivamente.
Se m|k existe somente um número múltiplo de m, é o primeiro número.
Portanto, dados m inteiros consecutivos um, e apenas um, deles é múltiplo de m.
Demonstração. Segunda demonstração
Seja k ∈ N∗ , então dado m ∈ N temos que mk ∈ Z, então os m números consecutivos
são

mk, mk + 1, mk + 2, mk + 3, mk + 4, . . . , mk + (m − 3), mk + (m − 2), mk + (m − 1)

Temos que qualquer número inteiro n pode-se escrever somente em uma das formas
da sequência acima, então, n = mk + r, onde 0 ≤ r < m − 1
Evidentemente quando r = 0, o primeiro número mk é múltiplo de m da sequência.
Portanto, dados m inteiros consecutivos um, e apenas um, deles é múltiplo de m.

Exercício 9.2.4.
Mostre que o produto de quatro números inteiros consecutivos, quaisquer, é sempre
múltiplo de 24.
Demonstração.

Por indução sobre n.


Se n = 1 temos que 1 × 2 × 3 × 4 = 24 é verdade.
Suponhamos que para n ≤ h cumpra h(h + 1)(h + 2)(h + 3) = 24α
Para n = h + 1 segue da hipótese auxiliar, e sabendo que o produto de três números
consecutivos é múltiplo de 6.

(h+1)(h+2)(h+3)(h+4) = h(h+1)(h+2)(h+3)+4(h+1)(h+2)(h+3) = 24α+4×6β = 24γ

Portanto, o produto de quatro números inteiros consecutivos, quaisquer, é sempre


múltiplo de 24.
Demonstração. Segunda demonstração
Seja n ≥ 0, n ∈ N e consideremos os números n − 1, n, n + 1, n + 2, estes são 4
números consecutivos.
Seu produto é P = (n − 1)n(n + 1)(n + 2), quando n-par isto é n = 2k, ∀k ∈ N
temos

P = (2k − 1)(2k)(2k + 1)(2k + 2) = 4(2k − 1)[k(k + 1)(2k + 1)] ⇔


62

k(k + 1)(2k + 1)
P = 24(2k − 1)[ ] = 24(2k − 1)[12 + 22 + . . . + n2 ] = 24α
6
Quando n-ímpar isto é n = 2k + 1, ∀ k ∈ N temos

P = (2k)(2k + 1)(2k + 2)(2k + 3) = 4[k(k + 1)(2k + 1)](2k + 3) =

k(k + 1)(2k + 1)
P = 24[ ](2k + 3) = 24(2k + 3)[12 + 22 + . . . + n2 ] = 24β
6
Portanto, o produto de quatro números inteiros consecutivos, quaisquer, é sempre
múltiplo de 24.

Exercício 9.2.5.
a) Ache o menor inteiro positivo n tal que o número 4n2 + 1 seja divisível por 65. b)
Mostre que existem infinitos múltiplos de 65 da forma 4n2 + 1. c) Mostre que se um dado
número divide um número da forma 4n2 + 1, ele dividirá uma infinidade desses números.
d) Para este último resultado, existe algo de especial nos números da forma 4n2 + 1?
Teste o seu resultado para números da forma an2 + bn + c, onde a, b, c ∈ Z, com a e
b não simultaneamente nulos. e) Mostre que existem infinitos múltiplos de 7 da forma
8n2 + 3n + 4.
Solução.

a) 4n2 + 1 = 65k = (64 + 1)k = (4 × 42 + 1)k ⇒ k = 1, n=4

b) Pela parte (a) temos que 4n2 + 1, podemos supor n = 4 + 65k, k ∈ N.

Então 4n2 + 1 = 4(4 + 65k)2 + 1 = (4 × 42 + 1) + 65(32k + 260k 2 ) = 65α onde


α ∈ N. Assim, 65|4n2 + 1 sendo n = 4 + 65k, k ∈ N.

c)

Exercício 9.2.6.
a) Sejam dados os dois números a = 10c + r e b = c − 2r, com c, r ∈ Z. Mostre que
a é divisível por 7 se, e somente se b é divisível por 7. b) Deduza o seguinte critério de
divisibilidade por 7: O número n = ar . . . a1 a0 é divisível por 7 se, e somente se, o número
ar . . . a1 − 2a0 é divisível por 7. c) Utilize repetidas vezes o critério acima para verificar
se 2.368 é ou não divisível por 7.
Demonstração.
63

a) Suponhamos que 7|a, então a = 7β ⇒ 7β = 10c + r como b = c − 2r segue


c = b + 2r assim

7β = 10c + r = 10(b + 2r) + r = 10 · b + 21 ⇒ 7β1 = 10b ⇒ 7|b

b) Seja n = ar . . . a1 a0 ⇒ n = ar . . . a1 0 + a0 de onde n = 10X + a0 , onde X =


ar . . . a1 .
Seja P o número ar . . . a1 − 2a0 ⇒ P = X − 2a0 .
Suponhamos que n seja múltiplo de 7, então n = 10X + a0 = m(7), isto é

n = 10X + a0 = m(7) ⇔ n = (7X + 3X) + a0 − 7a0 = m(7) − 7a0 ⇔

⇔ n = 3X−6a0 = m(7) ⇔ 3(X−2a0 ) = m(7) ⇔ X−2a0 = P = m(7)

Portanto, se o número n = ar . . . a1 a0 é divisível por 7 então o número ar . . . a1 − 2a0


é divisível por 7.
Inversamente (⇐)
Suponhamos que o número ar . . . a1 − 2a0 é divisível por 7 então o número P =
X − 2a0 é divisível por 7.
O número n = ar . . . a1 a0 podemos escrever na forma n = 10X + a0 então

n = 10X + a0 = n = 10(P + 2a0 ) + a0 = 10P + 21a0 = m(7)

Portanto, se o número ar . . . a1 − 2a0 é divisível por 7 então o número n = ar . . . a1 a0


é divisível por 7.

c) Temos para o número 2.368

236 − 16 = 220, 22 − 2 = 20

Como 20 não é múltiplo de 7, logo 2.368 não é múltiplo de 7.

Exercício 9.2.7.
Um número inteiro n é dito um quadrado se existe a ∈ Z tal que n = a2 . Dizemos
que n é uma potência m-ésima quando n = am . a) Mostre que o algarismo das unidades
de um quadrado só pode ser um dos seguintes: 0, 1, 4, 5, 6 e 9. b) Mostre que nenhum
dos números 22, 222, 2222, . . . , ou 33, 333, 3333, . . . ou 77, 777, 7777, . . . , ou ainda
88, 888, 8888, . . . pode ser um quadrado.
Demonstração. a)
64

Todo número n = ar ar−1 . . . a1 a0 podemos escrever na forma n = 10X + a0 , logo


n = 10(10X 2 + 2Xa0 ) + a20 = 10Y + a20
2

• Se a0 = 0 então n2 = 10Y + 02 = . . . 0, termina em 0.

• a0 = 1 então n2 = 10Y + 12 = . . . 1, termina em 1.

• a0 = 2 então n2 = 10Y + 22 = . . . 4, termina em 4.

• Se a0 = 3 então n2 = 10Y + 32 = . . . 9, termina em 9.

• a0 = 4 então n2 = 10Y + 42 = 10Y + 10 + 6 = . . . 6, termina em 6.

• a0 = 5 então n2 = 10Y + 52 = 10Y + 20 + 5 = . . . 5, termina em 5.

• a0 = 6 então n2 = 10Y + 62 = 10Y + 30 + 6 = . . . 6, termina em 6.

• Se a0 = 7 então n2 = 10Y + 72 = 10Y + 40 + 9 = . . . 9, termina em 9.

• a0 = 8 então n2 = 10Y + 82 = 10Y + 60 + 4 = . . . 4, termina em 4.

• a0 = 9 então n2 = 10Y + 92 = 10Y + 80 + 1 = . . . 1, termina em 1.

Portanto, o algarismo das unidades de um quadrado só pode ser um dos seguintes:


0, 1, 4, 5, 6 e 9.
Demonstração. b)
Aplicando o resultado da parte a), temos que o algarismo das unidades da sequência

• 22, 222, 2222, . . . , termina em 2 logo nenhum número será quadrado prefeito.

• 33, 333, 3333, . . . termina em 3 logo nenhum número será quadrado prefeito.

• 77, 777, 7777, . . . , termina em 7 logo nenhum número será quadrado prefeito.

• 88, 888, 8888, . . . termina em 8 logo nenhum número será quadrado prefeito.

Exercício 9.2.8.
a) Mostre que todo quadrado ímpar é da forma 4n + 1.
b) Mostre que nenhum número na sequência 11, 111, 1111, 11111, . . . ., é um quadrado.
c) Mostre que nenhum número na sequência 44, 444, 4444, 44444, . . . ., é um quadrado.
d) Mostre que nenhum número na sequência 99, 999, 9999, 99999, . . . ., é um quadrado.
e) Mostre que nenhum número na sequência 55, 555, 5555, 55555, . . . ., é um quadrado.
Demonstração. a)
65

Seja P = 2k + 1, k ∈ N qualquer número ímpar, então n2 = 4(k 2 + k) + 1 = 4n + 1.


Portanto, todo quadrado ímpar é da forma 4n + 1.
Demonstração. b)
Qualquer número M da sequência 11, 111, 1111, 11111, . . . . é um número ímpar e
podemos escrever na forma

M = 11111100 + 11 = 4β + 8 + 3 = 4n + 3, β, n ∈ N

Sendo M número ímpar, é da forma M = 4n + 3, logo não é quadrado perfeito.


Demonstração. c)
Sabemos que, se M ×P ∈ N ⇒ (M ×P )2 = M 2 ×P 2 , M, P ∈ N sendo (M ×P )2
um número quadrado.
Qualquer número N da sequência 44, 444, 4444, 44444, . . . ., podemos escrever na
forma
N = 44444 . . . 44444 = 4 × 1111 . . . 1111 = 22 × 1111 . . . 1111 =

Pela parte b) deste exercício o número 1111 . . . 1111 não é quadrado, logo o número
N não será quadrado de nenhum número natural.
Demonstração. d)
Análogo a c), qualquer número N da sequência 99, 999, 9999, 99999, . . . ., podemos
escrever na forma

N = 99999 . . . 99999 = 9 × 1111 . . . 1111 = 32 × 1111 . . . 1111 =

Pela parte b) deste exercício o número 1111 . . . 1111 não é quadrado, logo o número
N não será quadrado de nenhum número natural.
Demonstração. e)
Qualquer número M da sequência 55, 555, 5555, 55555, . . . . é um número ímpar e
podemos escrever na forma

M = 5555500 + 55 = 4β + 52 + 3 = 4n + 3, β, n ∈ N

Sendo M qualquer número da sequência 55, 555, 5555, 55555, . . . . esse número é da
forma M = 4n + 3, logo não é quadrado perfeito.

Exercício 9.2.9.
a) Mostre que nenhum número da forma 4n + 2 é um quadrado.
b) Mostre que nenhum dos números 66, 666, 6666, . . . é um quadrado.
Demonstração. a)

Todo número natural n pode ser escrito de uma das formas seguintes: 4k, 4k+1, 4k+2
66

ou 4k + 3, k ∈ N. Elevando cada um desses números ao quadrado obtemos:


n2 = (4k)2 = 16k 2 = 4.(4k 2 ) = 4x1 + 0
n2 = (4k + 1)2 = 16k 2 + 8k + 1 = 4(4k 2 + 2k) + 1 = 4x2 + 1
n2 = (4k + 2)2 = 16k 2 + 16k + 4 = 4(4k 2 + 4k + 1) = 4x3 + 0
n2 = (4k + 3)2 = 16k 2 + 24k + 9 = 16k 2 + 24k + 8 + 1 = 4(4k 2 + 6k + 2) + 1 = 4x4 + 1
Dessa forma vemos que todo quadrado é escrito de uma, e somente de uma das formas
seguintes: 4x1 + 0, 4x2 + 1, 4x3 + 0, 4x4 + 1.
Isto nos mostra que todo quadrado, quando divididos por 4 sempre deixam resto
0(zero) ou 1, daí podemos concluir que nenhum número da forma 4k + 2 é um quadrado .
Demonstração. b)
O número 66 é da forma 66 = 4(16) + 2 = 4α + 2, logo não é um quadrado perfeito.
Como 100 é múltiplo de 4 e em geral o número N = 666 . . . 666 podemos escrever na
forma
N = 666 . . . 666 = 666 . . . 66600 + 66 = 666 = 100 × 666 . . . 666 + 66

N = 4β + 66 = 4β + (4α + 2) = 4(β + α) + 2 = 4x1 + 2

Logo N é um múltiplo de 4k + 2 e portanto não é quadrado perfeito.

Exercício 9.2.10.
a) Mostre que a soma de quatro inteiros consecutivos nunca é um quadrado. b) Mostre
que a soma dos quadrados de quatro inteiros consecutivos nunca é um quadrado. Faça o
mesmo para a soma dos quadrados de três inteiros consecutivos.
Demonstração. a)

Sejam os inteiros consecutivos n − 1, n, n + 1, n + 2, a mostrar que sua soma


S = (n − 1) + n + (n + 1) + (n + 2) = 4n + 2 nunca é um quadrado.
Pelo exercício (9.2.9) - a) nenhum número da forma S = 4n + 2 é um quadrado.
Portanto, a soma de quatro inteiros consecutivos nunca é um quadrado.
Demonstração. b)
Sejam os inteiros consecutivos n − 1, n, n + 1, n + 2, a mostrar que sua soma
S = (n − 1)2 + n2 + (n + 1)2 + (n + 2)2 nunca é um quadrado.
Com efeito, S = 4n2 + 4n + 6 = 4n1 + 2, onde n1 = n2 + n + 1.
Pelo exercício (9.2.9) - a) nenhum número da forma S = 4n1 + 2 é um quadrado.
Portanto, a soma dos quadrados de quatro inteiros consecutivos nunca é um quadrado.
Para o caso de três inteiros consecutivos cuja soma seja T temos

T = (n − 1)2 + n2 + (n + 1)2 = 3n2 + 2

Seja n = 2k par, então T = 3(2k)2 + 2 = 4(3k 2 ) + 2, pelo Exercício (9.2.9) este número
não é par.
67

Seja n = 2k + 1 ímpar, então T = 3(2k + 1)2 + 2 = 4(3k 2 + 3k + 1) + 1,

Exercício 9.2.11.
a) Mostre que todo quadrado é da forma 8n, 8n + 1 ou 8n + 4. b) Mostre que nenhum
número na sequência 3, 11, 19, 27, etc., é um quadrado.
Demonstração. a)

Todo número natural n pode ser escrito de uma das formas : 4k, 4k + 1, 4k + 2 ou
4k + 3, k ∈ N. Elevando cada um desses números ao quadrado obtemos:
n2 = (4k)2 = 16k 2 = 8(2k 2 ) = 8x1 + 0
n2 = (4k + 1)2 = 16k 2 + 8k + 1 = 8(2k 2 + k) + 1 = 8x2 + 1
n2 = (4k + 2)2 = 16k 2 + 16k + 4 = 8(2k 2 + 2k) + 4 = 4x3 + 4
n2 = (4k + 3)2 = 16k 2 + 24k + 9 = 16k 2 + 24k + 8 + 1 = 8(2k 2 + 3k + 1) + 1 = 8x4 + 1.
Portanto, todo quadrado é da forma 8n, 8n + 1 ou 8n + 4.
Demonstração. b)
O primeiro termo da sequência é a1 = 3; o segundo termo da sequência é a2 =
3 + 8(1) = 11; o terceiro termo da sequência é a3 = 3 + 8(2) = 19; o quarto termo da
sequência é a4 = 3 + 8(3) = 27.
Em geral o termo n-ésimo da sequência é an = 3 + 8(n − 1).
Como qualquer termo da sequência é da forma 8(n − 1) + 3, pela parte a) deste
exercício, ele não é um quadrado.

Exercício 9.2.12.
Mostre que numa sequência de inteiros da forma

a, a + d, a + 2d, a + 3d, . . . ,

se existir algum número que é quadrado, existirão infinitos números que são quadrados.
Demonstração.
68

Exercício 9.2.13.
Dados dois inteiros a e b distintos, mostre que existem infinitos números n para os
quais mdc (a + n, b + n) = 1.
Demonstração.

Exercício 9.2.14.
Resolva o seguinte sistema de equações:
{
mdc(x, y) = 6
mmc(x, y) = 60

Demonstração.

Temos mdc(x, y) = (x, y) = 6 ⇒ 6|x, 6|y logo existem b, c ∈ N, tais que x = 6b


e y = 6c.
Temos mmc(x, y) = [x, y] = 60 ⇒ 60 = x.f = y.g, com f, g ∈ N.
Sabemos que (x, y)[x, y] = xy ⇒ 360 = xy = (6b)(6c) ⇒ 10 = bc
Como 60 = xf , bc = 10, x = 6b ⇒ xc = 6bc = 60 assim, c = f
Como 60 = yg, bc = 10, y = 6c ⇒ yb = 6cb = 60 assim, b = g

• Se b = 1 e c = 10 então x = 6 e y = 60.

• Se b = 2 e c = 5 então x = 12 e y = 30.

• Se b = 5 e c = 2 então x = 30 e y = 12.

• Se b = 10 e c = 1 então x = 60 e y = 6.

Exercício 9.2.15.
Observe que mdc(x, y) divide mmc(x, y), quaisquer que sejam x, y ∈ Z, não nulos.
a) Mostre que se no seguinte sistema:
{
mdc(x, y) = d
mmc(x, y) = m

d - m, ele não admite solução.


b) Mostre que se d|m, o sistema sempre admite solução.
Demonstração.
69

a) Temos mdc(x, y) = (x, y) = d ⇒ d|x, d|y logo existem b, c ∈ N, tais que x = bd


e y = cd.

Temos mmc(x, y) = [x, y] = m ⇒ m = x.f = y.g, com f, g ∈ N.

Se d - m, então existem k, r ∈ N, e únicos, tais que m = d.k + r, 1 < r < d.

Mas m = x.f = y.g, logo x = m/f e y = m/g

Sabemos que (x, y)[x, y] = xy ⇒ md = xy = (m/f )(m/g) ⇒ m = df g


pois f g ̸= 0

Assim temos m = d.k + r, 1 < r < d e m = df g ⇒ d(f g − k) = r, logo d|r


isto é um absurdo, pois 0 < r < d

Portanto, se d não divide m, então o sistema


{
mdc(x, y) = d
mmc(x, y) = m

não admite solução.

b) Temos mdc(x, y) = (x, y) = d ⇒ d|x, d|y logo existem b, c ∈ N, tais que


x = bd e y = cd.

Temos mmc(x, y) = [x, y] = m ⇒ m = x.f = y.g, com f, g ∈ N.

Se d | m, então existe k ∈ N, e único onde m = d.k.

Mas m = x.f = y.g, logo x = m/f e y = m/g

Sabemos que (x, y)[x, y] = xy ⇒ md = xy = (bd)(cd) ⇒ m = dbc.

Assim temos m = d.k e m = dbc ⇒ k = bc.

Portanto, se d divide m, então o sistema


{
mdc(x, y) = d
mmc(x, y) = m

admite solução.

Observação:

Observe que d e m são dados do problema, logo são números conhecidos,


k também é conhecido, pois é a multiplicidade de d em m.
A solução x = bd e y = cd nem sempre é única, depende dos fatores b e
c tais que k = bc.
70

Exercício 9.2.16.
Mostre que a) mdc(a2 , b2 ) = [mdc(a, b)]2 . b)mmc(a2 , b2 ) = [mmc(a, b)]2 . c) Genera-
lize.
Demonstração.

Exercício 9.2.17. Esse é um problema proposto no século XV I


Um total de 41 pessoas entre homens, mulheres e crianças foram a um banquete e
juntos gastaram 40 patacas. Cada homem pagou 4 patacas, cada mulher 3 patacas e cada
criança um terço de pataca. Quantos homens, quantas mulheres e quantas crianças havia
no banquete?
Demonstração.
Suponhamos sejam H o total de homens, M de mulheres e C de crianças como são 41
pessoas então
H + M + C = 41 (9.10)

Como todos juntos gastaram 40 patacas e sabemos que cada homem gastou 4 patacas,
cada mulheres 3 e cada criança 1/3 de pataca então:

4H + 3M + C/3 = 40 (9.11)

onde H, M ∈ N consequentemente C/3 ∈ N.


Desta forma vamos multiplicar os dois membros da equação 4H + 3M + C/3 = 40 por
3 assim teremos 12H + 9M + C = 120.
Multiplicando por 3 a igualdade (9.11) e resolvendo com a igualdade (9.10) resulta
11H + 8M = 79.
A resolver esta última equação para números naturais
Como 11H + 8M = 79 e (8, 11) = 1 assim, 1 = 8(−4) + 11(3) ⇒ 11(237) +
8(−316) = 79
Logo, 11H + 8M = 79 ⇔ 11(237 − 8t) + 8(11t − 316) = 79, t ∈ Z.
Como H = 237−8t ≥ 0 e M = 11t−316 ≥ 0 ⇒ 28, 72 ≤ t ≤ 29, 62 ⇒ t = 29.
Assim H = 5, M = 3 e C = 33.
Portanto, havia no banquete 5 homens, 3 mulheres e 33 crianças.
Exercício 9.2.18. Proposto por Euler
Um grupo de homens e mulheres gastaram numa taberna 1.000 patacas. Cada homem
pagou 19 patacas e cada mulher 13. Quantos eram os homens e quantas eram as mulheres?
Solução.
71

Suponha foram H homens e M mulheres, foram gastos pelos homens 19H e pelas
mulheres 13M .
Pelas condições do problema 19H + 13M = 1000.
Como 19H+13M = 1000 e (19, 13) = 1 assim, 1 = 13(3)+19(−2) ⇒ 19(−2000)+
13(3000) = 1000
Logo, 19H + 13M = 1000 ⇔ 19(13t − 2000) + 13(3000 − 19t) = 1000, t ∈ Z.
De onde H = 13t − 2000 ≥ 0 e M = 3000 − 19t ≥ 0 ⇒ 153, 84 ≤ t ≤ 157, 89
• Se t = 154, então H = 2 e M = 74.

• Se t = 155, então H = 15 e M = 55.

• Se t = 156, então H = 28 e M = 36.

• Se t = 157, então H = 41 e M = 17.


Os valores para H e M descritos acima, resolvem o problema.
Exercício 9.2.19. (Proposto por Euler)
Uma pessoa comprou cavalos e bois. Foram pagos 31 escudos por cavalo e 20 por boi
e sabe-se que todos os bois custaram 7 escudos a mais do que todos os cavalos. Quantos
cavalos e quantos bois foram comprados?
Solução.
Suponha foram comprados C cavalos e B bios, foram pagos 31C pelos cavalos e 20B
pelos bois. Pelas condições do problema 20B − 31C = 7.
Como 20B−31C = 7 e (20, 31) = 1 assim, 1 = 20(−17)−31(−11) ⇒ 20(−119)−
31(−77) = 7
Logo, 20B − 31C = 7 ⇔ 20(31t − 119) − 31(20t − 77) = 7, t ∈ Z.
Como B = 31t − 119 ≥ 0 e C = 20t − 77 ≥ 0 ⇒ 3, 83 ≤ t e 3, 55 ≤ t ⇒ t =
4, 5, . . ..
t= 4 5 6 7 8 ... t
B= 5 36 67 98 129 ... 31t − 119
C= 3 23 43 63 83 ... 20t − 77
Existem infinitas soluções para o problema, pois t ∈ N.
Exercício 9.2.20.
Em um certo país, as cédulas são de $4 e $7. Quais das afirmações a seguir são
verdadeiras? Com elas é possível pagar, sem troco, qualquer quantia inteira. a) a partir
de $11, inclusive. b) a partir de $18, inclusive. c) ímpar, a partir de $7, inclusive. d)
que seja $1 maior do que um múltiplo de $3. e) que seja $1 menor do que um múltiplo
de $3
Solução.
72

a) Suponha que 4X + 7Y = n para n ≥ 11, então como 4(2) + 7(−1) = 1 segue que
4(2n − 7t) + 7(4t − n) = n, t ∈ N.
Como 2n − 7t ≥ 0 e 4t − n ≥ 0 ⇒ 7n ≤ 28t ≤ 8n
Quando n = 11 ⇒ X = 1, Y = 1; quando n = 12 ⇒ X = 3, Y = 0,
quando n = 13 não existem cedulas que para pagar esse valor sem troco.
Portanto a) é falso.

b) Pela primeira parte, quando n ≥ 18 temos 4(2n − 7t) + 7(4t − n) = n, t ∈ N.


Como X = 2n − 7t ≥ 0 e Y = 4t − n ≥ 0 ⇒ 7n ≤ 28t ≤ 8n
Podemos representar na tabela

n 18 19 20 21 22 23 24 25 26 27 28 28 29 30
t 5 5 5 6 6 6 6 7 7 7 7 8 8 8
X 1 3 5 0 2 4 6 1 3 5 7 0 2 4
Y 2 1 0 3 2 1 0 3 2 1 0 4 3 2

Observe que 7n ≤ 28t ≤ 8n ⇒ 14n ≤ 56t ≤ 16n ⇒

8n
0 ≤ 14(4t − n) ≤ 2n ⇒ 0 ≤ 7(4t − n) ≤ n ⇒ 0 ≤ 4t ≤ ⇒
7
2n
0≤t≤ ⇒ ∃ t ≥ 5, ∀ n ≥ 18, n ∈ N
7
Portanto, esta resposta é verdadeira.

c) Pela primeira parte, quando n ≥ 7 sendo n-ímpar temos 4(2n − 7t) + 7(4t − n) = n,
t ∈ N, então X = 2n − 7t ≥ 0 e Y = 4t − n ≥ 0 ⇒ 7n ≤ 28t ≤ 8n.
Se n = 9 ⇒ 2, 25 ≤ t ≤ 2, 57, logo não existe t ∈ Z.
Portanto, não é possível pagar 9 reais.

d) Que seja $1 maior do que um múltiplo de $3. Se queremos pagar $10 com essas notas
de $4 e $7 é impossível.

e) Que seja $1 menor do que um múltiplo de $5. Se queremos pagar $9 com essas notas
de $4 e $7 é impossível.

Portanto, a resposta correta a esta questão é a letra c).


MA-14 - Aula 5

Semana 09/09 a 15/09

Unidade 10

10.2 Expressões Binômias


Nesta unidade, mostra-se como calcular o mdc de pares de números da forma an ± 1,
onde a, n ∈ N. Enunciaremos a seguir os principais resultados da unidade.

Propriedade 10.1. Se n, m, a ∈ N, com a ≥ 2, então

(am − 1, an − 1) = ad − 1; onde d = (m, n)

Lema 10.2.1. Sejam a, m, n, q, r ∈ N, com a ≥ 2 tais que m = nq + r então tem-se que

(am − 1, an + 1) = (am − 1, ar + 1)

Lema 10.2.2. Sejam a, m, n, q, r ∈ N, com a ≥ 2 tais que m = nq + r então tem-se que


{
(an + 1, ar − 1) se q é par
(am − 1, an + 1) =
(an + 1, ar + 1) se q é ímpar

Lema 10.2.3. Sejam a, m, n, q, r ∈ N, com a ≥ 2 tais que m = nq + r então tem-se que


{
m n (an + 1, ar + 1) se q é par
(a + 1, a + 1) =
(an + 1, ar − 1) se q é ímpar

m
Propriedade 10.2. Sejam n, m ∈ N, com n|m e par. Se a ∈ N, então,
n
{
1 se a é par
(am + 1, an + 1) =
2 se a é ímpar

Teorema 10.1. Se n, m, a ∈ N, com a ≥ 2, então

73
74

• (am − 1, an − 1) = a(m, n) − 1

• (am ± 1, an + 1) pode apenas assumir um dos seguintes valores: 1, 2 ou a(m, n) + 1.

Corolario 10.2.1. Tem-se que




 [m, n]

 a(m, n) + 1 se é ímpar

 (m, n)
 [m, n]
m n
(a + 1, a + 1) = 2 se é par e a ímpar

 (m, n)



 [m, n]
 1 se e a são pares
(m, n)

Corolario 10.2.2. Se a ∈ N tem-se que


 m n

 a(m, n) + 1 se é par e é ímpar
 (m, n) (m, n)
(am − 1, an + 1) = 2 caso contrário e a é ímpar



1 caso contrário e a é par

Exercício 10.2.1.
Sejam a, m, n ∈ N. Mostre que an − 1|am − 1 se, e somente se, n|m.
Solução.

Suponhamos que a, m, n ∈ N∗ onde m ≥ n.


Condição necessária (⇒)
Por hipótese an − 1|am − 1 então am − 1 = k(an − 1), k ∈ N, de onde como (k, 1) = 1

(am − 1, an − 1) = (k(an − 1), an − 1) = (an − 1) × (k, 1) = an − 1 (10.12)

Por outro lado, sabemos pela Propriedade 6.2.2 que:

(am − 1, an − 1) = a(m,n) − 1

Comparando com a equação (10.12) temos

an − 1 = a(m,n) − 1 ⇒ n = (m, n) ⇒ n|m

Condição suficiente (⇐)


Suponhamos que n|m, então m = kn para algum k ∈ N.

am − 1 = akn − 1 = (an )k − 1 = (an − 1)((an )k−1 + (an )k−2 + . . . + an + 1) = α(an − 1)

Portanto an − 1|am − 1.
75

Exercício 10.2.2.
Sejam n, m ∈ N com n|m e m, n ímpar. Se a ∈ N, mostre que (am +1, an +1) = an +1.
Solução.

Suponhamos a ≥ 2, para o caso a = 0 ou a = 1 é imediato.


Como n|m, seja m = nq, então como m e n são ímpares, segue que q também é ímpar,
logo

am + 1 = anq + 1 = (an )q + 1 = (an + 1)[(an )q−1 − (an )q−2 + . . . + an − 1] = β(an + 1)

Assim, para β ∈ N, temos que (am + 1, an + 1) = (β(an + 1), an + 1) = an + 1

Exercício 10.2.3.
Sejam a, m, n ∈ N, com m > n. Mostre que (a2 − 1, a2 + 1) = a2 + 1.
m n n

Solução.

Se m > n ⇒ m = nq + r logo M = 2m = (2n )q 2r = kP , onde P = 2n , k =


2r × 2n × 2n × . . . × 2n com q fatores, além disso M, P e k ≥ 2 são pares.

m n
(a2 − 1, a2 + 1) = (aM − 1, aP + 1) = (akP − 1, aP + 1)

Como k − 1 é ímpar, então podemos escrever

(aP )k−1 + 1 = (aP + 1)[(aP )k−2 − . . . + 1] = β(aP + 1)

logo aplicando o Lema de Euclides

n
= (aP [(aP )k−1 + 1] − (aP + 1), aP + 1) = (aP + 1, aP + 1) = aP + 1 = a2 + 1

Exercício 10.2.4.
Calcule: a) (5202 + 1, 574 + 1) b) (36497 + 1, 36210 + 1) c) (3144 − 1, 378 + 1)
Solução.

a) (5202 + 1, 574 + 1) = (52×74+54 + 1, 574 + 1) = (554 (52×74 − 1) + 554 + 1, 574 + 1) =

= (554 (574 −1)(574 +1)+554 +1, 574 +1) = (α(574 +1)+554 +1, 574 +1) = (554 +1, 574 +1) =

Pela Proposição 6.2.1 segue que (5202 + 1, 574 + 1) = 5(202,74) + 1 = 52 + 1 = 26

b) (36497 + 1, 36210 + 1) = (362×210+77 + 1, 36210 + 1) = (3677 (362×210 − 1) + 3677 + 1, 36210 +


1) =
= (α(36210 + 1) + 3677 + 1, 36210 + 1) = (3677 + 1, 36210 + 1) =
76

(3677 + 1, 36210 + 1) = (3677 + 1, 3656 (362×77 − 1) + 3656 + 1) =

= (3677 + 1, β(3677 + 1) + 3656 + 1) = (3677 + 1, 3656 + 1)

Pela Proposição 6.2.1 segue que (36497 + 1, 36210 + 1) = 36(497,210) + 1 = 367 + 1 =


78364164097

c) (3144 − 1, 378 + 1) = (378+66 − 1, 378 + 1) = (366 (378 + 1) − 366 − 1, 378 + 1) =

= (366 (378 +1)−(366 +1), 378 +1) = (α(378 +1)−(366 +1), 378 +1) = (366 +1, 378 +1) =

Pela Proposição 6.2.1 segue que (3144 − 1, 378 + 1) = 3(144,78) + 1 = 36 + 1 = 730

Problema 10.1.
Seja (Mn )n a sequência definida por Mn = 2n − 1. Mostre que
a) 3|Mn se, e somente se, n é par

b) 5|Mn se, e somente se, n é múltiplo de 4.

c) 9|Mn se, e somente se, n é múltiplo de 9.


Solução.

a) Suponhamos que 3|Mn então 3|2n − 1, logo existe α ∈ N tal que 2n − 1 = 3α.
Se n-par temos n = 2k de onde

3α = 2n −1 = 22k −1 = (22 )k −1 = (22 −1)[(22 )k−1 +(22 )k−2 +(22 )k−3 +. . .+22 +1] =

logo α = (22 )k−1 + (22 )k−2 + (22 )k−3 + . . . + 22 + 1 ∈ N.


Se n-ímpar, n = 2k + 1 de onde

3α = 2n − 1 = 22k+1 − 1 = 2[22k − 1] + 1 = 2(3β) + 1 = 3γ + 1

isto é um absurdo!. Um múltiplo de 3 não pode ser multiplo de 3 mais uma unidade.
Portanto, se 3|Mn então n é par.
Inversamente (⇐)
Suponhamos n-par, então podemos escrever n = 2k para k ∈ N, logo

2n − 1 = 22k − 1 = (22 )k − 1 = (22 − 1)((22 )k−1 + (22 )k−2 + (22 )k−3 + . . . + 22 + 1) = 3β

Assim, 3|2n − 1.
Portanto, se n-par, então 3|Mn .
77

b) Suponhamos que 5|Mn então 5|2n − 1, logo existe α ∈ N tal que 2n − 1 = 5α.
Todo número natural n podemos escrever na forma 4k, 4k + 1, 4k + 2 ou 4k + 3.
Se n = 4k temos

5α = 2n −1 = 24k −1 = (24 )k −1 = (24 −1)[(24 )k−1 +(24 )k−2 +(24 )k−3 +. . .+24 +1] =

logo α = 3[(24 )k−1 + (24 )k−2 + (24 )k−3 + . . . + 24 + 1] ∈ N.


Se n = 4k + 1 temos pela primeira parte deste item

5α = 24k+1 − 1 = 2[24k − 1] + 1 = 5β1 + 1 ⇒ 5|1 absurdo !

Se n = 4k + 2 temos pela primeira parte deste item

5α = 24k+2 − 1 = 22 [24k − 1] + 3 = 5β2 + 3 ⇒ 5|1 absurdo !

Se n = 4k + 3 temos pela primeira parte deste item

5α = 24k+3 − 1 = 23 [24k − 1] + 7 = 5β3 + 2 ⇒ 5|1 absurdo !

Portanto, se 5|Mn então n é múltiplo de 4.


Inversamente (⇐)
Suponhamos n múltiplo de 4, então podemos escrever n = 4k para k ∈ N, logo

2n − 1 = (24 )k − 1 = (24 − 1)((24 )k−1 + (22 )k−2 + (22 )k−3 + . . . + 22 + 1) = 5β

Assim, 5|2n − 1.
Portanto, se n múltiplo de 4, então 5|Mn .

c) Suponhamos que 9|Mn então 9|2n − 1, logo existe α ∈ N tal que 2n − 1 = 9α.
78

Unidade 11
Números de Fibonacci

11.1 Números de Fibonacci


Lema 11.1.1. Dois termos consecutivos da sequência de Fibonacci são primos entre si.

Lema 11.1.2. Se n; m ∈ N são tais que n|m, então, un |um .

Teorema 11.2. Seja (un )n a sequência de Fibonacci; então,

(um , un ) = u(m, n)

Corolário 11.1.1. Na sequência de Fibonacci, temos que un divide um se, e somente se,
n divide m.

O resultado anterior nos permite estabelecer alguns critérios de divisibilidade para os


termos da sequência de Fibonacci.
Assim, para acharmos, por exemplo, os termos um da sequência de Fibonacci divisíveis
por 3, basta notar que u4 = 3 e que

3|um ⇔ u(4, m) = (u4 , um ) = (3, um ) = 3 = u4 ;

e, portanto, 3|um se, e somente se, (4, m) = 4, o que equivale a dizer que 4|m.

Exercício 11.1.1.
Mostre que, se na sequência de Fibonacci existir um termo divisível por um número
natural m, então existem infinitos tais termos.
Demonstração.

Seja un tal que m|un , então un = αm. Suponhamos que uk seja o menor elemento da
sequência de Fibonaci tal que seja divisível por m, logo

m|un ⇔ u(n,k) = (un , uk ) = (αm, uk ) = βm, β - m, β|α

Como m|un , então un = α1 (βm), α = α1 β, uk = γ · βm. Logo pelo Corolário da


pg. 80, como m|βm então um |uβm , isto é uβm = p · um .
Portanto, na sequência de Fibonacci se existir um termo divisível por um número
natural m, então existem infinitos tais termos da forma uβm = p · um .

Exercício 11.1.2.
Na sequência de Fibonacci, mostre que
79

a) um é par se, e somente se, m é divisível por 3.

b) um é divisível por 5 se, e somente se, m é divisível por 5.

c) um é divisível por 13 se, e somente se, m é divisível por 7.

d) um é divisível por 7 se, e somente se, m é divisível por 8.

Demonstração.

a) Temos: u1 = 1, u2 = 1, u3 = 2, u4 = 3, u5 = 5, u6 = 8, u7 = 13, u8 =
21, u9 = 34 . . .
Suponhamos um = 2k, k ∈ N, então como u3 = 2

2|um ⇔ u(m,3) = (um , u3 ) = (2k, 2) = 2 = u3

Portanto, 2|um ⇔ (m, 3) = 3, isto é 3|m.

b) Suponhamos um = 5k, k ∈ N, então como u5 = 5

5|um ⇔ u(m,5) = (um , u5 ) = (um , 5) = 5 = u5

Portanto, 5|um ⇔ (m, 5) = 5, isto é 5|m.

c) Suponhamos um = 13k, k ∈ N, então como u7 = 13

13|um ⇔ u(m,7) = (um , u7 ) = (um , 13) = 13 = u7

Portanto, 13|um ⇔ (m, 7) = 7, isto é 7|m.

d) Suponhamos um = 7k, k ∈ N, então como u8 = 21

7|um ⇔ u(m,8) = (um , u8 ) = (um , 21) = 21 = u8

Portanto, 7|um ⇔ (m, 8) = 8, isto é 8|m.

Exercício 11.1.3.
Na sequência de Fibonacci, mostre que

a) um é divisível por 21 se, e somente se, m é divisível por 8.

b) um é divisível por 8 se, e somente se, m é divisível por 6.

c) um é divisível por 4 se, e somente se, m é divisível por 6.


80

Demonstração.

a) Suponhamos um = 21k, k ∈ N, então como u8 = 21

21|um ⇔ u(m,8) = (um , u8 ) = (um , 21) = 21 = u8

Portanto, 21|um ⇔ (m, 8) = 8, isto é 8|m.

b) Suponhamos um = 8k, k ∈ N, então como u6 = 8

8|um ⇔ u(m,6) = (um , u6 ) = (um , 8) = 8 = u6

Portanto, 8|um ⇔ (m, 6) = 6, isto é 6|m.

c) Temos: u10 = 55, u11 = 89, u12 = 144, u13 = 133, u14 = 277, u15 =
410, u16 = 687, u17 = . . .
Suponhamos um = 4k, k ∈ N, então como u6 = 8

4|um ⇔ u(m,6) = (um , u6 ) = (um , 8) = 8 = u6

Portanto, 4|um ⇔ (m, 6) = 6, isto é 6|m.

Exercício 11.1.4.
Dados n, m ∈ N∗ , com n ≥ 2, mostre que se un é o termo de ordem n da sequência
de Fibonacci, então un+m = un−1 um + un um+1 .
Demonstração.

Mostrarei por indução sobre n,


Seja m ∈ N∗ fixo, sabe-se que u1 = u2 = 1, uk = uk−1 + uk−2 , ∀ k > 2, logo, se
n = 2 é verdade que
u2+m = u1+m + um = u1 um + u2 um+1

Suponhamos que, para 2 < n ≤ h seja verdade que uh+m = uh−1 um + uh um+1 .
Por outro lado, para h + 1, aplicando a hipótese auxiliar temos:

uh+1+m = uh+m + uh−1+m = [uh−1 um + uh um+1 ] + [uh−2 um + uh−1 um+1 ]

= [uh−1 + uh−2 ]um + [uh + uh−1 ]um+1 = uh um + uh+1 um+1

Fixando n, mostra-se o mesmo resultado por indução para m ∈ N.


Portanto, un+m = un−1 um + un um+1 para todo n, m ∈ N∗ , com n ≥ 2.
MA-14 - Aula 6

Semana 16/09 a 22/09

Unidade 12

12.2 Teorema Fundamental Da Aritmética


Um número natural maior do que 1 que só possui como divisores positivos 1 e ele
próprio é chamado de número primo.
Dados dois números primos p e q e um número inteiro a qualquer, decorrem da
definição acima os seguintes fatos:

i) Se p|q então p = q.

ii) Se p - a então (p, a) = 1

Um número maior do que 1 e que não é primo será chamado composto. Portanto, se
um número inteiro n > 1 é composto, existirá um divisor natural n1 de n tal que n1 ̸= 1
e n1 ̸= n. Portanto, existirá um número natural n2 tal que

n = n1 n2 , com 1 < n1 < n, e 1 < n2 < n

Por exemplo, 2, 3, 5, 7, 11 e 13 são números primos, enquanto que 4, 6, 8, 9, 10 e


12 são compostos.
Do ponto de vista da estrutura multiplicativa dos naturais, os núumeros primos são
os mais simples e ao mesmo tempo são suficientes para gerar todos os números naturais,
logo todos os números inteiros não nulos, conforme veremos mais adiante no Teorema
Fundamental da Aritmética.
A seguir, estabelecemos um resultado fundamental de Euclides (Os Elementos, Pro-
posição 30, Livro V II).

Propriedade 12.3. (Lema de Euclides)


Sejam a; b; p ∈ Z, com p primo. Se p|ab, então p|a ou p|b.

81
82

Na realidade, a propriedade dos números primos descrita na proposição acima, os


caracteriza totalmente (Veja Problema 12.1.10).

Corolario 12.2.1. Se p, p1 , · · · , pn são números primos e, se p|p1 · · · pn , então p = pi


para algum i = 1, . . . , n.

Teorema 12.3. Todo número natural maior do que 1 ou é primo ou se escreve de modo
único (a menos da ordem dos fatores) como um produto de números primos.

Agrupando no teorema os fatores primos repetidos, se necessário, e ordenando os


primos em ordem crescente, temos o seguinte enunciado:

Teorema 12.4. Dado um número inteiro n ̸= 0, 1, −1, existem primos p1 < . . . < pr e
α1 , . . . , αr ∈ N, unívocamente determinados, tais que

n = ±pα1 1 · pα2 2 · pα3 3 · · · pαr r

Propriedade 12.4. Seja n = ±pα1 1 · pα2 2 · pα3 3 · · · pαr r um número natural escrito na forma
acima.
Se n′ é um divisor positivo de n, então n′ = pβ1 1 · pβ2 2 · pβ3 3 · · · pβr r ; onde 0 < βi < αi ,
para i = 1, . . . , r.

Denotando por d(n) o número de divisores positivos do número natural n, temos que
se n = pα1 1 · pα2 2 · pα3 3 · · · pαr r , onde p1 , . . . , pr sao numeros primos e α1 ; :::; αr ∈ N, então

d(n) = (α1 + 1)(α2 + 1) · · · (αr + 1)

A fatoração de números naturais em primos revela toda a estrutura multiplicativa


desses números, permitindo, entre muitas outras coisas, determinar facilmente o mdc e o
mmc de um conjunto qualquer de números.

Teorema 12.5. Sejam a = pα1 1 · pα2 2 · pα3 3 · · · pαnn e b = pβ1 1 · pβ2 2 · pβ3 3 · · · pβnn pondo

γi = min{αi , βi }, δi = max{αi , βi }, i = 1, 2, . . . , n

temos que
(a, b) = pγ11 · pγ22 · pγ33 · · · pγnn [a, b] = pδ11 · pδ22 · pδ33 · · · pδnn

Seja n > 4 um número natural, vamos provar que n é composto se, e somente se,
n|(n − 2)!.
Suponhamos n composto.
83

Provaremos inicialmente que n|(n − 1)!. De fato, suponha que n = n1 n2 com 1 < n1 <
n e 1 < n2 < n. Se n1 ̸= n2 , podemos supor que 1 < n1 < n2 , e portanto,

(n − 1)! = 1 · · · n1 · · · n2 · · · (n − 1)

o que mostra que n|(n − 1)!, neste caso.


Suponhamos que n1 = n2 > 2, neste caso 2n1 < n1 n2 = n. Logo,

(n − 1)! = 1 · · · n1 · · · (2n1 ) · · · (n − 1)

o que implica também que n(= n1 n1 ) divide (n − 1)!.


Agora, note que (n; n − 1) = 1 e que n|(n − 2)!(n − 1); portanto, n|(n − 2)!.
Reciprocamente, se n|(n − 2)!, n não pode ser primo, pois é maior do que os fatores
primos de (n − 1)!.
A propriedade acima pode ser generalizada como segue:
Se n > 4 é composto e p é o menor número primo que divide n, então, n|(n − p)!.
De fato, temos que

(n − 1; n) = (n − 2; n) = · · · = (n − (p + 1); n) = 1

Logo, segue-se que

((n − 1)(n − 2) · · · (n − p + 1); n) = 1

o que, em vista do fato de n|(n − 1)!, acarreta o resultado.

12.3 Problemas
Exercício 12.3.1.
Ache os possíveis valores de n, m ∈ N ∪ {0} de modo que o número 9m 10n tenha :
a) 27 divisores b) 243 divisores.
Solução.

a) Seja A = 9m 10n = 32m 5n 2n , o máximo número de divisores de A é dado por

d(A) = (2m + 1)(n + 1)(n + 1) = 27 = 3 × 32

logo 2m + 1 = 3 e (n + 1)2 = 9 assim, m = 1 e n = 2


Também temos o caso d(A) = (2m + 1)(n + 1)(n + 1) = 27 = 27 × 12 , neste caso
m = 13 e n = 0.
84

b) Seja B = 9m 10n = 32m 5n 2n , o máximo número de divisores de B é dado por

d(B) = (2m + 1)(n + 1)(n + 1) = 243 = 3 × 92

logo 2m + 1 = 3 e (n + 1)2 = 92 assim, m = 1 e n = 8


Também temos o caso d(B) = (2m + 1)(n + 1)(n + 1) = 243 = 243 × 12 , neste caso
m = 121 e n = 0.

Exercício 12.3.2.
Qual é a forma geral dos números naturais que admitem: a) um só divisor além de
1 e dele próprio? b) um número primo de divisores?
Solução.

a) O número da forma p21 , sendo p1 primo, tem três divisores.


Com efeito, se a = pα1 1 · pα2 2 · pα3 3 · · · pαnn então pelas condições do problema temos
que d(a) = (α1 + 1)(α2 + 1) · · · (αr + 1) = 3 de onde (α1 + 1) = 3 ⇒ α1 = 2
Portanto o número é da forma a = p21 .

b) O número da forma pp−1


1 , onde p é número primo, sendo p1 primo.

Com efeito, se a = pα1 1 · pα2 2 · pα3 3 · · · pαnn então pelas condições do problema temos
que d(a) = (α1 + 1)(α2 + 1) · · · (αr + 1) = p de onde (α1 + 1) = p ⇒ α1 = p − 1
Portanto o número é da forma a = pp−1
1 .

Exercício 12.3.3.
Sejam a, b ∈ N, com (a, b) = 1. Mostre que, se ab é um quadrado, então a e b são
quadrados. Generalize para ab uma potência r-ésima.
Solução.

Seja ab = N 2 onde N = pβ1 1 · pβ2 2 · pβ3 3 · · · pβnn , assim


( )2
ab = pβ1 1 · pβ2 2 · pβ3 3 · · · pr nβn = p2β
1
1
· p2β
2
2
· p2β
3 · · · pn
3 2βn

Sendo ab = pα1 1 · pα2 2 · pα3 3 · · · pαnn um quadrado, isto somente ocorre se e somente se
αi = 2βi , i = 1, 2, . . . , n.
Como (a, b) = 1 então não existem divisores comuns, logo eles são da forma a =

n ∏
n
α ∏
n ∏
n

pαi i e b = pj j onde i, j = 1, 2, . . . , n, i ̸= j. Isto é a = p2β
i
i
e b = pj j
i=1 j=1 i=1 j=1
onde i, j = 1, 2, . . . , n, i ̸= j.
Portanto, se ab é um quadrado sendo (a, b) = 1, então a e b cada um deles são
quadrados.
85

Para o caso ser ab uma potência r-ésima.


Seja ab = N r onde N = pβ1 1 · pβ2 2 · pβ3 3 · · · pβnn , assim
( )2
ab = pβ1 1 · pβ2 2 · pβ3 3 · · · pr nβn = p2β
1
1
· p2β
2
2
· p2β
3 · · · pn
3 2βn

Sendo ab = pα1 1 ·pα2 2 ·pα3 3 · · · pαnn uma potência r-ésima, isto somente ocorre se e somente
se αi = rβi , i = 1, 2, . . . , n.
Como (a, b) = 1 então não existem divisores comuns, logo eles são da forma a =
∏n ∏n
α ∏n ∏n

pαi i e b = pj j onde i, j = 1, 2, . . . , n, i ̸= j. Isto é a = pnβ
i
i
e b= pj j
i=1 j=1 i=1 j=1
onde i, j = 1, 2, . . . , n, i ̸= j.
Portanto, se ab é uma potência r-ésima sendo (a, b) = 1, então a e b cada um deles é
uma potência r-ésima.

Exercício 12.3.4.
Seja m ∈ N. Pode o número m(m + 1) ser a sétima potência de um número natural?
(Generalize.)
Solução.

Nem sempre, pois se m = 6 então m(m + 1) = 42 que não é sétima potência de algúm
número.
Por outro lado, em geral suponhamos que k ∈ N é tal que m(m + 1) = k 7 para m ∈ N.
Se k = 2s + 1 ⇒ k 7 = β + 1 = m(m + 1) porém m(m + 1) sempre é número par,
assim k nunca poderá ser ímpar.
Se k = 2s

Exercício 12.3.5.
(ENC-2002) Qual é o menor valor do número natural n que torna n! divisível por
1000?
Solução.

Temos 1000α = 103 α = 23 53 α = n! = 1 × 2 × 3 × (22 ) × 5 × · · · (2 × 5) × · · · × (3 × 5)


Logo n = 15.
Isto é, para que seja possível que n! ter números cujo produto seja 1000, deve acontecer
1000 = 23 53
Com o produto de três números pares conseguimos o 23 ; para o caso 53 acontece
quando multiplicamos os números 5, 10 e 15, logo 15! = 1000 × k, k ∈ N.
86

Exercício 12.3.6.
Mostre que a soma de todos os números naturais menores ou iguais a n divide o seu
produto se, e somente se, n + 1 é composto.
Solução.

O número n + 1 será composto sempre que n ≥ 3.


(⇒) Condição necessária.
Suponhamos que a soma de todos os números naturais menores ou iguais a n divide o
seu produto, isto é

n(n + 1) α
α· = n! ⇒ (n + 1) = (n − 1)!
2 2
2
Portanto o número (n + 1) = · (n − 1)! é composto.
α
(⇐) Condição suficiente.
Suponhamos que n + 1 seja composto, então n + 1 = n1 n2 e como (n, n + 1) = 1 segue
que 1 < n1 < n e 1 < n2 < n.
Seja n1 < n2 , então n! = 1 · 2 · 3 · · · n1 · · · n2 · · · (n − 1)n. Assim, reordenando estes
números
n(n + 1)
n! = 1 · 2 · 3 · · · n · (n1 n2 ) = αn(n + 1) = 2α
2

n(n − 1)
então n!.
2
Portanto, a soma de todos os números naturais menores ou iguais a n divide o seu
produto.
Suponhamos n1 = n2 , então sabemos que n! = 1 · 2 · 3 · · · n1 · · · (n − 1)n. Assim,
reordenando estes números

1 n(n + 1)
n! = [1 · 2 · 3 · · · n1 · · · n · (n1 n2 )] = αn(n + 1) = 2α
n1 2

n(n − 1)
então n!.
2
Portanto, se n + 1 é composto, então a soma de todos os números naturais menores
ou iguais a n divide o seu produto.

Exercício 12.3.7.
Usando a caracterização de mdc e mmc de dois números a e b através da fatoração
em primos desses números, prove que (a, b)[a, b] = ab.
Solução.

Sejam p1 , p2 , . . . , pr números primos, sabe-se que todo número a, b ∈ N podemos es-


pr , com αi , βi ∈ N∗ .
α β
crever na forma a = pα1 1 pα2 2 pα3 3 . . . pr−1
r−1 αr
pr e b = pβ1 1 pβ2 2 pβ3 3 . . . pr−1
r−1 βr
87

Pelo teorema 7.2, considerando δi = max{ αi , βi }, e σi = min{ αi , βi }.

σ δ
(a, b) = pσ1 1 pσ2 2 pσ3 3 . . . pr−1
r−1 σr
pr e [a, b] = pδ11 pδ22 pδ33 . . . pr−1
r−1 δr
pr

logo
σ +δr−1 σr +δr
(a, b)[a, b] = pσ1 1 +δ1 pσ2 2 +δ2 pσ3 3 +δ3 . . . pr−1
r−1
pr (12.13)

Sabemos que

αi + βi + |αi − βi | αi + βi − |αi − βi |
δi = max{ αi , βi } = e σi = min{ αi , βi } =
2 2

Somando estas duas igualdades temos δi + βi = αi + βi , ∀ i = 1, 2, . . . r assim, em


(12.13) segue

α +βr−1 αr +βr
(a, b)[a, b] = pα1 1 +β1 pα2 2 +β2 pα3 3 +β3 . . . pr−1
r−1
pr ⇒

α β
(a, b)[a, b] = pα1 1 pα2 2 pα3 3 . . . pr−1 pr × pβ1 1 pβ2 2 pβ3 3 . . . pr−1
r−1 αr r−1 βr
pr = ab

Portanto, (a, b)[a, b] = ab.

Exercício 12.3.8.
Mostre que todo número primo p > 2 escreve-se de modo único como diferença de dois
quadrados.
Demonstração.

Exercício 12.3.9.
Seja p > 1 um número natural com a seguinte propriedade: Se p divide o produto de
dois inteiros quaisquer, então p divide um dos fatores. Mostre que p é necessáriamente
primo.
Demonstração. Primeira

Suponhamos que p divide o produto dos números a, b ∈ Z.


a) Se a e b são primos, então p é primo.
88

b) Se a é primo, b composto e p|a, então p é primo. Se p - a então p|b.


β
Suponhamos que b = pβ1 1 pβ2 2 pβ3 3 . . . pr−1
r−1 βr
pr , onde os pi são primos.

pr ), então p|p1 · a(p1β1 −1 pβ2 2 pβ3 3 . . . pr−1


β β
Como p|ab então p|a(pβ1 1 pβ2 2 pβ3 3 . . . pr−1
r−1 βr r−1 βr
pr ). Se
p|p1 então p = p1 , logo p é primo. Se p - p1 então p - p1β1 −1 e p|apβ2 2 pβ3 3 . . . pr−1
r−1 βrβ
pr

Se p|p2 então p = p2 , logo p é primo. Se p - p2 então p - p2β2 −1 e p|apβ3 3 pβ4 4 . . . pr−1


r−1 βrβ
pr

Se p|p3 então p = p3 , logo p é primo. Se p - p3 então p - p3β3 −1 e p|apβ4 4 pβ5 5 . . . pr−1


r−1 βr β
pr .
Podemos continuar com este processo obtendo sempre p primo, até chegar a uma expressão
do tipo, se p|pr−1 então p = pr−1 , logo p é primo.
−1
e p|pr (apβr r −1 ). Se p|pr então p = pr , logo p é primo, se
β
Se p - pr−1 então p - pr−1r−1

p - pr então p - pβr r −1 e como p - a é um absurdo!.

Portanto, se p|ab em particular a um dos fatores de a ou b, então p primo.


α β
c) Se a e b compostos da forma a = pα1 1 pα2 2 pα3 3 . . . pr−1 pr e b = pβ1 1 pβ2 2 pβ3 3 . . . pr−1
r−1 αr r−1 βr
pr ,
∗ αr−1 αr
com αi , βi ∈ N , e suponhamos que p|a, então p|p1 p2 p3 . . . pr−1 pr . Como na parte
α1 α2 α3

(b) temos que p|pi para algum pi primo, logo p é primo.


Análogo para o caso p|b.
Portanto, se p divide o produto de dois inteiros quaisquer, então p divide um dos
fatores, e p é necessáriamente primo.
Demonstração. Segunda
São
Hipótese: Se p divide o produto de dois inteiros quaisquer, então p divide um dos
fatores.
Tese: p é necessáriamente primo.
Temos que mostrar

(∀ a, b ∈ Z)(p|ab ⇒ p|a ou p|b) ⇒ q é primo

Po contradição.
Temos que mostrar

q não é primo ⇒ (∃ a, b ∈ Z)(p|ab e p - a e p - b)

Com efeito, suponhamos que p > 1 não é primo, logo é composto, ou seja, existem
naturais, a, b ∈ N com 1 < a < p e 1 < b < p tais que p|ab.
Além disso p - a e p - b pois 1 < a < p e 1 < b < p.
Portanto, se p divide o produto de dois inteiros quaisquer, então p divide um dos
fatores, e p é necessáriamente primo.
89

Exercício 12.3.10.
Mostre que, se n e m são dois números naturais não nulos tais que (n, m) = 1, então
d(n · m) = d(n) · d(m).
Solução.

Exercício 12.3.11.
Mostre que, se n é composto, então o n-ésimo número de Fibonacci un é composto.
Solução.

Sendo n composto podemos supor n = ab, logo ua |un e ub |un , assim temos que

ua ub ua ub
un e un ⇒ un = · · α, α∈Z
(a, b) (a, b) (a, b) (a, b)

Portanto, se n é composto, então o n-ésimo número de Fibonacci un é composto.


90

Unidade 13
Pequeno Teorema de Fermat

13.1 Pequeno Teorema de Fermat


A demonstração do Teorema de Fermat se baseia no lema a seguir.
( )
p
Lema 13.1.1. Seja p um número primo. Os números , onde 0 < i < p, são todos
i
divisíveis por p.

Teorema 13.6. (Pequeno Teorema de Fermat)


Dado um número primo p, tem-se que p divide o número ap − a, para todo a ∈ Z.

Exemplo
Dado um número qualquer n ∈ N, tem-se que n9 e n, quando escritos na base 10, têm
o mesmo algarismo da unidade.
A afirmação acima é equivalente a 10|n9 − n.
Como n9 e n têm a mesma paridade, segue-se que n9 − n é par; i.e, 2|n9 − n.
Por outro lado,

n9 − n = n(n4 − 1)(n4 + 1) = (n5 − n)(n4 + 1)

Logo, pelo Pequeno Teorema de Fermat, temos que 5|n5 − n e, portanto, 5|n9 − n.
Tem-se, então, que 10|n9 − n.

13.2 Problemas
Exercício 13.2.1.
Mostre que 42|a7 − a para todo número natural a.
Solução.

Pelo pequeno teorema de Fermat, sabe-se que 7|a7 − a para todo a ∈ N.


Por outro lado, sabemos que o produto de três números consecutivos sempre é múltiplo
de 6, logo

a7 − a = a(a6 − 1) = a(a − 1)(a2 + a + 1)(a + 1)(a2 − a + 1) =

= (a − 1)a(a + 1) · (a2 + a + 1)(a2 − a + 1) = 6K(a2 + a + 1)(a2 − a + 1)

isto é 6|a7 − a.
91

Como (6, 7) = 1 então a7 − a = 6 × 7 × β = 42β, β ∈ N.


Portanto, 42|a − a para todo número natural a.
7

Exercício 13.2.2.
Ache o resto da divisão de 12p−1 por p quando p é primo.
Solução.

Suponhamos que 12 seja um número não divisível por p-primo, então pelo corolário
do teorema de Fermat segue que

12p−1 − 1 = αp, p = 5, 7, 11, 13, 17, 19, . . .

logo o resto sempre é 1.


Se 12 é divisível por p, então p = 2 ou p = 3. Quando p = 2, o resto de dividir 12p−1
por p é zero.
Quando p = 3, o resto de dividir 12p−1 por p é zero.

Exercício 13.2.3.
3 5 2 3 11
Mostre que, para todo n ∈ N, é natural o número n + n + n.
5 3 15
Solução.
3 2 11
Seja P = n5 + n3 + n, podemos agrupar na forma
5 3 15
3 3 2 2 11
P = (n5 − n) + n + (n3 − n) + n + n
5 5 3 3 15

Aplicando o pequeno teorema de Fermat

3 2 30
P = · 5α + · 3β + n = 3α + 2β + 2n, α, β ∈ N
5 3 15
3 5 2 3 11
Portanto, o número n + n + n é natural para todo n ∈ N.
5 3 15
Exercício 13.2.4.
Mostre que, para todo n ∈ N, 15|3n5 + 5n3 + 7n.
Solução.

Sabe-se que n5 − n = 5α, n3 − n = 3β, logo 3n5 − 3n = 15α, 5n3 − 5n = 15β.


Assim, 3n5 + 5n3 = 15(α + β) + 8n de onde

3n5 + 5n3 + 7n = 15(α + β) + 8n + 7n = 15(α + β + n)

Portanto, 15|3n5 + 5n3 + 7n.


92

Exercício 13.2.5.
Seja n ∈ N. Mostre que: a) Se 5 - n, 5 - n − 1, 5 - n + 1, então 5|n2 + 1. b) Se
7 - n, 7 - n − 1, 7 - n3 + 1, então 7|n2 + n + 1.
Solução.

a) Em geral, temos pelo pequeno teorema de Fermat que 5|n5 − n, isto é n5 − n =


5α, α ∈ N, de onde

n5 − n = n(n4 − 1) = n(n2 − 1)(n2 + 1) = n(n − 1)(n + 1)(n2 + 1) = 5α

Como 5 - n, 5 - n − 1, 5 - n + 1, então 5|n2 + 1.

b:) Em geral, temos pelo pequeno teorema de Fermat que 7|n7 − n, isto é n7 − n =
7β, β ∈ N, de onde

n7 − n = n(n6 − 1) = n(n3 − 1)(n3 + 1) = n(n − 1)(n2 + n + 1)(n3 + 1) = 7β

Como 7 - n, 7 - n − 1, 7 - n3 + 1, então 7|n2 + n + 1.

Exercício 13.2.6.
Sejam a, k ∈ N. Mostre que 7|a6k − 1, se (a, 7) = 1. Generalize.
Solução.

Por indução sobre k.


Se k = 0 temos para (a, 7) = 1, e pelo corolário do pequeno teorema de Fermat,
7|a0 − 1.
Se k = 1 temos para (a, 7) = 1, e pelo corolário do pequeno teorema de Fermat,
7|a6 − 1.
Suponhamos que para k = h temos para (a, 7) = 1, e pelo corolário do pequeno
teorema de Fermat, 7|a6k − 1 isto é a6k − 1 = 7α, α ∈ N.
Seja k = h + 1, e (a, 7) = 1, logo

a6(k+1) − 1 = a6k+6 − 1 = (a6k )6 − 1 = ((a6k )3 − 1)((a6k )3 + 1) =

Da hipótese auxiliar

= (a6k − 1)((a6k )2 + (a6k ) + 1)((a6k )3 + 1) = 7α((a6k )2 + (a6k ) + 1)((a6k )3 + 1)

isto é 7|a6(k+1) − 1.
Portanto, temos que 7|a6k − 1, se (a, 7) = 1.
93

Exercício 13.2.7.
Um terno de primos é dito de primos trigêmeos se for da forma p; p + 2 e p + 4.
Mostre que 3; 5 e 7 é o único terno de primos trigêmeos..
Solução.

Exercício 13.2.8.
a)Mostre que a12 −b12 é divisível por 13, se a e b são primos com 13. b)Mostre também
que é divisível por 91, se a e b são primos com 91.
Solução.

a) Seja P = a12 − b12 então P = (a12 − 1) − (b12 − 1).


Pelo corolário do pequeno teorema de Fermat, temos que, como 13 - a, então a13−1 −1 =
13δ, δ ∈ N. De modo análogo, temos que b13−1 − 1 = 13γ, γ ∈ N. Assim

P = (a12 − 1) − (b12 − 1) = 13δ − 13γ = 13(δ − γ)

Portanto, a12 − b12 é divisível por 13, se a e b são primos com 13.
b) Seja M = a12 − b12 então M = [(a2 )6 − 1] − [(b2 )6 − 1].
Como (a, 91) = (b, 91) = 1, então 7 - a, em particular 7 - a2 . De modo análogo 7 - b2 ,
de onde (a2 , 7) = (b2 , 7) = 1.
Pelo corolário do pequeno teorema de Fermat, temos que (a2 )7−1 − 1 = 7δ, δ ∈ N.
De modo análogo, temos que (b2 )7−1 − 1 = 7γ, γ ∈ N. Assim

M = (a12 − 1) − (b12 − 1) = (a2 )6 − (b2 )6 = 7δ − 7γ = 7(δ − γ)

Logo, a12 − b12 é divisível por 7, se a e b são primos com 91.


Pela parte (a) a12 − b12 é divisível por 13, e como também a12 − b12 é divisível por 7
sendo (13, 7) = 1.
Portanto, a12 − b12 é divisível por 91 = 13 × 7.

13.3 Exercícios suplementares


Exercício 13.3.1.
Com quantos zeros termina o número 1000!? Qual é a potência de 3 que aparece na
decomposição de 1000! em fatores primos?
Solução. a)

Temos: 1000! = 1 × 2 × 3 × · · · × 998 × 990 × 1000.

1000! = α · 5200 (1 × 2 × 3 × · · · × 198 × 199 × 200) para algum α ∈ N


94

1000! = β · 5200 [540 (1 × 2 × 3 × · · · × 38 × 39 × 40)] para algum β ∈ N

1000! = γ · 5240 [58 (1 × 2 × 3 × · · · × 6 × 7 × 8)] = 5249 δ para algum γ, δ ∈ N

Por outro lado

1000! = σ · 2500 (1 × 2 × 3 × · · · × 498 × 499 × 500) para algum σ ∈ N

sendo a potência de 2 maior que 5000 segue que 1000! = 5249 2249 n = 10249 n para algum
n ∈ N.
Portanto, 1000! termina em 249 zeros.
Solução. b)
Temos: 1000! = 1 × 2 × 3 × · · · × 998 × 990 × 1000.

1000! = α · 3333 (1 × 2 × 3 × · · · × 331 × 332 × 333) para algum α ∈ N

1000! = β · 3333 [3111 (1 × 2 × 3 × · · · × 109 × 110 × 111)] para algum β ∈ N

1000! = δ · 3444 [337 (1 × 2 × 3 × · · · × 35 × 36 × 37)] para algum δ ∈ N

1000! = γ · 3481 [312 (1 × 2 × 3 × · · · × 10 × 11 × 12)] para algum γ ∈ N

1000! = θ · 3493 [34 (1 × 2 × 3 × 4)] = 3498 n para algum θ, n ∈ N

Exercício 13.3.2.
Mostre que existem infinitos valores de n ∈ N para os quais 8n2 + 5 é divisível por 77
Solução.

O número n ∈ N podemos escrever na forma n = 77α + β, α, β ∈ N, logo

8n2 + 5 = 8(77α + β)2 + 5 == 8(772 α2 + 154αβ) + 8β 2 + 5 = 77q + 8β 2 + 5

Para o número 8n2 +5 é divisível por 77 deve acontecer que 8β 2 +5 = 77 ⇒ β 2 = 9


como β ∈ N logo β = 3, isto é os números que satisfazem as condições do problema são
da forma n = 77α + 3, α ∈ N. São infinitos.

Exercício 13.3.3.
Mostre que, se a, b ∈ N e n ∈ N ∪ {0}, então (an , bn ) = (a, b)n e que [an , bn ] = [a, b]n .
Solução.

Sejam p1 , p2 , . . . , pr números primos, sabe-se que todo número a, b ∈ N podemos es-


pr , com αi , βi ∈ N∗ .
α β
crever na forma a = pα1 1 pα2 2 pα3 3 . . . pr−1
r−1 αr
pr e b = pβ1 1 pβ2 2 pβ3 3 . . . pr−1
r−1 βr

nα nβ
Então an = pnα1 nα2 nα3
1 p2 p3 . . . pr−1r−1 pnα
r
r
e bn = pnβ 1 nβ2 nβ3
1 p2 p3 . . . pr−1r−1 pnβ
r , com
r

αi , βi ∈ N∗
95

Pelo teorema 7.2, considerando δi = max{ αi , βi }, e σi = min{ αi , βi }, então


nδi = max{ nαi , nβi }, e nσi = min{ nαi , nβi }.

nσ nδ
(an , bn ) = pnσ 1 nσ2 nσ3
1 p2 p3 . . . pr−1r−1 pnσ
r
r
e [an , bn ] = pnδ 1 nδ2 nδ3 r−1 nδr
1 p2 p3 . . . pr−1 pr

logo
σ
(an , bn ) = [pσ1 1 pσ2 2 pσ3 3 . . . pr−1
r−1 σr n
pr ] = (a, b)n
δ
[an , bn ] = [pδ11 pδ22 pδ33 . . . pr−1
r−1 δr n
pr ] = [a, b]n

Exercício 13.3.4.
Quais dos números abaixo são primos?

a) 239 b) 241 c) 247 d) 253 e) 1789


Solução.

Exercício 13.3.5.
(ENC-98) Uma das afirmativas abaixo sobre números naturais é FALSA. Qual é ela?
Solução.

(A) Dado um número primo, existe sempre um número primo maior do que ele.

(B) Se dois números não primos são primos entre si, um deles é ímpar.

(C) Um número primo é sempre ímpar. FALSO


O número 2 é primo e é par.

(D) O produto de três números naturais consecutivos é múltiplo de 6.

(E) A soma de três números naturais consecutivos é múltiplo de três.

Exercício 13.3.6.
Mostre que a13 − a é divisível por 2, 3, 5, 7, 13 e 273, para todo a ∈ N.
Solução.
96

Observe a13 − a = a(a6 − 1)(a6 + 1) = a(a − 1)(a2 + a + 1)(a3 + 1)(a6 + 1)


Como 13|a13 − a, 7|a6 − 1, 2|a1 − 1, então a13 − a é divisível por 2, 7 e 13
Também, a13 − a = a[(a2 )3 − 1][(a2 )3 + 1] = a(a2 − 1)[(a2 )2 + a2 + 1][(a2 )3 + 1]
Como 3|a2 − 1, então a13 − a é divisível por 3.
Do fato (3, 13) = (7, 13) = 1, segue que a13 − a é divisível por 273 = 3 × 7 × 13.
Por último, como a13 − a = a[(a2 )3 − 1][(a2 )3 + 1] =

= a(a2 − 1)(a2 + 1)[(a2 )2 + a2 + 1][(a2 )2 − a2 + 1] = a(a4 − 1)[(a2 )2 + a2 + 1][(a2 )2 − a2 + 1]

Como 5|a4 − 1, então a13 − a é divisível por 5.


Portanto, a13 − a é divisível por 2, 3, 5, 7, 13 e 273, para todo a ∈ N

13.4 O Renascimento da Aritmética


Exercício 13.4.1. Teorema de Fermat (caso particular n = 4).
Não existem inteiros x, y, z que satisfazem a equação de Diofanto: z 4 + y 4 = x4 .

Demonstração.
O primeiro a fazer é reescrever a equação na forma (z 2 )2 + (y 2 )2 = (x2 )2
Note que { z 2 , y 2 , x2 } é uma terna Pitagórica (TP).
Se eles não forem coprimos devemos dividir toda a terna pelo seu máximo divisor
comum m.d.c. obtendo uma terna de números primos e obteremos uma primeira terna.
Pela fórmula de Euclides, sabemos que x2 = a2 + b2 , y 2 = a2 − b2 ou y 2 = 2ab.
Suponhamos que y 2 = a2 − b2 , então x2 y 2 = a4 − b4 = (xy)2 de onde (xy)2 + (b2 )2 =
(a2 )2 , logo achamos outra terna TP da forma { xy, b2 , a2 }.
Os números desta segunda terna são menores que a primeira terna.
Continuando assim chegaremos a uma descida infinita o qual é impossível.
Logo y 2 = a2 − b2 é absurdo !
Supondo que y = 2ab com a > b, considerando a par e b ímpar (sem perda de
generalidade) temos o seguinte: x2 = a2 + b2 , supondo a = 2cd, b = c2 − d2 temos
x = c2 + d2 com c > d coprimos, logo y 2 = 2ab = 2(2cd)(c2 − d2 ) dividindo por 4 segue
y2 y
que = cd(c2 − d2 ) = ( )2
4 2
Podemos observar que o produto de c, d e c2 − d2 é um quadrado perfeito, pelo que
cada um de eles é um quadrado perfeito, logo: c2 = e2 , d2 = f 2 e c2 − d2 = g 2 .
Substituindo na última equação obtemos (e2 )2 − (f 2 )2 = g 2 = e4 − f 4 , assim achamos
outra terna TP { g 2 , f 2 , e2 } menor do que a primeira terna { z 2 , y 2 , x2 }, criando assim
97

uma descida infinita o qual é impossível.


Portanto podemos concluir que a equação Diofantina z 4 + y 4 = x4 não tem soluções
inteiras.
98
MA-14 - Aula 7

Semana 12/10 a 19/10

Unidade 15

15.2 Primos de Fermat e de Mersenne


Exercício 15.2.1.
Se p e q são números primos p ≥ q ≥ 5, então 24|p2 − q 2
Solução.

Como 3 é primo e p ≥ q ≥ 5 então pelo Corolário do PTF temos que 3|p2 −1 e 3|q 2 −1,
logo 3|p2 − q 2 = (p2 − 1) + (q 2 − 1).
Por outro lado, p = 2m + 1 e q = 2n + 1 então

p2 − q 2 = 4m2 + 4m − 4n2 − 4n = 4(m2 − n2 ) + 4(m − n) = 4(m − n)(m + n − 1)

Se m e n tiverem a mesma paridade segue que m − n é par e, se tiverem paridades


diferentes, segue que m + n − 1 é par, assim temos que p2 − q 2 = 8α, α ∈ N.
Como (3, 8) = 1, 3|p2 − q 2 e 8|p2 − q 2 então 24|p2 − q 2 .

Exercício 15.2.2.
Todo primo da forma 3n + 1 é também da forma 6m + 1.
Solução.

Seja p-primo tal que p = 3n + 1 para algum n ∈ N.


Se n-ímpar então p não é primo, logo necessáriamente n-par. Suponhamos n =
2m, m ∈ N, então p = 3n + 1 = 3(2m) + 1 = 6m + 1.

Exercício 15.2.3.
Mostre que o único número primo da forma n3 − 1 é 7.
Solução.

Seja p-primo tal que p = n3 − 1 para algum n ∈ N, então p = (n − 1)(n2 + n + 1).

99
100

Se n-ímpar então p-par, isto é absurdo, para o caso n-par como p ≥ 2, então n =
2, 4, · · · , 2k, k ∈ N.
Se n = 2 ⇒ p = 3, para o caso n ≥ 4 ⇒ (n − 1) ≥ 3 e (n2 + n + 1) ≥ 21
logo p = (n − 1)(n2 + n + 1) é composto, não é primo.
Portanto, p = 23 − 1 = 7.

Exercício 15.2.4.
O único número primo n tal que 3n + 1 é um quadrado é 5.
Solução.

Suponhamos n seja primo tal que 3n + 1 = m2 para algum m ∈ N, então 3n = m2 − 1


de onde 3n = (m − 1)(m + 1), m ≥ 2 sendo m-par, pois n é ímpar.
Consideremos m = 2k, ∀ k ∈ N então 3n = (2k − 1)(2k + 1) ⇒ 3n + 1 = 4k 2 .
Quando k = 1, ⇒ 3n + 1 = 4 × 12 ⇒ n = 1 não é primo.
Quando k = 2, ⇒ 3n + 1 = 4 × 2 = 4
2 2
⇒ n = 5 é primo.
Quando k = 3, ⇒ 3n + 1 = 4 × 32 = 62 ⇒ n∈
/ N.
Quando k = 3β, β ∈ N temos 3n + 1 = 4 × (3β) = (6β)2
2
⇒ n∈
/ N.
Suponhamos 3 - k então k = m(3) + 1 ou k = m(3) + 2 de onde k 2 = 3α + 1, α ∈ N,
logo
3n + 1 = 4k 2 ⇒ 3n + 1 = 4(3α + 1)2 = 4(9α2 + 6α + 1) ⇒

3n + 1 = 3(12α2 + 8α + 1) + 1 ⇒ 3n = 3(2α + 1)(6α + 1)

logo n = (2α + 1)(6α + 1) não é primo.


Portanto, o único número primo n tal que 3n + 1 é um quadrado é 5.

Exercício 15.2.5.
Seja k ∈ N, k > 2. Mostre que

a) Se k divide a1 − 1; a2 − 1; . . . ; ar − 1, então k divide a1 a2 · · · ar − 1.

b) Se n > 0, então existe um primo p tal que k - (p − 1) e p|(nk − 1).

c) Existem infinitos primos p tais que k - (p − 1).

Solução. a)

Indução sobre r ∈ N.
Se r = 2 temos que k divide a1 − 1 e a2 − 1 então a1 = αk + 1 e a2 = βk + 1, α, β ∈
N ⇒ a1 a2 = (αk + 1)(βk + 1) = γk + 1 ⇒ a1 a2 − 1 = γk ⇒ k|a1 a2 − 1.
Suponhamos para r ∈ N que, se k divide a1 − 1; a2 − 1; . . . ; ar − 1, então k divide
a1 a2 · · · ar − 1.
101

Seja r + 1 ∈ N e suponhamos que k divide a1 − 1; a2 − 1; . . . ; ar − 1; ar+1 − 1, logo pela


hipótese indutiva k divide a1 a2 · · · ar − 1 e k|ar+1 − 1 então

a1 a2 · · · ar − 1 = δk, ar+1 − 1 = θk; δ, θ ∈ N ⇒

a1 a2 · · · ar ar+1 = (δk + 1)(θk + 1) = εk + 1, ε∈N

logo k|a1 a2 · · · ar ar+1 − 1, ∀ k ∈ N.


Portanto, se k divide a1 − 1; a2 − 1; . . . ; ar − 1, então k divide a1 a2 · · · ar − 1.
Solução. b)

Exercício 15.2.6.
Mostre que existe uma correspondência biunívoca entre pares de primos gêmeos e nú-
meros n tais que n2 − 1 possui quatro divisores.
Solução.

Dois números primos são números primos gêmeos se a diferença entre eles for igual a
dois. Os primeiros pares de números primos gêmeos são 3 e 5, 5 e 7, 11 e 13, 17 e 19,
29 e 31, 41 e 43, 59 e 61, 71 e 73, 101 e 103, 107 e 109.
Os maiores números primos gêmeos conhecidos são 2.003.663.613 × 2195.000 ± 1, desco-
bertos em janeiro de 2007. Existem um mil números primos gêmeos abaixo de 100.000 e
oito mil abaixo de 1.000.000.
Como n2 − 1 tem quatro divisores, então

n2 − 1 = (n − 1)(n + 1) = p11 × p12 , onde p1 , p2 primos com p1 ̸= p2

se n − 1 = p1 e n + 1 = p2 então p2 = p1 + 2. Sejam

A = {(pi , pj )/. pi , pj primos gêmeos}eB = {n ∈ N/. n2 +1 com quatro divisores}

definimos quad f : A −→ B onde f (pi , pj ) = pi + 1 sendo pi + 1 = n.


A função f é bem definida, pois a cada elemento (pi , pj ) ∈ A corresponde o
elemento ni = pi + 1 ∈ B
A função f é injetora, se (pi , pj ), (pk , pl ) ∈ A, tais que (pi , pj ) ̸= (pk , pl ), logo
pi ̸= pk e pj ̸= pl por ser cada par de primos gêmeos, temos

(pi , pj ) ̸= (pk , pl ) ⇒ pi ̸= pk ⇒ pi + 1 ̸= pk + 1 ⇒ f (pi , pj ) ̸= f (pk , pl )

A função f é sobrejetora, se n ∈ B então é tal que n2 − 1 tem 4 divisores, logo é


da forma n2 − 1 = pi · pj ⇒ pj = pi + 2 logo pi e pj são primos gêmeos e (pi , pj ) ∈ A.
102

Portanto, existe uma correspondência biunívoca entre pares de primos gêmeos e nú-
meros n tais que n2 − 1 possui quatro divisores.

Exercício 15.2.7.
Mostre que o produto dos divisores de um inteiro positivo n é ns/2 , onde s é o número
de divisores de n.
Demonstração. Primeira

Suponhamos n = pα1 1 pα2 2 pα3 3 · · · pαr r


O produto de todos os divisores p1 denotado PD (p1 ) é

PD (p1 ) = 1 · p1 · p21 · p31 · · · pα1 1 −1 · pα1 1 ou PD (p1 ) = pα1 1 · p1α1 −1 · · · p31 · p21 · p1 · 1

assim temos

PD (p1 )PD (p1 ) = (1 · pα1 1 )(p1 pα1 1 −1 )(p21 pα1 1 −2 )(p31 p1α1 −3 ) · · · (p1α1 −2 p21 )(p1α1 −1 p1 )(pα1 1 · 1)

PD (p1 )PD (p1 ) = (pα1 1 )α1 +1 ⇒ [PD (p1 )]2 = (pα1 1 )α1 +1

De modo análogo mostra-se para qualquer um dos divisores primos de n que

α
[PD (pj )]2 = (pj j )αj +1 ; j = 1, 2, 3, . . . , r

Por outro lado denotado por D(pα1 1 ) o conjunto dos divisores de pα1 1 temos

D(pα1 1 ) = {1, p1 , p21 , p31 , · · · , pα1 1 −1 , pα1 1 } e D(pα2 2 ) = {1, p2 , p22 , p32 , · · · , p2α2 −1 , pα2 2 }

Multiplicando-se cada divisor de pα1 1 por cada divisor de pα2 2 temos que

PD (pα1 1 pα2 2 ) = (1 · p1 · p21 · p31 · · · pα1 1 −1 pα1 1 )(1 · p2 · p22 · p32 · · · p2α2 −1 pα2 2 )
| {z } | {z }
α2 + 1 vezes α1 + 1 vezes
pois cada elemento de D(pα1 1 ) e D(pα2 2 ) repetirá respectivamente α2 + 1 e α1 + 1
vezes. Isto é equivalente a

PD (pα1 1 pα2 2 ) = [PD (p1 )]α2 +1 [PD (p2 )]α1 +1 ⇒

[PD (pα1 1 pα2 2 )]2 = [(pα1 1 )(α1 +1) ]α2 +1 [(pα2 2 )(α2 +1) ]α1 +1 = (pα1 1 pα2 2 )(α1 +1)(α2 +1)

Prosseguindo com esse raciocínio encontraremos que

[PD (pα1 1 pα2 2 · · · pαr r )]2 = (pα1 1 pα2 2 · · · pαr r )(α1 +1)(α2 +1)···(α2 +1)
103

Seja s o número de divisores de n = pα1 1 pα2 2 pα3 3 · · · pαr r , então sabemos que s = (α1 +
1)(α2 + 1) · · · (α2 + 1), logo

[PD (pα1 1 pα2 2 · · · pαr r )]2 = (pα1 1 pα2 2 · · · pαr r )s = (n)s ⇒ PD (n) = ns/2

Portanto, o produto dos divisores de um inteiro positivo n é ns/2 , onde s é o número


de divisores de n. 

Demonstração. Segunda
n n
Sabemos que se d é um divisor de n, então também é divisor de n, pois d · = n .
d d
Assim se d1 , d2 , d3 , . . . , ds−1 , ds , são os divisores positivos de n, podemos escrever o
seu produto P (n) de duas maneiras:
• P (n) = d1 · d2 · d3 . . . ds−1 · ds
n n n n n
• P (n) = · · ··· ·
d1 d2 d3 ds−1 ds
Multiplicando estas duas igualdades
( )( n n n n n)
[P (n)]2 = d1 · d2 · d3 . . . ds−1 · ds · · ··· · = ns = (ns/2 )2
d1 d2 d3 ds−1 ds

Portanto, o produto dos divisores de um inteiro positivo n é ns/2 , onde s é o número


de divisores de n.

Exercício 15.2.8.
Prove que, se r é o número de fatores primos distintos de n ∈ N∗ , o número de modos
em que n pode ser fatorado como produto de dois números relativamente primos é 2r−1 .
Solução. Primeira

Seja n = pα1 1 · pα2 2 · pα3 3 · · · pαr r .


Indução sobre r.

Suponha r = 3 então n = pα1 1 · pα2 2 · pα3 3 .

Temos que os números pα1 1 , pα2 2 e pα3 3 são relativamente primos dois a dois. Isto é:

mdc{ pn1 1 ; pα2 2 pα3 3 } = 1 mdc{ pα2 2 ; pα1 1 pα3 3 } = 1

mdc{ pα3 3 ; pα1 1 pα2 2 } = 1, mdc{ 1; (pα1 1 pα2 2 pα3 3 ) } = 1

Ao todo são 23−1 modos diferentes que podemos escrever n como o produto de dois
fatores relativamente primos.
Suponhamos para r = h, o número de modos diferentes que podemos escrever n como
o produto de dois fatores relativamente primos seja 2h−1 .
104

α α
Para r = h + 1 temos que n = pα1 1 · pα2 2 · pα3 3 · · · pαh h · ph+1
h+1
= P · ph+1
h+1
.
α
Logo, podemos escrever n de dois modos diferentes a saber: n = P · (ph+1
h+1
) ou
α α α
n = 1 · (P · ph+1
h+1
) onde mdc{ P ; ph+1 h+1
} = 1, mdc{ 1; (P · ph+1 h+1
} = 1, sendo que P é
h−1
um número com 2 modos de escrever como o produto de dois divisores.
Logo, n podemos escrever de 2h−1 · 2 = 2(h+1)−1 modos diferentes como o produto de
dois divisores.
Portanto, o número de modos diferentes em que podemos decompor n em dois fatores
que são relativamente primos é 2r−1 .
Solução. Segunda
Seja n = pα1 1 ·pα2 2 ·pα3 3 · · · pαr r , podemos supor n = a·b onde a = pη11 e b = pη22 ·pη33 · · · pηr r .
Também podemos supor n = a · b onde a = pη11 pη22 e b = pη33 · pη44 · · · pηr r ou também
podemos supor n = a · b onde a = pη11 pη22 pη33 e b = pη44 · pη55 · · · pηr r .
Podemos observar que temos 2r pares distintos de números (a, b) tais que n = ab
sendo (a, b) = 1.
Na contagem, o par (pη11 , pη22 · pη33 · · · pηr r ) = (pη22 · pη33 · · · pηr r , pη11 ) representa a mesma
fatoração para n, e; como contamos duas vezes pares similares temos que o número de
modos em que n pode ser fatorado como produto de dois números relativamente primos
2r
é = 2r−1 .
2
Exercício 15.2.9.
Seja n > 2. Mostre que entre n e n! existe pelo menos um número primo.
Solução.

Joseph Louis F. Bertrand (1822 − 1900) foi um matemático, historiador de ciências e


acadêmico francês.
Em 1845 lançou a conjetura que sempre existe ao menos um número primo entre n e
2n−2 para todo n maior do que 3. P. Tchebychev demonstrou essa conjetura, o postulado
de Bertrand, em 1850.
Pela conjetura de J. Bertrand já demonstrada temos que existe um primo p tal que
3 < n < p < 2(n − 1)
Como 2(n − 1) < n! segue que existe um primo p tal que 3 < n < p < n!
Para o caso n = 3 ⇒ 3 < 5 < 3!
Portanto, entre n e n! existe pelo menos um número primo.

Exercício 15.2.10.
Mostre que se p, p + 2 e p + 4 são primos, então p = 3.
Solução.

Por hipótese os números p, p+2 e p+4 são primos, suponhamos que p = 2k +1, k≥
1.
105

Assim temos então que os números p, p + 2 e p + 4 são primos; isto é os números

2k + 1, 2k + 3, 2k + 5 ⇔ 2k + 1, 2(k + 1) + 1, 2(k + 2) + 1; k≥1 (15.14)

são primos, além disso são três números ímpares consecutivos.


Sabemos que todo número natural k podemos escrever como algum elemento do con-
junto A = {3n, 3n + 1, 3n + 2; ∀ n ∈ N }.
Para n ≥ 1 segue:
Em (15.14) se k = 3n ⇒ p = 6n + 1, p + 2 = 3(2n + 1), p + 4 = 6n + 5.
Em (15.14) se k = 3n+1 ⇒ p = 3(2n+1), p+2 = 3(2n+1)+2, p+4 = 3(2n+2)+1.
Em (15.14) se k = 3n+2 ⇒ p = 3(2n+1)+2, p+2 = 3(2n+2)+1, p+4 = 3(2n+3).
Com qualquer hipótese para n ≥ 1, um dos números p, p + 2 e p + 4 é composto e
múltiplo de três.
Quando n = 0 temos p = 3, p + 2 = 5 e p + 4 = 7
Portanto, p = 3.

Exercício 15.2.11.

a) Sejam m, n ∈ N de paridade distinta. Mostre que 3|am − an .

b) Seja p > 3 um número primo. Mostre que ap − a e ap b − bp a são divisíveis por 6p,
para todos a ∈ N, com a > b.

Solução. a)

Suponhamos m-ímpar e n-par, então m = αn + 1 para algum α ∈ N.

am − an = aαn+1 − an = a(aαn ) − an = a(an )α − an ⇒

considerando n = 2k, k∈N

am − an = a(a2k )α − a2k = a[(akα )2 − 1] − [(ak )2 − 1] + (a − 1) =

Se 3|a nada a mostrar.


Suponhamos 3 - a, logo 3 - akα e 3 - ak .
Pelo corolário do PTF segue 3|(akα )2 − 1 e 3|(ak )2 − 1. Assim temos

am − an = m(3) + (a − 1) =?

Contraexenplo
Se a = 5, m = 3 e n = 2 temos 3 - 53 − 52 = 100 acredito que falta informação a
106

respeito de a
Solução. b)
Suponhamos p > 3 número primo, pelo PTF temos que p|ap − a.
Por outro lado, ap e a têm a mesma paridade, de onde ap − a é par, assim 2|ap − a.
seja p = 2k + 1, k ∈ N

ap − a = a2k+1 − a = a(a2k − 1) = a[(ak )2 − 1]

Se 3|a ⇒ 3|ap − a. Se 3 - a ⇒ 3 - ak , k ∈ N, assim, pelo Corolário do PTF


temos 3|(ak )3−1 − 1 ⇒ 3|ap − a
Logo, p|ap − a, 2|ap − a, 3|ap − a e como (2, 3, p) = 1 segue que 6p|ap − a. 

A demonstrar que ap b − bp a é divisível por 6p, para todos a ∈ N, com a > b.


Come feito, sabemos que 6p|ap − a e 6p|bp − b logo 6p|ap b − ab e 6p|bp a − ba de onde
6p|(ap b − ab) − (bp a − ba) ⇒ 6p|ap b − bp a
Portanto, ap b − bp a é divisível por 6p, para todos a ∈ N, com a > b.
Exercício 15.2.12.
Sejam a, b ∈ N, com (a, b) = 1, e n ∈ N tal que n + 2 = p é um número primo.
Mostre que o mdc de a + b e a2 − nab + b2 deve ser 1 ou p.
Solução.
Suponhamos d = (a + b, a2 − nab + b2 ) = (a + b, (a + b)2 − ab(2 + n)), aplicando o
Lema de Euclides temos d = (a + b, −abp)
Assim temos que d|a + b e d|abp.
Sendo (a, b) = 1 então existem β, γ ∈ N tais que βa + γb = 1.
Como d|a + b, então existe θ ∈ N tal que a + b = θd. Assim temos

1 = βa + γb = β(a + b) + b(γ − β) = βθd + b(γ − β) ⇒ (d, b) = 1

De modo análogo

1 = βa + γb = a(β − γ) + γ(a + b) = a(β − γ) + βθd ⇒ (d, a) = 1

Logo temos que d - a e d - b e como d|abp segue que d|p, isto significa que d = 1 ou
d = p.
Portanto, o mdc de a + b e a2 − nab + b2 deve ser 1 ou p.
Exercício 15.2.13.
Seja p um número primo ímpar. Mostre que pode-se escrever p = y 2 −x2 , com x, y ∈ N,
de modo único.
Solução.
107

Temos que p ̸= 2, logo p ≥ 3. Seja p = 2n + 1 para algum n ∈ N, então p = 1 × p.


Suponhamos existam números naturais x, y tais que y − x = 1 e y + x = 2n + 1,
disto segue que a única solução do sistema é y = n + 1 e x = n para algum n ∈ N.
Então podemos escrever p = 1 × (2n + 1) = (y − x)(y + x), isto é p = y 2 − x2 .

Exercício 15.2.14.
Sejam a; b; n; m ∈ N∗ e suponha que an + bm seja um número primo. Mostre que
(n, m) = 1, ou (n, m) = 2r , para algum r ∈ N.
Solução.

Suponhamos an + bm seja um número primo é primo. O caso a = b = 1 é imediato e


não será considerado, logo an + bm > 2 sendo a e b de paridades distintas.
Suponhamos (n, m) = d então m = αd e m = βd, α, β ∈ N. Assim, temos

an + bm = aβd + bαd = (aβ )d + (bα )d ⇒

• Se d-ímpar, é da forma d = 2k + 1, k ∈ N e temos.

an + bm = (aβ )2k+1 + (bα )2k+1 = (aβ + bα )[(aβ )2k − (aβ )2k−1 bα + · · · + (bα )2k ]

observamos que an + bm é número composto desde que k ≥ 1.


Sendo por hipótese an +bm primo então k = 0, logo d = 1 consequentemente (n, m) = 1.
• Se d-par, é da forma d = 2α, α ∈ N e temos.

an + bm =


108

Exercício 15.2.15.
Sejam n; a ∈ N.

a) Mostre que existe m ∈ N tal que (a + 1)n = ma + 1.

b) Mostre que, se a > 0, então existe m ∈ N tal que (a − 1)2n+1 = ma − 1.

c) Mostre que, se a > 1, então existe m ∈ N tal que (a − 1)2n = ma + 1.

Sugestão: Por indução sobre n.


Solução.

a) Se n = 1 tem-se (a + 1)1 = 1 · a + 1, logo existe 1 ∈ N.


Suponhamos para n ≤ n tem-se que existe m ∈ N, tal que (a + 1)h = am + 1.
Para n = h + 1, aplicando a hipótese auxiliar segue que

(a+1)h+1 = (a+1)h (a+1) = (ma+1)(a+1) = ma(a+1)+(a+1) = (ma+1+m)a+1 = m1 a+1

onde (ma + 1 + m) = m1 ∈ N.
Portanto, existe m ∈ N tal que (a + 1)n = ma + 1.

b) Mostre que, se a > 0, então existe m ∈ N tal que (a − 1)2n+1 = ma − 1.

c) Mostre que, se a > 1, então existe m ∈ N tal que (a − 1)2n = ma + 1.

x
MA-14 - Aula 09

Semana 21/10 a 27/10

Unidade 17
Primos de Fermat e de Mersenne

17.1 Primos de Fermat


Os números de Fermat são os números da forma

n
Fn = 2 2 + 1

Fermat achava que esses números eram todos primos.


De fato, F1 = 5, F2 = 17, F3 = 257, F4 = 65537 são primos.
Em 1732, Euler mostrou que

5
F5 = 22 + 1 = 4.294.967.297 = 641 × 6700417,

portanto, composto, desfazendo assim esta crença de Fermat.


Os números de Fermat primos são chamados de primos de Fermat.
Até hoje, não se sabe se existem outros primos de Fermat além dos quatro primeiros.
Provamos no Corolário da Proposição 6 da Unidade 10 que

(F n; F m) = 1; se n ̸= m

17.2 Primos de Mersenne


Os números de Mersenne são os números da forma

Mp = 2p − 1;

109
110

onde p é um número primo.


No intervalo 2 ≤ p ≤ 5000 os números de Mersenne que são primos, chamados de
primos de Mersenne, correspondem aos seguintes valores de p:

2, 3, 5, 7, 13, 19, 31, 61, 89, 107, 127, 521, 607, 1279, 2203, 2281, 3217, 4253 e 4423

.
Até o presente momento, o maior primo de Mersenne conhecido é M43112609 , descoberto
em agosto de 2008 e que possui no sistema decimal 12978189 dígitos.

17.3 Teorema da Dirichlet


Enunciaremos a seguir, sem demonstração, um resultado profundo devido a Lejeune
Dirichlet:

Teorema 17.7. Em uma PA de números naturais, com primeiro termo e razão primos
entre si, existem infinitos números primos.

A demonstração deste resultado é bem difícil e pertence à teoria analítica dos números.
Nos limitamos no texto a demonstrar alguns casos particulares de teorema.
O primeiro caso particular é o seguinte:

Proposição 17.1. Na progressão aritmética

3; 7; 11; 15 ; . . . , 4n + 3; . . .

existem infinitos números primos.

O que a proposição nos diz é que existem infinitos primos da forma 4n + 3.


A prova de que existem infinitos primos da forma 4n + 1 é um pouco mais sutil.

17.4 Problemas
Exercício 17.4.1.
Mostre que todo divisor de um número de Fermat Fn é da forma 4m + 1.
Sugestão: Use o Lema 5.
Solução.
n
Sabe-se que Fn = 22 + 1, este número é ímpar, logo admite como divisores somente
números ímpares. Logo, qualquer divisor de Fn é ímpar.
n (n−1) ( (n−1) )2 (n−1)
Como Fn = 22 + 1 = 22·2 + 1 = 22 + 1, isto é Fn = x2 + 1 onde x = 22 .
111

O Lema 5 diz: “Seja x ∈ N, com x ≥ 2. Todo divisor ímpar de x2 + 1 é da forma


4n + 1”.
Em virtude deste Lema temos que, todo divisor de um número de Fermat Fn é da
forma 4m + 1.

Exercício 17.4.2.
Se p e q são dois números primos distintos, mostre que (Mp ; Mq ) = 1
Solução.

Consideremos os números de Mersenne Mp = 2p − 1 e Mq = 2q − 1, sabemos que


(an − 1, am − 1) = a(m,n) − 1.
Como (p, q) = 1 ⇒ (Mp ; Mq ) = (2p − 1, 2q − 1) = 21 − 1 = 1.

Exercício 17.4.3.
Sejam dados n; m ∈ N, a) Mostre que, se m < n, então Fm |Fn − 2. b) Dê uma
outra prova para: (Fn , Fm ) = 1, se n ̸= m.
Solução.

a) Sejam m, n ∈ N tais que m < n, sabemos que Fn = 22 + 1 e Fm = 22 + 1. Logo,


n m

Fn − 2 = 22 + 1 − 2 = 22 − 1.
n n

Como m < n ⇒ n = m + α assim

n m+α m α
Fn − 2 = 2 2 − 1 = 2 2 − 1 = (22 )2 − 1

Sabemos que 2α é par também, se a, b, k ∈ N cumpre a + b|a2n − b2k , logo temos

m α α m
Fn − 2 = (22 )2 − 12 = β(22 + 1) = βFm

Portanto, se m < n, então Fm |Fn − 2.

b) Sejam m, n ∈ N tais que m ̸= n, sabemos que Fn = 22 + 1 e Fm = 22 + 1, logo


n m

Fn ̸= Fm .
n m
Por outro lado, como (Fn , Fm ) = (22 + 1, 22 + 1), pelo Corolário 3 da unidade
10 {
1 se a par
(an + 1, am + 1) =
2 se a ímpar

Como a = 2 é par, segue que (Fn , Fm ) = 1

Exercício 17.4.4.
Mostre que existem infinitos números primos da forma 6n + 5.
Demonstração.
112

Suponhamos que exista uma quantidade finita de números primos da forma 6n + 5,


digamos

p1 = 6n1 + 5, p2 = 6n2 + 5, ..., pm−1 = 6nm−1 + 5, pm = 6nm + 5

onde 5 < p1 < p2 < . . . < pm−1 < pm .


Com estes números podemos construir o número

P = 6(p1 p2 · · · pm−1 pm ) + 5 = 6n + 5

Observe que nenhum dos números 5, p1 , p2 , . . . , pm−1 , pm é um divisor de P . Sendo


P primo é da forma P = 6nm+1 + 5.
Logo, supor que existam m primos da forma 6n + 5 é absurdo pois achamos mais um
dessa forma.
Portanto, existem infinitos números primos da forma 6n + 5.

Exercício 17.4.5.
Mostre que existem infinitos números primos da forma 3n + 2.
Solução.

Exercício 17.4.6.
Seja pn o n-ésimo número primo. Mostre que pn ≤ 22
n−2
+ 1.
Sugestão: Utilize os seguintes fatos: (Fi ; Fj ) = 1, se i ̸= j, 2 - Fi e F5 é composto.
Demonstração.

Para n = 1, 2, 3 é imediato.

2−2 3−2
2 = p1 , 3 = p2 ≤ 22 + 1 = 3, 5 = p3 ≤ 22 +1=5

n−2
Afirmo: Se pn é o n-ésimo número primo então pn < 22 sempre que n > 4.
24−2 22
Com efeito, se n = 4 temos p4 = 7 < 2 =2 = 16
h−2
Sponhamos que seja verdade a desigualdade para n = h; isto é ph < 22 é verdadeira
Multiplicando todas as desigualdades até h temos

2 3 4 5 h−3 h−2
p4 p5 p6 · · · ph < 22 · 22 · 22 · 22 · · · · 22 · 22
113

22 (1 − 2h−1 )
mas, a PG 22 + 23 + 24 + · · · 2h−3 + 2h−2 = = 2h−1 − 4, assim temos
1−2
h−1 −4 h−1
p4 p5 p6 p7 p8 · · · ph < 22 ⇒ 24 · p4 p5 p6 p7 p8 · · · ph < 22

Em particular
h−1
22 · p4 p5 p6 · · · ph − 1 < 22

Observe que 22 · p4 p5 p6 · · · ph − 1 é um número composto. Seja α ∈ N um número


primo tal que α|22 · p4 p5 p6 · · · ph − 1, logo α ̸= 2, α ̸= pi , i = 4, 5, . . . h pois α ̸= 1
Assim, temos α ≥ ph+1 de onde

h−1 (h+1)−2
ph+1 < 22 · p4 p5 p6 · · · ph − 1 < 22 ⇒ ph+1 < 22

Portanto, pn ≤ 22 n ≥ 4, o caso n = 2, 3 é imediato.


n−2
+ 1,

Exercício 17.4.7.
Considere a sequência de Fibonacci (un ). Mostre que, se n é ímpar, então os divisores
ímpares de un são da forma 4k + 1.
Sugestão: Mostre que a sequência de Fibonacci satisfaz a

u2n = un−1 un+1 + (−1)n

Com isto, mostre que se n é ímpar, então u2n − un−1 un+1 = u2n−1 + 1. Conclua que
todo divisor de un é divisor de u2n−1 + 1 e utilize o Lema 5.
Solução.

x
114

Unidade 18
Números Perfeitos

18.1 Números Perfeitos


Os números como 6 e 28, com a propriedade de serem iguais à metade da soma de seus
divisores, tiveram o poder de fascinar os gregos antigos, que os chamaram de números
perfeitos.
Até a Idade Média, conheciam-se apenas os seguintes números perfeitos:

6, 28, 496, 8128e33550336

Atualmente, conhecem-se mais alguns números perfeitos. Um fato curioso é que todos
os números perfeitos conhecidos são pares. Não se sabe nada sobre a existência ou não de
números perfeitos ímpares.
Denotemos por S(n) a soma de todos os divisores de um número natural n.
Tem-se que S(1) = 1 e se pα1 1 · · · pαr r é a decomposição em fatores primos de n > 1,
então
pα1 +1 − 1 pαr +1 − 1
S(n) = 1 ··· 1
p1 − 1 pr − 1
Exemplo 18.1.

32 − 1
• S(3) = =4
3−1
22 − 1 32 − 1
• S(6) = S(2 × 3) = × = 12
2−1 3−1
22 − 1 33 − 1
• S(18) = S(2 × 3 ) =
2
× = 39
2−1 3−1
23 − 1 72 − 1
• S(28) = S(22 × 7) = × = 56
2−1 7−1
Podemos então dizer que um número n é um número perfeito se S(n) = 2n.
A função n 7→ S(n) possui as seguintes propriedades:

1. A função S(n) é multiplicativa; isto é, se (n;m) = 1, então S(n × m) = S(n) × S(m).

2. Se n > 1, tem-se que n é primo se, e somente se, S(n) = n + 1.

O teorema que enunciaremos a seguir caracterizará os números perfeitos pares, relacionando-


os com os números de Mersenne definidos na unidade anterior
115

Teorema 18.8. (Euclides-Euler)


Um número natural n é um número perfeito par se, e somente se, n = 2p−1 (2p − 1),
onde 2p − 1 é um primo de Mersenne.

A primeira parte da demonstração do teorema acima, sem dúvida a mais fácil, já se


encontra nos Elementos de Euclides (Proposição 36, livro IX). A recíproca data do século
18 e é devida a Euler.
O fato do número 2p −1, no enunciado do teorema, ser um número primo de Mersenne,
implica que p é primo. Note, ainda, que o teorema reduz a existência ou não de um número
infinito de números perfeitos pares ao problema análogo para primos de Mersenne.

18.2 Problemas
Exercício 18.2.1.
Mostre que a soma dos inversos dos divisores de um número perfeito par é sempre
igual a 2.
Demonstração.

Um número perfeito par é da forma n = 2p−1 (2p − 1), onde 2p − 1 é um primo de


Mersenne. Os divisores de n são:

• 1, 2, 22 , 23 , 24 , . . . , 2p−1 , 2p

• 2p − 1

• (2p − 1), 2(2p − 1), 22 (2p − 1), 23 (2p − 1), 24 (2p − 1), . . . , 2p−1 (2p − 1), 2p (2p − 1)

A soma dos inversos dos divisores parcialmente

1 1 1 1 1 1 1 − 21p 2p − 1
• + + 2 + 3 + 4 + . . . + p−1 = =
1 2 2 2 2 2 1 − 12 2p−1
[ ] [ p ]
1 1 1 1 1 1 1 1 2 −1
• + + + + + . . . + p−1 = p
(2p − 1) 1 2 22 23 24 2 (2 − 1) 2p−1

A soma S destas três parcelas é


[ p ] [ p ]
2p − 1 1 2 −1 2p 2 −1 2p+1 − 2
S = p−1 + p = = =2
2 (2 − 1) 2p−1 (2p − 1) 2p−1 (2p − 1)

Portanto, a soma dos inversos dos divisores de um número perfeito par é sempre igual
a 2.
116

Exercício 18.2.2.
Seja an = 22n (22n+1 − 1). Mostre por indução sobre n que a) a2n+1 = 256a2n−1 +
60(16n ); e b) a2n+2 = 256a2n + 240(16n ):
Demonstração.

a) Para a2n+1 = 256a2n−1 + 60(16n ).


Se n = 1 temos a1 = 22 (23 − 1) = 28
Também, se n = 1 segue a3 = 256a1 + 60(161 ) = 256 × 28 + 60(16) = 8128 = 26 (27 − 1)
o qual é verdadeiro.
Suponhamos para n = h ∈ N que a2h+1 = 256a2h−1 + 60(16h ) seja verdade, isto é
28 a2h−1 = a2h+1 − 60(16h ) é verdade. Observe que

a2h−1 = 22(2h−1) (22(2h−1)+1 − 1) = 24h−2 (24h−1 − 1)

Para n = h + 1 segue

a2(h+1)+1 = a2h+3 = 22(2h+3) (22(2h+3)+1 − 1) = 24h+6 (24h+7 − 1)

a2(h+1)+1 = 28 × 24h−2 (28 × 24h−1 − 1) = 28 × 24h−2 [28 (24h−1 − 1) + 28 − 1]

a2(h+1)+1 = 216 × 24h−2 (24h−1 − 1) + 26 × 24h [28 − 1] = 216 × a2h−1 + 26 × 24h [28 − 1]

Pela hipótese de indução

a2(h+1)+1 = 28 [a2h+1 − 60(16h )] + 26 × 24h [28 − 1] ⇒

a2(h+1)+1 = 28 a2h+1 − 28 × 60(16h ) + 26 × 24h [28 − 1] ⇒

a2(h+1)+1 = 28 a2h+1 − 24 × 60(16h+1 ) + 22 (16h+1 )[28 − 1] ⇒

a2(h+1)+1 = 28 a2h+1 + (16h+1 )[22 [28 − 1] − 24 × 60] ⇒

a2(h+1)+1 = 256a2(h+1)−1 + 60(16h+1 )

Portanto, se an = 22n (22n+1 − 1) então a2n+1 = 256a2n−1 + 60(16n ) para todo n ∈ N.


a) Outra solução para a2n+1 = 256a2n−1 + 60(16n ).
Em an = 22n (22n+1 − 1)
Fazendo n = 2h − 1 segue a2h−1 = 24h−2 (24h−1 − 1)
Fazendo n = 2h + 1 segue a2h+1 = 24h+2 (24h+3 − 1) ⇒

a2h+1 = 24 × 24h−2 (24 × 24h−1 − 1) = 24 × 24h−2 [24 × (24h−1 − 1) + (24 − 1)]

a2h+1 = 28 × 24h−2 (24h−1 − 1) + 24 × 24h−2 (24 − 1)


117

a2h+1 = 28 a2h−1 + 22 × 24h (24 − 1) = 256a2h−1 + 60 × 16h

Portanto, se an = 22n (22n+1 − 1) então a2n+1 = 256a2n−1 + 60(16n ) para todo n ∈ N.


b) Para a2n+2 = 256a2n + 240(16n ).
Em an = 22n (22n+1 − 1) se n = 2 temos a2 = 24 (25 − 1) = 496
Por outro lado, em a2n+2 = 256a2n + 240(16n ) se n = 1 segue

a4 = 256a2 + 240(161 ) = 256 × 496 + 240(16) = 130816 = 28 (29 − 1)

o qual é verdadeiro.
Suponhamos para n = h ∈ N que a2h+2 = 256a2h + 240(16h ) seja verdade, isto é
28 a2h = a2h+2 − 240(16h ) é verdade. Observe que

a2(h+1) = 22(2(h+1)) (22(2(h+1))+1 − 1) = 24(h+1) (24(h+1)+1 − 1)

Para n = h + 1 segue a2(h+1)+2 = a2h+4 = 22(2h+4) (22(2h+4)+1 − 1) ⇒

a2(h+1)+2 = 24h+8 (24h+9 − 1) = 24 × 24(h+1) (24 × 24(h+1)+1 − 1)

a2(h+1)+2 = 24 × 24(h+1) [24 × (24(h+1)+1 − 1) + (24 − 1)]

a2(h+1)+2 = 28 × 24(h+1) (24(h+1)+1 − 1) + 24 × 24(h+1) (24 − 1)

a2(h+1)+2 = 28 a2(h+1) + 24 × (16h+1 )(24 − 1) = 256a2(h+1) + 240 × (16h+1 )

Portanto, se an = 22n (22n+1 − 1) então a2n+2 = 256a2n + 240(16n ) para todo n ∈ N.


118
MA-14 - Aula 08

Semana 28/10 a 03/11

Unidade 17
Fatoração do Fatorial em Primos

17.1 O Teorema de Legendre


Nesta Unidade mostramos como achar a fatoração em números primos de n!, onde n
é um número natural arbitrário. [b]
Se a e b são números naturais, vamos designar pelo símbolo o quociente da divisão
a
de b por a, na divisão euclidiana.
[b]
Uma observação útil é que se a > b > 0, então .
a
Dados um número primo p e um número natural m, vamos denotar por Ep (m) o
expoente da maior potência de p que divide m, ou seja, o expoente da potência de p que
aparece na fatoração de m em fatores primos.

Teorema 17.9. (Legendre)


Sejam n um número natural e p um número primo. Então,
[n] [n] [n]
Ep (n!) = + + + ···
p p2 p3

Na prática, é fácil calcular Ep (n!). Isto se faz com o uso do seguinte algoritmo:

n = pq1 + r1

q1 = pq2 + r2

···

q = pqs + rs

119
120

Como q1 > q2 > . . ., segue-se que, para algum s, tem-se que qs < p. Portanto, segue-se
que
Ep (n!) = q1 + q2 + q3 + · · · + qs

Exemplo 17.2. Determinar a decomposição de 10! em fatores primos e descobrir com


quantos zeros termina a representação decimal desse número..

Para resolvermos o problema, deveremos achar Ep (10!) para todo primo p ≤ 10.
Sendo, E2 (10!) = 5 + 2 + 1 = 8, E3 (10!) = 3 + 1 = 4, E5 (10!) = 2, E7 (10!) = 1
segue-se que 10! = 28 34 52 7
Consequentemente, como há dois fatores iguais a 5 e oito fatores iguais a 2 na de-
composição de 10! em fatores primos, vê-se, imediatamente, que 10! termina com dois
zeros.
O próximo resultado relacionará Ep (n!) com a representação p-ádica de n (i.e., a
representação relativa á base p de n).

Teorema 17.10. Sejam p, n ∈ N com p-primo. Se

n = nrp + nr−1 pr−1 + · · · + n1 p + n0

é a representação p-ádica de n, então

n − (n0 + n1 + · · · + nr )
Ep (n!) =
p−1

17.2 Problemas
Exercício 17.2.1.
Ache a decomposição em fatores primos de 50! e determine com quantos zeros termina
esse número.
Solução.
50 50 50 50 50
Temos E2 [50!] = + 2 + 3 + 4 + 5 = 25 + 12 + 6 + 3 + 1 = 47
2 2 2 2 2
50 50 50
E3 [50!] = + 2 + 3 = 16 + 5 + 1 = 22
3 3 3
50 50 50 50
E5 [50!] = + 2 = 10 + 2 = 12, E7 [50!] = + 2 =7+1=8
5 5 7 7
50 50 50 50
E1 1[50!] = = 4, E13 [50!] = = 3, E17 [50!] = = 1, E19 [50!] = =1
11 13 17 19
50 50 50 50
E23 [50!] = = 1, E29 [50!] = = 1, E31 [50!] = = 1 E37 [50!] = =1
23 29 31 37
121

50 50 50
E41 [50!] = = 1, E43 [50!] = = 1, E47 [50!] = =1
41 43 47
Assim temos: 50! = 247 322 512 78 114 133 171 191 231 291 311 371 411 431 471
Consequentemente temos 12 fatores iguais a 5 e 47 fatores iguais a 2 na decomposição
de 50! em fatores primos, imediatamente observamos que 50! termina em 12 zeros.

Exercício 17.2.2.
a) Ache as maiores potências de 2 e de 5 que dividem 1000!. b) Determine com
quantos zeros termina o número 100!. c) Ache a maior potência de 104 que divide 1000!.
d) Ache o menor número natural n tal que 57 |n!.
Solução.

Revisar bem este problema, foi feito para 10000! a parte (a), consequentemente afeita
o resultado da parte (c) e (d).

a) • Calculemos a decomposição de 1000! em potências de 2

103 103 103 103 103 103


E2 (1000!) = + 2 + 3 + 4 + 5 + 6 =
2 2 2 2 2 2

E2 (1000!) = 5000 + 2500 + 1250 + 625 + 312 + 156 + 78 + 39 + 19 + 9 + 4 + 1 = 9995

A maior potência de 2 que divide 1000! é 9995, pois 29995 |1000!.


• Calculemos a decomposição de 1000! em potências de 5

103 103 103 103 103


E5 (1000!) = + 2 + 3 + 4 + 5 = 2000 + 400 + 80 + 16 + 3 = 2499
5 5 5 5 5

A maior potência de 5 que divide 1000! é 2499, pois 52499 |1000!.

b) Calculemos a decomposição de 100! em potências de 2

100 100 100 100 100 100


E2 (100!) = + + + + + = 50 + 25 + 12 + 6 + 3 + 1 = 97
2 4 8 16 32 64
100 100 100 100
E3 (100!) = + + + = 33 + 11 + 3 + 1 = 48
3 9 27 81
100 100 100 100
E5 (100!) = + = 20 + 4 = 24, E7 (100!) = + =2
5 25 7 49
100 100 100
E11 (100!) = = 9, E13 (100!) = = 7, E17 (100!) = = 5,
11 13 17
100 100 100
E19 (100!) = = 5, E23 (100!) = = 4, E29 (100!) = = 3,
19 23 29
100 100 100
E31 (100!) = = 3, E37 (100!) = = 2, E41 (100!) = = 2,
31 37 41
122

100 100 100


E43 (100!) = = 2, E47 (100!) = = 2, E53 (100!) = = 1,
43 47 53
E59 (100!) = E61 (100!) = E67 (100!) = E71 (100!) = E73 (100!) =

= E79 (100!) = E83 (100!) = E97 (100!) = 1

Assim, podemos escrever que a decomposição de 100! será dado por:

100! = 297 348 524 716 119 137 175 195 234 293 313 372 412 432 472 53·59·61·67·71·73·79·83·97

Com relação a quantidade de zeros de 100!, como está vinculado as potências de 5,


podemos afirmar que a quantidade de zeros será determinado pelo expoente de 5 e
neste caso será de 24.

c) Observe que 104 = 23 · 13, resultados para E2 (1000!) = 9995, por outro lado

103 103 103


E13 (1000!) = + 2 + 3 = 769 + 59 + 4 = 832
13 13 13

Como a potência de 13 é menor do que a potência de 23 tem-se que 104832 |1000!

d) É suficiente observar as potências de 5. Resposta 2499.

Exercício 17.2.3. ( )
2357
(Profmat 2011) É possível repartir exatamente objetos entre 49 pessoas?.
528
Solução.

Exercício 17.2.4.
Mostre que não há nenhum número natural n tal que 37 seja a maior potência de 3
que divida n!.
Solução.

Calculemos as potências de 3 com respeito a n!.


n − (n1 + n2 + . . . + nr )
Pelo Teorema 8.3.2 temos que E3 (n!) = = 7, logo
3−1

n − (n1 + n2 + . . . + nr ) = 14 ⇒ n = 14 + n1 + n2 + . . . + nr

Como n ≥ 14, podemos considerar 14! aqui temos pelo menos mais uma potência de 3

14 14 15 15
E3 (14!) = + 2 = 4 + 1 = 5; E3 (15!) = + 2 =5+1=6
3 3 3 3
16 16 17 17
E3 (16!) = + 2 = 5 + 1 = 6; E3 (17!) = + 2 =5+1=6
3 3 3 3
123

18 18
E3 (18!) = + 2 =6+2=8
3 3
Observe que as potências de 3 entre 15! e 18! (múltiplos de 3) é 6 e 8, mostrando que
não aparecerá nenhuma potência de 3 com expoente 7.
Portanto, podemos afirmar que não divide nenhum n!.

Exercício 17.2.5.
Mostre que, se m; n ∈ N são tais que (m; n) = 1, então,

(m + n − 1)!
∈N
m!n!

Solução.
(m + n)! (m + n − 1)!
Pelo Corolário da página 106 tem-se ∈ N e também ∈N,
m!n! (m − 1)!n!
logo
(m + n)! (m + n) × (m + n − 1)! m + n (m + n − 1)!
= = · ∈N
m!n! m!n! m (m − 1)!n!
(m + n − 1)!
Como (m, n) = 1 ⇒ m - m + n, logo m| , assim
(m − 1)!n!

(m + n − 1)!
(m − 1)!n! (m + n − 1)! (m + n − 1)!
= = ∈N
m m(m − 1)!n! m!n!

Exercício 17.2.6.
a) Mostre que, para todo n ∈ N, tem-se que 2n - n!. b) Mostre que 2n−1 |n! se, e
somente se, existe m ∈ N tal que n = 2m . c) Determine todos os números naturais n tais
que 2n−2 |n!. d) Se r ∈ N, determine todos os números naturais n tais que 2n−r |n!.
Solução.

Exercício 17.2.7.
Sejam n; m ∈ N; mostre que (nm)! é divisível por [(n!)m ; (m!)n ].
Solução.

Exercício 17.2.8.
Para todo n ∈ N, mostre que (n!)(n−1)! divide (n!)!.
Solução.

Exercício 17.2.9.
Seja m; n ∈ N, com n > m > 1. Mostre que é inteiro o número
( )
(n; m) n
n m
124

Solução.

17.3 Exercícios suplementares


Exercício 17.3.1.
Mostre que 2n divide (2n)!. Mostre que, geralmente, o produto de 2n números naturais
consecutivos é divisível por 2n .
Solução.

Exercício 17.3.2.
Mostre que 2n |(n + 1)(n + 2) · · · (2n), mas 2n+1 - (n + 1)(n + 2) · · · (2n).
Solução.

Exercício 17.3.3.
Mostre que n!2n 3n divide (3n)!.
Solução.

Exercício 17.3.4. ( )
pn
Se 1 ≤ r ≤ pn com Ep (r) = k, mostre que é divisível por pn−k , mas não por
r
pn−k+1 .
Solução.

Exercício 17.3.5.
Mostre que
21000 |1001 × 1002 · · · × 2000

; mas que
21001 - 1001 × 1002 · · · × 2000

Solução.
125

Unidade 18
Congruências

18.1 Congruências
Nesta unidade, apresentamos uma das noções mais fecundas da aritmética, introduzida
por Gauss no seu livro “Disquisitiones Arithmeticae”, de 1801. Trata-se da realização de
uma aritmética com os restos da divisão euclidiana por um número fixado.
Seja m um número natural diferente de zero. Diremos que dois números inteiros a
e b são congruentes módulo m se os restos de sua divisão euclidiana por m são iguais.
Quando os inteiros a e b são congruentes módulo m, escreve-se

a∼
= b mod m

Por exemplo, 21 ∼
= 13 mod 2, já que os restos da divisão de 21 e de 13 por 2 são iguais
a 1.
Note que a ∼
= b mod m se, e somente se, m|b − a.
Quando a relação a ∼= b mod m for falsa, diremos que a e b não são congruentes, ou
que são incongruentes, módulo m. Escreveremos, neste caso, a  b mod m.
Decorre da definição, que a congruência, módulo um inteiro fixado m, é uma relação
de equivalência;

Propriedade 18.5. Seja m ∈ N. Para todos a; b; c ∈ N, tem-se que

i) a∼
= a mod m

ii) Sea ∼
= b mod m, então b∼
= a mod m

iii) Se a ∼
= b mod m e b ∼
= c mod m, então a∼
= c mod m.

O que torna útil e poderosa a noção de congruência é o fato de ser uma relação de
equivalência compatível com as operações de adição e multiplicação nos inteiros, conforme
veremos na proposição a seguir.

Propriedade 18.6. Sejam a; b; c; d; m ∈ Z, com m > 1.

i) Se a ∼
= b mod m e c ∼
= d mod m , então

a+c∼
= b + d mod m
126

ii) Se a ∼
= b mod m e c ∼
= d mod m , então

a×c∼
= b × d mod m

Corolario 18.1.1. Para todos n ∈ N, a; b ∈ Z, se a ∼


= b modm, então an ∼
= bn mod m.

18.1.1 O Pequeno Teorema de Fermat


Com a notação de congruências, o Pequeno Teorema de Fermat se enuncia como se
segue: Se p é número primo e a ∈ Z, então

ap ∼
= a mod p

Além disso, se p - a, então


ap−1 ∼
= 1 mod p

A seguir, um resultado relacionado com o cancelamento multiplicativo;

Propriedade 18.7. Sejam a, b, c, m ∈ Z com c ̸= 0 e m > 1. Temos que

m
ac ∼
= bc mod m ⇔ a∼
= b mod
(c, m)

Corolario 18.1.2. Sejam a, b, c, m ∈ Z com m > 1 e (c, m) = 1. Temos que

ac ∼
= bc mod m ⇔ a∼
= b mod m

A seguir, propriedades adicionais das congruências relacionadas com a multiplicação:

Propriedade 18.8. Sejam a, b ∈ Z, se m, n, m1 , m2 , . . . , mr são inteiros maiores que


1, temos:

i) Se a ∼
= b mod m e n|m, então a ∼
= b mod n;

ii) a∼
= b mod m1 , ∀ i = 1, 2, 3, . . . , r ⇔ a∼
= b mod [m1 , m2 , . . . , mr ]

i) Se a ∼
= b mod m, então (a; m) = (b; m).

18.2 Problemas
Exercício 18.2.1.
= 1 mod p, então a ∼
Sejam a; p ∈ N, com p primo. Mostre que, se a2 ∼ = 1modp ou

a = −1modp.
Solução.
127

Suponhamos que a ≥ 1, como todo primo p > 1, então como a2 ∼


= 1modp, logo p|a2 −1
de onde p|(a + 1)(a − 1).
Se p|a − 1 ⇒ a ∼ = 1 mod p.
Se p|a + 1 ⇒ p|a + 1 − p ⇒ ⇒ a∼
p|a − (p − 1) = p − 1 mod p.
2 ∼
Portanto, se a = 1 mod p, então a ∼
= 1 mod p ou a ∼
= (p − 1) mod p.

Exercício 18.2.2.
Ache o resto da divisão de: a) 710 por 51 b) 2100 por 11

c) 521 por 127 d) 14256 por 17 e) (116 + 1717 )21 por 8

f) 1316 − 225 515 por 3 g) 1! + 2! + . . . + (1010 )! por 40

Solução.

a) Primeira solução:

Tem-se que 72 +2 ∼
= 0mod51 de onde (72 +2)5 ∼
= 0mod51, isto é 710 +32 ∼
= 0mod51.

Como 51|710 + 32 ⇒ 51|710 + (32 + 19) − 19, logo 51|710 − 19.

Assim, 710 ∼
= 19 mod 51.

O resto de dividir 710 por 51 é 19.


Segunda solução:

710 = (72 )5 = 495 = (51 − 2)5 = m(51) − 25 = m(51) + 51 − 32 = m(51) + 19.

Assim, o resto de dividir 710 por 51 é 19.

b) 2100 = (25 )20 = 3220 = (m(11) − 1)20 = m(11) + (−1)20 = m(11) + 1.

Assim, o resto de dividir 2100 por 11 é 1.

c) Tem-se que 53 +2 ∼= 0mod127 de onde (53 +2)7 ∼= 0mod127, isto é 521 +27 ∼
= 0mod127.
Observe que 2 = 128 = 127 + 1. Assim, 5 + 1 ∼
7 21
= 0 mod 127.

Como 127|521 + 1 ⇒ 127|710 + (1 + 126) − 126, logo m|521 − 126. Assim,


521 ∼
= 126 mod 127.

O resto de dividir 321 por 127 é 126.

d) Pelo Teorema de Fermat, como (14, 17) = 1, tem-se que 1417−1 ∼


= 1 mod 17.

Logo, (1416 )16 ∼


= 116 mod 17 ⇒ 14256 ∼
= 1 mod 17.

O resto de dividir 14256 por 17 é 1.


128

e) Temos que 116 ∼


= 4 mod 8, e 17 ∼
= 1 mod 8, logo 1717 ∼
= 1 mod 8.
Assim, 116 + 1717 ∼
= 4 + 1 mod 8 ⇒ (116 + 1717 )21 = (m(8) + 5)21 = m(8) + 521 .
Como 52 = m(8) + 1 segue que (52 )10 = (m(8) + 1)10 = m(8) + 1 ⇒ 521 =
m(8) + 5.
Logo, (116 + 1717 )21 = (m(8) + 5)21 = m(8) + 521 = m(8) + 5, isto é (116 + 1717 )21 ∼
=
5 mod 8.
Portanto, o resto de dividir (116 + 1717 )21 por 8 é 5.

f ) Tem-se que 132 ∼


= 1 mod 3 ⇒ 1316 ∼
= 1 mod 3.
Por outro lado, 2 · 5 ∼
= 1 mod 3 ⇒ 215 · 515 ∼ = 1 mod 3. Também 22 ∼
=
1 mod 3 ⇒ 210 ∼ = 1 mod 3. Logo 210 · 215 · 515 ∼
= 1 mod 3.
Assim,1316 ∼
= 1 mod 3 e 225 · 515 ∼
= 1 mod 3.
Portanto, 1316 − 225 515 dividido por 3 tem resto zero.

g)

h)

Exercício 18.2.3.
(ENC 98) O resto da divisão de 1212 por 5 é:
(A) 0 (B) 1 (C) 2 (D) 3 (E) 4
Solução.

Pelo pequeno Teorema de Fermat, 124 ∼ = 1 mod 5, então (124 )3 ∼


= 1 mod 5. Logo
12 ∼
12 = 1 mod 5.
O resto da divisão de 1212 por 5 é 1.

Exercício 18.2.4.
Para todo n ∈ N, mostre que
a) 1016n − 1 é divisível por 70 b) 198n − 1 é divisível por 17.
Demonstração.

a) Tem-se 101 ∼
= 1 mod 10 ⇒ 1016 ∼
= 1 mod 10.
Por outro lado, pelo pequeno Teorema de Fermat 1016 ∼= 1 mod 7. Como (7, 10) = 1
segue da Proposição 9.1.7 que 1016 ∼
= 1 mod 70, isto é 70|1016n − 1.
Portanto, 1016n − 1 é divisível por 70
129

b) Temos 19 ∼ = 2 mod 17. Pelo corolário 1 da proposição 9.1.3 temos 198 ∼


= 28 mod 17,
isto é 198 ∼
= 1 mod 17
Pelo corolário 1 da proposição 9.1.3 temos 198n ∼
= 1mod17 ⇒ 198n −1 ∼
= 0mod17
Portanto, 198n − 1 é divisível por 17.

Exercício 18.2.5.
Determine o resto da divisão por 7 do número
2 3 100
a) 1010 + 1010 + 1010 + . . . + 1010 b) 17 + 27 + . . . + 1007

c) 16 + 26 + . . . + 1006 d) 22225555 + 55552222

Solução.

a) Tem-se 10 ∼
= 3 mod 7 ⇒ 102 ∼
= 2 mod 7 ⇒ 1010 ∼
= 4 mod 7.
Como 1010 ∼
= 4 mod 7 então 10100 ∼
= 410 mod 7. Como 42 ∼
= 2 mod 7 ⇒ 410 ∼
=
4 mod 7.
Logo, 10100 ∼
= 410 mod 7 e 1010 ∼
= 4 mod 7 implicam 10100 ∼
= 4 mod 7.
2
Assim, 1010 e 1010 ao dividir por 7 cada um deles têm resto 4.
Por outro lado, 10100 ∼
= 4mod7, logo (10100 )1 0 ∼
= 410 ∼
= 4mod7, assim 1010 ∼
3
= 4mod7.
Em geral, temos 100 restos iguais a 4

2 3
1010 + 1010 + 1010 + . . . + 1010
100

= 4 + 4 + ... + 4 ∼
= 400 ∼
= 1 mod 7

Portanto, o resto da divisão é 1.

b) 17 + 27 + . . . + 1007 . Pelo PTF (p|ap − a, p-primo) temos

17 + 27 + . . . + 1007 − (1 + 2 + 3 + · · · + 99 + 100) ∼
= 0 mod 7 ⇒

17 + 27 + . . . + 1007 − 5050 ∼
= 0 mod 7 ⇒

17 + 27 + . . . + 1007 − (721 × 7 + 3) ∼
= 0 mod 7 ⇒

17 + 27 + . . . + 1007 ∼
= 3 mod 7 ⇒

Portanto, o resto da divisão é 3.

c) 16 + 26 + . . . + 1006 , Pelo Corolário do PTF (p|ap−1 − 1, p-primo e p - a) além disso

76 ∼
= 0 mod 7, 146 ∼
= 0 mod 7, · · · , 916 ∼
= 0 mod 7, 986 ∼
= 0 mod 7,
130

temos
(16 + 26 + 36 + 46 + 56 + 66 ) − 6 ∼
= 0 mod 7

(86 + 96 + 106 + 116 + 126 + 136 ) − 6 ∼


= 0 mod 7
.. .. ..
. . .

(926 + 936 + 946 + 956 + 966 + 976 ) − 6 ∼


= 0 mod 7

(986 + 996 + 1006 ) − 3 ∼


= 0 mod 7

(16 + 26 + . . . + 1006 ) − (14 × 6 + 2) ∼


= 0 mod 7 ⇒

(16 + 26 + . . . + 1006 ) − (12 × 7 + 2) ∼


= 0 mod 7 ⇒

16 + 26 + . . . + 1006 ∼
= 2 mod 7 ⇒

Portanto, o resto da divisão é 2.

d) 22225555 + 55552222 . Sabemos que 2222 ∼


= 3 mod 7 ⇒ 22223 ∼
= 33 mod 7 assim

22223 ∼
= −1 mod 7 ⇒ (22223 )1851 ∼
= −1 mod 7

logo, 22225553 ∼
= −1 mod 7 como 22222 ∼
= 32 mod 7 segue

22225553 × 22222 ∼
= (−1) × 9 mod 7 ⇒ 22225555 ∼
= 5 mod 7 (18.15)

Por outro lado, sabemos que 5555 ∼


= 4 mod 7 ⇒ 55553 ∼
= 1 mod 7 assim

55553 ∼
= 1 mod 7 ⇒ (55553 )740 ∼
= 1 mod 7

logo, 55552220 ∼
= 1 mod 7 como 55552 ∼
= 42 mod 7 segue

55552220 × 55552 ∼
= (1) × 42 mod 7 ⇒ 55552222 ∼
= 2 mod 7 (18.16)

Das igualdade (18.15) e (18.16) temos

22225555 + 55552222 ∼
= 5 + 2 mod 7

Portanto o resto é zero.

Exercício 18.2.6.
Determine o resto da divisão por 4 do número
131

a) 1 + 2 + 22 + . . . + 219 b) 15 + 25 + . . . + 1005
Demonstração.

a) Seja N = 1 + 2 + 22 + . . . + 219 então N = 3 + 22 (1 + 2 + . . . + 217 ), logo N ∼


= 3 mod 4.
Portanto o resto de da divisão de N por 4 é 3.

b) Seja M = 15 + 25 + . . . + 1005 então M = (15 + 35 + . . . + 995 ) + (25 + 45 + . . . + 1005 ),


logo como 25 + 45 + . . . + 1005 ∼ = 0 mod 4, o resto se obtém de 15 + 35 + . . . + 995 .
∑ 3
Como (2k + 1)5 = C5i (2k)5−i + 2k + 1, então (2k + 1)5 ∼ = 2k + 1 mod 4, assim
i=0
temos
15 + 35 + . . . + 995 ∼
= (1 + 3 + . . . + 99) mod 4

a soma dos 50 primeiros números ímpares 1 + 3 + . . . + 99 = 502 = 4k, k ∈ N,


assim temos
15 + 25 + . . . + 1005 ∼
= 0 + 0 mod 4

Portanto o resto de da divisão de M por 4 é 0.

Exercício 18.2.7.
9
Determine o algarismo das unidades do número 99 .
Solução.

Primeira solução:
Sabe-se que 9+1 ∼= 0mod10, logo pelo Corolário 2 pág. 112 como 9 = 2(4)+1 é ímpar,
então 99 + 19 ∼= 0 mod 10 então 10|99 + 1 + 9 − 9, logo 10|99 − 9, isto é 99 ∼
k k k k k
= 9 mod 10.
Portanto, o algarismo das unidades é 9.
Segunda solução:
Tem-se que 99 = (10 − 1)9 = (10)9 + (−1)9 como 99 é ímpar, tem-se
9 9 9 9

9 9 9
99 = (10)9 − 1 ⇒ 99 = . . . 000 − 1 = . . . 9999

O algarismo das unidades é 9.

Exercício 18.2.8.
Ache os algarismos das centenas e das unidades do número 7999999 .
Sugestão: Observe que 74 = 2401 ∼ = 1 mod 100.

Observe que 7999996 = (74 )249999 ∼


= 1249999 mod 100. Logo, 7999996 ∼
= 1 mod 100.
Por outro lado, 7 = 343 = 43 mod 100, assim pela Propriedade segue que 7999996 · 73 ∼
3 ∼ =
1 · 343 mod 100, de onde

7999999 ∼
= 343 mod 100 ⇒ 7999999 = m(100) + 343 = . . . 00343
132

Os algarismos das centenas e unidade é 3.

Exercício 18.2.9.
Mostre, para todo n ∈ N, que.
a) 102n ∼= 1 mod 11 b) 102n+1 ∼
= −1 mod 11
Solução.

a) Sabemos que 102 −1 = m(11) ⇒ 102 ∼


= 1mod11, logo (102 )n ∼
= 1n mod11, n ∈ N.
Portanto, 102n ∼
= 1 mod 11

b) Pela parte (a) temos 102n ∼


= 1 mod 11 ⇒ 102n+1 ∼
= 10 mod 11 então

102n+1 ∼
= (10 + 1) − 1 mod 11 ⇒ 102n+1 ∼
== 1 mod 11

Portanto, 102n+1 ∼
= −1 mod 11, n∈N

Exercício 18.2.10.
(ENC 2000) Se x2 ∼= 1 mod 5, então

a) x = 1 mod 5 b) x ∼
= 2 mod 5 c) x ∼
= 4 mod 5
d) x ∼= 1 mod 5 ou x ∼
= 4 mod 5 e) x ∼
= 2 mod 5 ou x ∼
= 4 mod 5
Solução.

Temos x2 ∼= 1 mod 5 ⇔ x2 − 1 = 5β, ⇔ (x − 1)(x + 1) = 5β, β∈N


Se (x − 1) = 5 ⇒ x ∼ = 1 mod 5, ou
Se (x + 1) = 5 ⇒ (x + 1) + 4 = 4 + 5 ⇒ x ∼
= 4 mod 5
Resposta: (d) x ∼
= 1 mod 5 ou x ∼
= 4 mod 5

Exercício 18.2.11.
1 · · · pr . Mostre que
Suponha que m = pα1 αr

a∼
= bmodm ⇔ a∼
= bmodpαi
i ; i = 1; . . . ; r

Demonstração.

Seja m = pα1 1 pα2 2 pα3 3 · · · pαr r ∈ N

a∼
= b mod m ⇒ a − b = αpα1 1 pα2 2 pα3 3 · · · pαr r ⇒ a − b = βi pαi i

onde βi ∈ N isto implica a ∼


= b mod pαi i ; i = 1, 2, 3, . . . , r.
Inversamente.
Suponhamos que a ∼ = b mod pαi i ; i = 1, 2, 3, . . . , r, como

(pα1 1 , pα2 2 , pα3 3 , . . . , pαr r ) = 1 ⇒ a − b = βpα1 1 pα2 2 pα3 3 · · · pαr r ⇔


133

a − b = βn ⇔ a∼
= b mod n

Exercício 18.2.12.
Ache o menor número natural que deixa restos 5, 4, 3 e 2 quando dividido, respectiva-
mente, por 6, 5, 4 e 3.
Solução.

Seja n ∈ N o menor número, então para m ∈ N temos n ∼ = (m − 1)mod m, quando


m = 5, 4, 3 e 2.
Isto é n + 1 ∼
= 0 mod m, quando m = 5, 4, 3 e 2.
Calculando o mdc isto é (5, 4, 3, 2) = 60 = m logo n + 1 ∼
= 0 mod 60, de onde o menor
n é 59.
Portanto, o menor número natural que deixa restos 5, 4, 3 e 2 quando dividido, respec-
tivamente, por 6, 5, 4 e 3 é o 59.

Exercício 18.2.13.
Mostre que a soma dos quadrados de quatro números naturais consecutivos nunca pode
ser um quadrado.
Demonstração.

Suponhamos os números consecutivos n − 1, n, n + 1, n + 2 ∈ N então

S = (n − 1)2 + n2 + (n + 1)2 + (n + 2)2 = 4n2 + 4n + 6 = (2n + 1)2 + 5

Temos pelo exercicio anterior que (2n + 1)2 ∼


= α mod 8 onde α = 0 1 ou 4. e como

5 = 5 mod 8 temos

S = (2n + 1)2 + 5 ∼
= α + 5 mod 8, α = 0, 1, 4

assim S ∼= 5 mod 8, S ∼
= 6 mod 8 ou S ∼ = 1mod 8.
Podemos observar que S é par, logo o caso S ∼= 1 mod 8 nunca acontece (ver demons-
tração parte (a) do exercício anterior). Logo S ∼
= 5 mod 8 ou S ∼
= 6 mod 8
Portanto, a soma dos quadrados de quatro números naturais consecutivos nunca pode
ser um quadrado.

18.3 Exercícios suplementares


Problema 18.1.
Ache o menor valor de n, de modo que a maior potência de 5 que divide n! seja 584.
Quais são os outros números que gozam dessa propriedade?
Solução.
134

Calculemos as potências de 5 com respeito a n!.


n − (n1 + n2 + . . . + nr )
Pelo Teorema 8.3.2 temos que E5 (n!) = = 84, logo
5−1

n − (n1 + n2 + . . . + nr ) = 336 ⇒ n = 336 + n1 + n2 + . . . + nr

Como n ≥ 336, podemos considerar 340! aqui temos pelo menos mais uma potência
de 5
340 340 340
E5 (340!) = + 2 + 3 = 68 + 13 + 2 = 83
5 5 5
Observe que 341!, 342!, 343! e 344! não acrescentam potências de 5.

345 345 345


E5 (345!) = + 2 + 3 = 69 + 13 + 2 = 84
5 5 5

Os números que tem como fator 584 são 345!, 346!, 347! e 348!.
O menor deles é 345!.

Problema 18.2.
Dados a1 , a2 , . . . , am ∈ N e b ∈ N, mostre que
[a ] [a ] [ ] [ ] [a ]
1 m a1 + . . . + am a1 m
+ ... + ≤ ≤ + ... + +m:
b b b b b

Demonstração.

Suponhamos ai = bqi + ri , ri ≤ b − 1, i = 1, 2, 3, . . . , n. Tem-se

a1 + a2 + . . . + am r1 + r2 + . . . + rm
= (q1 + q2 + . . . + qm ) +
b b
como pode acontecer r1 + r2 + . . . + rm > b − 1, então
[ ]
a1 + a2 + . . . + am
≥ (q1 + q2 + . . . + qm )
b

isto é [ ] [ ] [ ] [a ]
a1 + a2 + . . . + am a1 a2 m
≥ + + ... + (18.17)
b b b b
Como ai = bqi + ri , ri ≤ b − 1, i = 1, 2, 3, . . . , n, então ai ≤ bqi + (b − 1), ri ≤
b − 1, i = 1, 2, 3, . . . , n. Somando

a1 + a2 + . . . + am ≤ (q1 + q2 + . . . + qm )b + m(b − 1) < (q1 + q2 + . . . + qm )b + mb


[ ] [ ] [ ] [a ]
a1 + a2 + . . . + am a1 a2 m
≤ + + ... + +m (18.18)
b b b b
De (18.17) e (18.18) segue a desigualdade procurada.
135

Problema 18.3.
Sejam m; n; b ∈ N com b ̸= 0. Mostre que

[ ] [ ] [ ] [ ] [ ]
2m 2n m n m+n
a) + ≥ + +
b b b b b
(2m)!(2n)!
b) é um número natural
m!n!(m + n)!
Demonstração.

a) Pelo Problema (18.2) temos


[m] [n] [ ] [ ] [ ] [ ]
m+n m + n + (m + n) 2m 2n
+ + ≤ ≤ + +2
b b b b b b

(m + m)! (n + n)!
b) Aplicando o corolário da página 106 sabemos que os números ,
m!m! n!n!
(m + n)!
e são números naturais, logo é natural o número
m!n!

(m + m)! (n + n)! (m + n)! (2m)!(2n)! (m + n)!


· · = · ∈N
m!m! n!n! (m + n)! m!n!(m + n)! m!n!

(2m)!(2n)!
Portanto, é um número natural.
m!n!(m + n)!

Problema 18.4.
d(n − 1)!
Sejam n; a1 ; . . . , ar ∈ N e d = (a1 ; . . . ; ar ). Mostre que é natural o número .
a1 ! · · · ar !
Demonstração.

Problema 18.5.
136

(a) Mostre que todo quadrado perfeito é congruente, módulo 8, a um dos números 0, 1 ou
4.

(b) Mostre que não há nenhum quadrado perfeito na sequência:

2; 22; 222; 2222; 22222; . . .

(c) Mostre que não há nenhum quadrado perfeito na PA: 3, 11, 19, . . . .

Solução.

a) Todo número natural podemos escrever como algum elemento do conjunto

A = { 8k + r /. r = 0, 1, 2, 3, 4, 5, 6, 7 }

Se n = 8k + r então n2 = (8k + r)2 = m(8) + r2 .

Se r = 0 ⇒ n2 = m(8), se r = 1 ⇒ n2 = m(8) + 1, se r = 2 ⇒ n2 = m(8) + 4,


se r = 3 ⇒ n2 = m(8) + 1, se r = 4 ⇒ n2 = m(8), se r = 5 ⇒ n2 = m(8) + 1,
se r = 6 ⇒ n2 = m(8) + 4, se r = 7 ⇒ n2 = m(8) + 1.

Assim temos que se n = 8k + r então:

Se r = 0, 4 temos n2 ∼
= 0 mod 8; se r = 1, 3, 5, 7 temos n2 ∼
= 1 mod 8 e se r = 2, 6
2 ∼
temos n = 4 mod 8.

Portanto, todo quadrado perfeito é congruente, módulo 8, a um dos números 0, 1


ou 4.

b) Temos 22 = 16 + 6 ⇒ 22 ∼ = 6 mod 8. Depois do termo 22, qualquer elemento da


sequência podemos escrever na forma

22222222 . . . 2222 = 22222222 . . . 2200 + 22

Sabemos que 22222222 . . . 2222 = 22222222 . . . 2200 ∼


= 0 mod 8 e 22 ∼
= 6 mod 8, logo

22222222 . . . 2222 ∼
= 6 mod8

Pela parte (a) deste exercício, não existe quadrado perfeito em algum elemento da
sequência.

Portanto, não há nenhum quadrado perfeito na sequência dada.


137

c) Qualquer elemento da sequência podemos escrever na forma an = 3+8(n−1), n ∈ N∗ .


Assim an ∼ = 3 mod 8 de onde pela parte (a) an não é quadrado perfeito.

Portanto, não há nenhum quadrado perfeito na PA: 3, 11, 19, . . . .

Problema 18.6.
Mostre que nenhum número natural da forma 4n + 3 pode ser escrito como a soma de
dois quadrados.
Solução.

Qualquer número m, n, k ∈ N pode ser escrito como

n2 = m(8), m2 = m(8) + 1, k 2 = m(8) + 4

A soma dois a dois

n2 + m2 = m(8) + 1, m2 + k 2 = m(8) + 5, n2 + k 2 = m(8) + 4

em termos da multiplicidade de 4

n2 + m2 = m(4) + 1, m2 + k 2 = m(4) + 1, n2 + k 2 = m(4) + 0

Assim temos que a soma de dois quadrados

n 2 + m2 ∼
= 1 mod 4, n2 + k 2 ∼
= 0 mod 4

Portanto, nenhum número natural da forma 4n + 3 pode ser escrito como a soma de
dois quadrados.

Problema 18.7.
Se k > 2, mostre, para a ímpar, que a2
k−2

= 1 mod 2k .
Solução.

Indução sobre k ∈ N, k ≥ 3 onde a = 2α + 1, α∈N


23−2
Se k = 3 temos a = a2 = (2α + 1)2 = 22 (α2 + α) + 1, temos que α2 + α é par para
qualquer α ∈ N, logo a2 = m(23 ) + 1 ⇒ a2 ∼
3−2 3−2
= 1 mod 23 .
4−2
Se k = 4 temos a2 = a4 = (2α + 1)4 = 23 (2α4 + 4α3 + 3α2 + α) + 1 observe que

(2α4 + 4α3 + 3α2 + α) = 2(α4 + 2α3 + α2 ) + (α2 + α)

e como α2 + α é par para α ∈ N então a2


4−2
= m(24 ) + 1 ⇒ a2
4−2

= 1 mod 24 .
138

Suponhamos seja válida para qualquer k ∈ N, isto é

a2
k−2

= 1 mod 2k ⇔ a2
k−2
− 1 = m(2k )

Para k + 1 ∈ N temos aplicando a hipótese de indução

(k+1)−2 k−2 k−2 k−2 k−2


a2 − 1 = [a2 ]2 − 1 = [a2 − 1][a2 + 1] = m(2k )[a2 + 1]

− 1 = m(2k+1 )
k−2 (k+1)−2
e como a-ímpar então a2 + 1 é par assim a2
Portanto, a2 ∼
k−2
= 1 mod 2k para a-ímpar e k > 2

Problema 18.8.
Sejam a, b, c, d, m, a1 , b1 , . . . , an , bn ∈ N, com m > 1.

a) Mostre que, se a + b ∼
= 0 mod m e c + d ∼
= 0 mod m, então ac ∼
= bd mod m.

b) Mostre que, se a ∼
= b mod m e c + d ∼
= 0 mod m, então ac + bd ∼
= 0 mod m.

c) Suponha que ai + bi ∼ = 0mod m, i = 1; . . . ; n. Mostre que se n é ímpar, então


a1 · · · an + b1 · · · bn = 0 mod m; e, se n é par, então a1 · · · an ∼
∼ = b1 · · · bn mod m

d) Dê uma outra prova para o Corolário 2 da Proposição 3.

Solução.

a) Se a + b ∼
= 0 mod m então m|a + b. Como c + d ∼
= 0 mod m então m|c + d.
Por outro lado, (ac − bd) = c(a + b) − a(c + d), logo como m|c(a + b) e m|a(c + d)
tem-se que e assim que m|ac − bd.
Portanto, ac ∼
= cd mod m.

b) Como a ∼
= b mod m e c + d ∼
= 0 mod m, então m|a − b e m|c + d.
Logo, m|c(a − b) e m|b(c + d), de onde m|c(a − b) + b(c + d) ⇒ m|ac + bd
Portanto, ac + bd ∼
= 0 mod m.

c)

d)

Problema 18.9.
Sejam a; b; c; m; x0 ∈ N, com m > 2, a ≥ c e 0 ≤ x0 < m
139

a) Mostre que, se a ∼
= b + c mod m, então a − c ∼
= b mod m.

b) Mostre que (m − x0 )2 ∼
= x20 mod m.

Demonstração.

a) Por hipótese a ∼
= b + c mod m então m|a − (b + c), logo m|(a − c) − b.
Portanto, a−c∼
= b mod m.

b) Tem-se que (m − x0 )2 = m2 + 2mx0 + x20 então como m2 + 2mx0 é múltiplo de m,


segue que m|(m − x0 )2 − x20 .
Portanto, (m − x0 )2 ∼
= x20 mod m
140
MA-14 - Aula 09

Semana 04/11 a 10/11

Unidade 19
Aplicações de Congruências
Este material é apenas um resumo de parte do conteúdo da disciplina e o seu estudo
não garante o domínio do assunto.
O material completo a ser estudado encontra-se no Capítulo 9 - Seção 9.2 do livro
texto da disciplina: Aritmética, A. Hefez, Coleção PROFMAT.

19.1 Aplicações de Congruências


Esta unidade é dedicada a apresentar um leque de aplicações da noção de congruência,
mostrando um pouco de sua vasta utilização. Esta unidade é dedicada a apresentar um
leque de aplicações da noção de congruência, mostrando um pouco de sua vasta utilização.
Note que nem todo número de Mersenne é primo. O primeiro número de Mersenne
que não é primo é M11. De fato, é fácil verificar que M2 ; M3 ; M5 ; M7 são primos e que

M11 = 211 − 1 = 2047 = 23 × 89 :

Os exemplos tratados nesta Unidade são os seguintes:

Exemplo 19.3. Mostra que o número de Mersenne M83 = 283 − 1 não é primo, apesar
de 83 ser primo. Isto é, nem todo número da forma 2p − 1 é primo quando p é primo.

De fato, temos que 28 = 256 ∼


= 89mod167, 216 = 892 = 7921 ∼
= 72mod167,

232 = 722 = 5184 ∼


= 7mod167, 264 ∼
= 72 = 49mod167

Dai segue-se que 283 = 264 216 23 ∼


= 94 × 72 × 8mod167, o que implica que 283 − 1 é
divisível por 167.
5
Exemplo 19.4. Mostra que o quinto número de Fermat F5 = 22 + 1 não é primo.

141
142

Note que, da igualdade 641 = 5 × 27 + 1, temos que 5 × 27 ∼


= −1mod641.
Portanto, segue-se que

54 228 = (5 × 27 )4 ∼
= (−1)4 = 1mod641 (19.19)

Disto, e da igualdade 54 + 24 = 641, temos que 54 × 228 + 232 ∼


= 0mod641, logo, de
(19.19)
1 + 22 ∼
5
= 0mod641

o que mostra que 641|F5 .

Exemplo 19.5. Discute critérios de divisibilidade por 2, 5 e 10.

Notando que
10 ∼
= 0mod2, 10 ∼
= 0mod5 e 10 ∼
= 0mod10

temos que, para todo i ≥ 1,

ni 10i ∼
= 0mod2; ni 10i ∼
= 0mod5; ni 10i ∼
= 0mod10

Portanto, dado um número n = nr nr−1 · · · n0 , na base 10, temos que

n∼
= n0 mod2; n∼
= n0 mod5; n∼
= n0 mod10

; o que nos diz que n é divisível por 2, 5 ou 10 se, e somente se, n0 é divisível por 2, 5 ou
10, respectivamente.

Exemplo 19.6. Discute critérios de divisibilidade por 3 e 9.

Como
10 ∼
= 1mod3 e 10 ∼
= 1mod9,

= ni mod3 e ni 10i ∼
segue-se que ni 10i ∼ = ni mod9.
Isto mostra que, se n é representado na base 10 como nr nr−1 · · · n0 , então

= nr + nr−1 + · · · + n0 mod3 e n ∼
n∼ = nr + nr−1 + · · · + n0 mod9;

o que prova que n é divisível por 3 ou 9 se, e somente se, nr + nr−1 + · · · + n0 é divisível,
respectivamente, por 3 ou por 9.

19.1.1 Regra dos nove fora


Isto justifica a famosa regra dos noves fora, que se enuncia como se segue:
143

Para verificar se um dado núumero é divisível por 3 ou por 9, somam-se os seus


algarismos, desprezando-se, ao efetuar a soma, cada parcela igual a nove.
Se o resultado final for 0, então o número é divisível por 9.
Se o resultado for um dos algarismos 0, 3 ou 6, então o número é divisível por 3.

Exemplo 19.7. Discute critério de divisibilidade por 11.

Como 10 ∼
= −1mod11, temos que

102i ∼
= 1mod11 e 102i+1 ∼
= −1mod11.

Seja n = nr · · · n5 n4 n3 n2 n1 n0 um número escrito na base 10.


Temos, então, que n0 ∼ = n0 mod11, n1 10 ∼ = −n1 mod11

n2 102 ∼
= n2 mod11, n3 103 ∼
= −n3 mod11, . . .

Somando, membro a membro, as congruências acima, temos que

n∼
= n0 − n1 + n2 − n3 + · · · mod11.

Logo, 11|n se, e somente se, 11|n0 − n1 + n2 − n3 + · · · .

Exemplo 19.8. Discute a prova dos nove.

A prova dos nove é um teste que se realiza nas quatro operações para detectar erros
de contas.
Como exemplo, suponhamos que efetuamos a multiplicação a × b, obtendo o resultado
c, cuja exatidão queremos verificar.
Suponha que na base 10 tenhamos

a = an an−1 . . . a1 a0 ; b = bm bm−1 . . . b1 b0 ; c = cr cr−1 . . . c1 c0

Apóos ter posto os noves fora em a0 + a1 + · · · + an , obtém-se o algarismo a′ .


Fazendo o mesmo para b e c, obtemos os algarismos b′ e c′ . Efetua-se a multiplicação
a′ × b′ e põem-se os noves fora, obtendo c′′ .
Se c′ ̸= c′′ , então, certamente, foi cometido um erro na operação.
A justificativa é a seguinte:

c′ ∼
=c∼
=a×b∼
= a′ × b′ ∼
= c′′ mod9;

com 0 ≤ c′ < 9 e 0 ≤ c′′ < 9.


144

Caso c′ = c′′ , nada podemos afirmar quanto á exatidão da operação efetuada, mas
podemos garantir que a nossa conta tornou-se mais confiável por ter passado por um
teste.

19.1.2 Representação decimal de número perfeito par


Mostra que a representação decimal de todo número perfeito par ou termina em 28 ou
termina em a6 , onde a é um algarismo ímpar.

Exemplo 19.9. Todo número da forma an = 22n (22n+1 − 1), onde n ∈ N, na sua repre-
sentação decimal, ou termina em 28 ou termina em a6, onde a é um algarismo ímpar.
Em particular, todo número perfeito par termina de um desses modos.

De fato, recorde que, pelo Problema 8.2.6, temos que

a2k+2 = 256a2k + 240 × 16k e a2k+1 = 256a2k−1 + 60 × 16k

Faremos agora a análise dos últimos dois algarismos de 16n ao variar n em N. Temos
que
16 ∼
= 16mod100 164 ∼
= 36mod100

162 ∼
= 56mod100 165 ∼
= 76mod100

163 ∼
= 96mod100 166 ∼
= 16mod100

e, daí para a frente, esses números se repetem periodicamente. Portanto, para todo n ∈ N,
os dois últimos algarismos de 16n são da forma b6, onde b é ímpar.
Observe agora que a2 = 496, logo, termina em a6, onde a é ímpar. Vamos provar, por
indução sobre n, que o mesmo ocorre para todos os números da forma a2n .
Suponha que a2n termina em a6, onde a é um algarismo ímpar; logo,

a2(n+1) = 256a2n + 240 × 16n ∼


= 56 × a6 + 40 × 16n ∼
=

= 10(6a + 3 + 4) + 6 ∼
= (50 + 6)(10a + 6) + 40(10b + 6) ∼
∼ = 10c + 6mod100

onde c é um algarismo.
O resultado, portanto, segue-se neste caso, pois o número 6a + 3 + 4 é ímpar.
Observe agora que a1 = 28; logo, termina em 28. Vamos provar por indução sobre n
que o mesmo ocorre para todos os números da forma a2n+1 .
Suponha que a2n−1 termina em 28. Logo,

a2n+1 = 256a2n−1 +60×16n ∼


= 56×28+60×16n ∼
= 56×28+60(10b+6) ∼
= 68+60×28mod100
145

Exemplo 19.10. Mostra que dado um número natural m, existe um número de Fibonacci
un tal que m|un .

Isto é dado um número natural m, existe um número de Fibonacci un tal que m|un .
De fato, sejam r1 ; r2 ; · · · , respectivamente, os restos da divisão de u1 ; u2 ; · · · , por m.
Como, para todo i , tem-se que 0 ≤ ri < m, segue-se que existem, no máximo, m2
pares ri ; ri+1 distintos.
Portanto, dentre os pares r1 ; r2 ; r2 ; r3 ; . . . ; rm2 +1 ; rm2 +2 existe pelo menos um par que
se repete.
Seja k o menor índice para o qual rk ; rk+1 se repete.
Vamos mostrar que k = 1.
Suponha, por absurdo, que k > 1. Seja rl ; rl+1 um par que repete rk ; rk+1 .
Como rk−1 ∼
= uk−1 = uk+1 −uk ∼
= rk+1 −rk = rj+1 −rj ∼ = uj+1 −uj = uj−1 ∼ = rj−1 modm;
segue-se que o par rk−1 ; rk é igual ao par rj−1 ; rj , o que contradiz a minimalidade de k.
Seja agora rs ; rs+1 , com s > 2, um par que repete o par r1 ; r2 , que é o par 1, 1.
Temos então que

us−1 = us+1 − us ∼
= rs+1 − rs = 1 − 1 = 0modm

Isto nos diz que m|us−1 , provando que existe pelo menos um número de Fibonacci
divisível por m.
Decorre daí e do Problema 6.3.1 que existem infinitos números de Fibonacci divisíveis
por m.
Deduz-se, ainda, que, dado um número primo p, qualquer, existe um número de Fi-
bonacci divisível por p; ou seja, nas decomposições dos números de Fibonacci em fatores
primos aparecem todos os números primos.

Exemplo 19.11. Descreve todos os n para os quais un é divisível por m.

Seja m um número natural, definamos

Dm = { n ∈ N; m|un }

Do exemplo anterior, sabemos que Dm ̸= ∅.


Seja m0 o menor elemento de Dm . Vamos mostrar que

Dm = m0 N = { m0 x; t ∈ N }

De fato, sabemos que m0 N ⊂ Dm , já que, pelo Corolário 6.26, todo elemento m0 t de


m0 N é tal que um0 |um0 t, logo m|um0 t ; ou seja, m0 t ∈ Dm .
146

Reciprocamente, seja n ∈ Dm . Escrevamos

n = m0 t + r; com 0 ≤ r < m0

Pelo Teorema 6.25, temos que

(un ; um0 ) = (um0 ; ur )

Daí, como m|un e m|um0 , segue-se que m|ur , o que contradiria a minimalidade de m0 ,
a menos que r = 0.
Portanto, conclui-se que também vale Dm ⊂ m0 N.
Assim, provamos que Dm = m0 N.
Portanto, para achar os números de Fibonacci divisíveis por um número natural m,
basta achar o primeiro deles um0 e tomar todos os un para os quais n ∈ m0 N.
Por uma análise grosseira do Exemplo 8, pode-se ver facilmente que o número m0 se
encontra no conjunto { 1; 2; :::; m2 }.
O exemplo acima, esclarece vários resultados que encontramos pelo caminho, como,
por exemplo, o Problema 6.3.2 e o Exemplo 6.27.

19.2 Problemas
Problema 19.10.

a) Usando o fato de que 100 é divisível por 4, 25 e 100, ache critérios de divisibilidade
por 4, 25 e 100.

b) Considerando que 1000 é divisível por 8, 125 e 1000, ache critérios de divisibilidade
por 8, 125 e 1000.

Solução.

Seja n = . . . n8 n7 n6 . . . n3 n2 n1 n0 ∈ N, logo podemos decompor na forma polinômica

n = . . . + 107 n8 + 107 n7 + 106 n6 . . . + 103 n3 + 102 n2 + 10n1 + n0

a) Como 100 = 102 , então 102 ∼


= 0 mod 4, mod 25, mod 100, logo

102 ai ∼
= 0 mod 4, mod 25, mod 100

em geral, 10k ak ∼
= 0 mod 4, mod 25, mod 100, k ≥ 2, k ∈ N.
Logo, n∼
= 10n1 + n0 mod 4, mod 25, mod 100
147

Portanto, o número n = . . . n8 n7 n6 . . . n3 n2 n1 n0 será múltiplo de 4, 25 e 100, se o


número m = n1 n0 for múltiplo de 4, 25 e 100.

b) Como 1000 = 103 , então 103 ∼


= 0 mod 8, mod 125, mod 1000, logo

103 a3 ∼
= 0 mod 8, mod 125, mod 1000

em geral, 10k ak ∼
= 0 mod 8, mod 125, mod 1000, k ≥ 3, k ∈ N.
Logo, n∼
= 102 n2 + 10n1 + n0 mod 8, mod 125, mod 1000
Portanto, o número n = . . . n8 n7 n6 . . . n3 n2 n1 n0 será múltiplo de 8, 125 e 1000, se
o número m = n2 n1 n0 for múltiplo de 8, 125 e 1000.

Problema 19.11.
Mostre que um número na base 10 é divisível por 6 se, e somente se, a soma do
algarismo da unidade com o quádruplo de cada um dos outros algarismos é divisível por
6.
Solução.

Seja n = . . . n8 n7 n6 . . . n3 n2 n1 n0 ∈ N, logo podemos decompor na forma polinômica

n = . . . + 107 n8 + 107 n7 + 106 n6 . . . + 103 n3 + 102 n2 + 10n1 + n0

Em geral 10k ∼
= 4k mod 6, k ≥ 1, pois 10k = (6 + 4)k = m(6) + 4k .
Por outro lado: Afirmo 4k ∼ = 4 mod 6.
Indução sobre k. se k = 2 então 42 − 4 = m(6) ⇒ 42 ∼ = 4 mod 6.
Suponhamos para k ∈ N que 4k ∼ = 4 mod 6, isto é 4k = m(6) + 4.
Para k + 1 ∈ N segue

4k+1 = 4 × 4k = 4(m(6) + 4) = m(6) + 42 = m(6) + 16 = m(6) + 4

então 4k+1 ∼
= 4 mod 6.
Como 10 ∼ = 4 mod 6, 102 ∼= 4 mod 6, 103 ∼ = 4 mod 6, 104 ∼ = 4 mod 6, . . . , logo
10k ∼
= 4 mod 6 para k ∈ N∗ .
Logo, 10k nk ∼
= 4nk mod 6, k ≥ 1 e n ∼ = 4(. . . + n5 + n4 + n3 + n2 + n1 ) + n0 mod 6.
Portanto, um número na base 10 é divisível por 6 se, e somente se, a soma do algarismo
da unidade com o quádruplo de cada um dos outros algarismos é divisível por 6.

Problema 19.12.
Usando o fato de que

103 + 1 ∼
= 0 mod 7; mod 11; mod 13
148

prove o seguinte critério de divisibilidade por 7, 11 e 13


Um número n = nr . . . n2 n1 n0 , escrito na base 10, é divisível por 7, 11 ou 13, se, e
somente se,

n5 n4 n3 + n11 n10 n9 + · · · ∼
= n2 n1 n0 + n8 n7 n6 + · · · mod 7; mod 11; mod 13

Demonstração.

Seja n = . . . n8 n7 n6 . . . n3 n2 n1 n0 ∈ N, logo podemos decompor na forma polinômica

n = . . . + 108 n8 + 107 n7 + 106 n6 . . . + 103 n3 + 102 n2 + 10n1 + n0

103 + 1 ∼
= 0 mod 7 ⇔ 103 ∼
= −1 mod 7

então para k ∈ N
{
−1 mod 7, mod 11, mod13 se k − ímpar
(103 )k ∼
=
+1 mod 7, mod 11, mod13 se k − par

Decorre disto que


{
−ni+2 ni+1 ni mod 7, mod 11, mod13 se k − ímpar
(103 )k ni+2 ni+1 ni ∼
=
+ni+2 ni+1 ni mod 7, mod 11, mod13 se k − par

Seja o número natural n = . . . n16 n15 n14 . . . n2 n1 n0


Para os números de três algarismos: n2 n1 n0 , n5 n4 n3 , n8 n7 n6 , n11 n10 n9 , n14 n13 n12
n17 n16 n15 temos

n = . . . 1015 n17 n16 n15 + 1012 n14 n13 n12 + 109 n11 n10 n9 + 106 n8 n7 n6 + 103 n5 n4 n3 + n2 n1 n0

Este número podemos escrever na forma

n = . . . (103 )5 n17 n16 n15 + (103 )4 n14 n13 n12 + (103 )3 n11 n10 n9 + (103 )2 n8 n7 n6 +

+103 n5 n4 n3 + n2 n1 n0

n = · · · − n16 n15 + n14 n13 n12 − n11 n10 n9 + n8 n7 n6 − n5 n4 n3 + n2 n1 n0 ∼


= 0 mod 7;

mod 11, mod 13

n = m(7) + (n2 n1 n0 + n8 n7 n6 + n14 n13 n12 ) − (n5 n4 n3 + n11 n10 n9 + n16 n15 ) ∼
= 0 mod 7;

mod 11, mod 13


149

Portanto,

n5 n4 n3 + n11 n10 n9 + · · · ∼
= n2 n1 n0 + n8 n7 n6 + · · · mod 7 mod 11, mod 13

Problema 19.13.
Analisando a tabela do Exemplo 2.14 (do livro texto), determine os números de Fibo-
nacci que são divisíveis por 8, por 11, por 13 ou por 16.
Demonstração.

Os primeiros termos da sequência de Fibonacci são

1, 1, 2, 3, 5, 8, 13, 21, 34, 55, 89, 144, 233, 377, 610, 987, 1597, 2584, 4181, 6765 . . .

Observando os elementos da sequência e aplicando o Exemplo 9 temos que

u6k ∼
= 0 mod 8, u10k ∼
= 0 mod 11, k ∈ N∗

u7k ∼
= 0 mod 13, u12k ∼
= 0 mod 16, k ∈ N∗

Problema 19.14.
Mostre que um número da forma an = 2(2n − 1) para n > 2 é congruente a 1 módulo
9. Conclua que todo número perfeito par maior do que 6, assim como a soma de seus
algarismos, é da forma 9k + 1.
Sugestão: Utilize as fórmulas do Problema 8.2.3 e indução
Demonstração.

Problema 19.15.
Mostre que se nageq2 então o número de Fermat Fn tem algarismo da unidade igual
a 7.
Demonstração.
150

Problema 19.16.
a) Mostre que para todo n ≥ 1 tem-se que Fn ≡ 5 .mod 12. b) Mostre que nenhum
número de Fermat pode ser um quadrado ou cubo.
Demonstração.
151

Unidade 20
Os Teoremas de Euler e Wilson

20.1 A Função φ Euler


Seja m ∈ N. Denota-se por φ(m) o número de naturais t tais que 1 ≤ t ≤ m − 1 e
(t; m) = 1. Isto define uma importante função

φ : N −→ N

chamada função f i de Euler.


Tem-se que φ(1) = 1 e φ(m) ≤ m − 1, para todo m ≥ 2.
Também vale a igualdade φ(m) = m − 1 se, e somente se, m é um número primo.

20.1.1 O Cálculo de φ(m)


O cálculo de φ(m) em geral é feito através dos dois resultados a seguir:

Propriedade 20.9. Sejam m; m′ ∈ N tais que (m; m′ ) = 1. Então

φ(m · m′ ) = φ(m)φ(m′ )

Propriedade 20.10. Se p é um número primo e r, um número natural, então tem-se


que ( 1)
φ(p ) = p − p
r r r−1
=p 1−
r
p
Assim, obtemos

Teorema 20.11. Seja m > 1 e seja m = pα1 · · · pαn a decomposição de m em fatores


primos. Então,
( 1) ( 1)
φ(m) = pα1 1 · · · pαnn 1 − ··· 1 −
p1 pn

20.1.2 Teorema de Euler


Recorde o Pequeno Teorema de Fermat (PTF):
Se p é primo de a ∈ Z é tal que (a; p) = 1, então ap−1 ∼
= 1 mod p.
Pois, bem, como φ(p) = p − 1, por ser p primo, podemos reescrever o PTF usando a
função de Euler como segue:
152

Se p é primo de a ∈ Z é tal que (a; p) = 1, então

aφ(p) ∼
= 1 mod p

Este resultado se generaliza para um número natural m qualquer no lugar do primo


p, como segue:

Propriedade 20.11. (Euler)


Sejam m; a ∈ Z com m > 1 e (a; m) = 1. Então,

aφ(m) ∼
= 1mod m

20.1.3 Teorema de Wilson


Temos o seguinte teorema:

Propriedade 20.12. (Wilson)


O número p é primo se, e somente se,

(p − 1)! ∼
= −1 mod p

20.2 Problemas
Problema 20.1.
Ache o resto da divisão de; a) 560 por 26 b) 3100 por 10.
Solução. Primeira

a) Temos (5, 26) = 1 logo podemos aplicar o Teorema de Euler, isto é temos 5φ(26) ∼
=
1 mod 26. Por outro lado

φ(26) = φ(2 × 13) = φ(2) · φ(13) = (2 − 1)(13 − 1) = 12

assim temos 512 ∼


= 1 mod 26. Logo

560 = (512 )5 ∼
= 15 = 1 mod 26

Portanto, o resto é 1.

b) Temos (3, 10) = 1 logo podemos aplicar o Teorema de Euler, isto é temos 3φ(10) ∼
=
1 mod 10. Por outro lado

φ(10) = φ(2 × 5) = φ(2) · φ(5) = (2 − 1)(5 − 1) = 4


153

assim temos 34 ∼
= 1 mod 10. Logo

3100 = (34 )25 ∼


= 125 = 1 mod 10

Portanto, o resto é 1.

Solução. Segunda

a) 560 = 2530 = (26 − 1)30 = (m(26) + (−1)30 = m(26) + (−1)30 = m(26) + 1.


Portanto, 560 ∼
= 1 mod 26, isto é o resto da divisão de 560 por 26 é 1.

b) 3100 = 950 = 8125 = (80 + 1)25 = (m(10) + 1)25 = m(10) + 125 .


Portanto, 3100 ∼
= 1 mod 10, isto é o resto da divisão de 3100 por 10 é 1.

Problema 20.2.
Mostre que, se m > 2, então φ(m) é par.
Demonstração.

1o Se m > 2 é primo, então é ímpar e tem-se que φ(m) = m − 1 é um número par.

2o Suponhamos p1 e q1 primos, e m = p1 q1 > 2, (p1 , q1 ) = 1 onde sem perda de


generalidade p1 ≥ 2 e q1 > 2, então tem-se pela Proposição 10.1.3 (Livro texto)
φ(m) = φ(p1 · q1 ) = φ(p1 )φ(q1 ), logo φ(m) = φ(p1 )φ(q1 )
Esta última igualdade é produto de números pares, logo φ(m) é par.

3o Suponhamos m na forma de produto de números primos, m = pα1 1 pα2 2 · · · pαnn .


Pelo Teorema 10.1.2, podemos escrever na forma
( 1 )( 1) ( 1)
φ(m) = pα1 1 pα2 2 · · · pαnn 1 − 1− ··· 1 − ⇒
p1 p2 pn

φ(m) = pα1 1 −1 pα2 2 −1 · · · pαnn −1 (p1 − 1)(1 − p2 )(1 − p3 ) · · · (1 − pn )

Como (p1 − 1) é par, logo φ(m) também é par.

Problema 20.3.
Mostre que se p é um número primo, então para todo a ∈ Z e para todo k ∈ N, tem-se
que
ak(p−1)+1 ≡ a mod p

Demonstração.
154

Problema 20.4.
∑ 1
a) Mostre que i = m · φ(m)
(i;m)=1
2
i<m

b) Mostre que, se m1 ; . . . ; mφ(m) é um sistema reduzido de resíduos módulo m, então m


divide m1 + · · · + mφ(m) .

Sugestão: Note que a igualdade é trivialmente verificada para m = 2. Portanto,


pode-se supor m > 2. Neste caso, φ(m) é par. O resultado segue-se notando que (i; m) =
1 ⇔ (m − i; m) = 1.
Solução. a)

Suponhamos m-primo, temos para todo i = 1, 2, 3, . . . , (m − 1)

∑ (m − 1)(m − 1 + 1) m(m − 1)
i = 1 + 2 + 3 + · · · + (m − 1) = =
(i;m)=1
2 2
i<m

Como para todo i < m temos (i; m) = 1 e m é primo, de onde φ(m) = m − 1, assim

∑ m(m − 1) 1
i= = m · φ(m)
(i;m)=1
2 2
i<m

Suponhamos m não seja primo.


Então todos os i formam um sistema reduzido de resíduos módulo m, pois (i; m) = 1.
Ao todo são φ(m) elementos do sistema reduzido de resíduos módulo m.
Por outro lado como (i; m) = 1 ⇔ (m − i; m) = 1 então temos que i + (m − i) =
ri +rφ(m)−i = m então como estamos pegando os elementos do sistema reduzido de resíduos
módulo m dois a dois, teremos que
[ ]
1
r1 + r2 + · · · + rφ(m) = φ(m) · m
2
∑ 1
Portanto, i = m · φ(m)
(i;m)=1
2
i<m

Solução. b)
Seja m1 ; . . . ; mφ(m) um sistema reduzido de resíduos módulo m, sabemos que φ(m) ≤
m − 1, logo

mi = m1 + m2 + m3 + . . . + mφ(m) =
(i;m)=1
i<m

Pela parte a) deste exercício


1
= mφ(m)
2
155

Logo, m|m1 + m2 + m3 + . . . + mφ(m) .

Problema 20.5.
Resolva em m ∈ N as equações a) φ(m) = 12 b) φ(m) = 8 c)φ(m) = 16
d) φ(m) = 24.
Solução.

a) Se m-primo, então φ(m) = m − 1 = 12 ⇒ m = 13


Se m-composto, podemos supor m = 2α , então φ(2α ) = 2α−1 = 12 absurdo para
α ∈ N.
Suponhamos m = 2α1 3α2 , então
( 1 )( 1) ( 1 )( 1)
φ(m) = 12 = 2α1 · 3α2 1 − 1− = 22 · 32 1 − 1− ⇒ m = 36
2 3 2 3

Suponhamos m = 2α1 7α2 , então


( 1 )( 1) ( 1 )( 1)
φ(m) = 12 = 2 α1
·7 α2
1− 1− =2 ·7 1−
2
1− ⇒ m = 28
2 7 2 7

Suponhamos m = 2α1 13α2 , então


( 1 )( 1) ( 1 )( 1)
φ(m) = 12 = pα1 1 pα2 2 1− 1− = 2 · 13 1 − 1− ⇒ m = 26
p1 p2 2 13

Suponhamos m = 3α1 7α2 , então


( 1 )( 1) ( 1 )( 1)
φ(m) = 12 = pα1 1 pα2 2 1 − 1− =3·7 1− 1− ⇒ m = 21
p1 p2 3 7

Se m-composto, podemos supor m = pα1 1 pα2 2 pα3 3 , então


( 1 )( 1 )( 1)
φ(m) = 12 == 2 · 3 · 7 1 − 1− 1− ⇒ m = 42
2 3 7


156
( 1 )( 1 )( 1 )( 1)
b) φ(m) = 8 = 22 (3 − 1) = 23 · 3 1 − 1− 1− 1− ⇒ m = 24
2 2 2 3
( 1 )( 1)
c) φ(m) = 16 = 20 (17 − 1) ⇒ φ(m) = 2 · 17 1 − 1− ⇒ m = 34
2 17
( 1 )( 1 )( 1)
d) φ(m) = 24 = 21 (13 − 1) = 22 · 13 1 − 1− 1− ⇒ m = 52
2 2 13

Problema 20.6.
Supondo que (a; m) = (a − 1; m) = 1, mostre que

1 + a + a2 + · · · + aφ(m)−1 ∼
= 0 mod m

Solução. Primeira

Temos

aφ(m) − 1
S = 1 + a + · · · + aφ(m)−1 = ⇒ (a − 1)S = aφ(m) − 1
a−1

Pelo Teorema de Euler, sabemos que aφ(m) − 1 ∼


= 0 mod m, assim

(a − 1)S = aφ(m) − 1 ∼
= 0 mod m ⇒ (a − 1)S = αm, β∈N

Por hipótese (a; m) = (a − 1; m) = 1, logo m|S, isto é S ∼


= 0 mod m.
φ(m)−1 ∼
Portanto, 1 + a + · · · + a = 0 mod m.
Solução. Segunda
Como (a; m) = 1, pelo Teorema de Euler aφ(m) ∼= 1modm ⇒ aφ(m) = 1+mult(m).
Como (a − 1; m) = 1, então a − 1 - m e existe β ∈ N tal que a − 1|β.

aφ(m) − 1 mult(m) mβ
1 + a + a2 + · · · + aφ(m)−1 = = = = mult(m) + 0
a−1 a−1 a−1

Portanto, 1 + a + a2 + · · · + aφ(m)−1 ∼
= 0 mod m

Problema 20.7.
Mostre que, se φ(m) = 2r , para algum r ∈ N, então m é um produto de uma potência
de 2 e de primos de Fermat distintos 2.
Sugestão: Se não conseguiu resolver o problema, veja a solução no final da unidade
Solução.

Seja m = pα0 0 pα1 1 . . . pαk k a decomposição de m em fatores primos, onde

2 = p0 < p1 < . . . < pk


157

Temos então, pelo Teorema 3, que

φ(m) = pα0 0 −1 pα1 1 −1 . . . pαk k −1 (p0 − 1) . . . (pk − 1) = 2r

Como p1 ; . . . ; pk são diferentes de 2, devemos ter α1 = . . . = αk = 1. Além disso,


pi − 1 = 2βi , para i = 1; . . . ; k, logo, pi = 2βi + 1.
Como pi é primo, segue-se da Proposição 1, Unidade 14, que βi = 2ni para algum
ni ∈ N. Logo
n n n
m = 2α0 (22 1 + 1)(22 2 + 1) · · · (22 k + 1)
nk
+ 1, · · · 22
n1 n2
onde 22 + 1, 22 + 1 são primos de Fermat distintos.

Problema 20.8.
Supondo que (m; n) = 1, mostre que

mφ(n) + nφ(m) ∼
= 1 mod nm

Solução.

Como (m; n) = 1, pelo Teorema de Euler mφ(n) ∼ = 1 mod n e nφ(m) ∼


= 1 mod m.
φ(n) ∼ φ(m) ∼
Também sabemos que m = 0 mod m e n = 0 mod n.
Pelas propriedades da congruência

mφ(n) + nφ(m) ∼
= 1 + 0 mod n ∼
= 1 mod n ⇒ n|mφ(n) + nφ(m) − 1

mφ(n) + nφ(m) ∼
= 1 + 0 mod m ∼
= 1 mod m ⇒ n|mφ(n) + nφ(m) − 1

Pela Propriedade 9.1.7 parte ii) (Livro texto) temos que

[m, n] | mφ(n) + nφ(m) − 1 ⇒ mφ(n) + nφ(m) − 1 ∼


= 0 mod [m, n]

como (m, n) = 1 ⇒ [m, n] = mn.


Portanto, mφ(n) + nφ(m) ∼
= 1 mod nm.

Problema 20.9.
Sejam a; m ∈ N∗ , com m > 1, tais que (a; m) = 1. Mostre que, se n1 ∼
= n2 mod φ(m),
n1 ∼ n2
então a = a mod m.
Solução.

Como n1 ∼= n2 mod φ(m) então φ(m) | n1 − n2 ⇒ n1 − n2 = β · φ(m) para algum


β ∈ N.
Seja a ∈ N tal que (a; m) = 1, pelo Teorema de Euler segue que aφ(m) ∼
= 1 mod m,
isto é m | aφ(m) − 1 ⇒ aφ(m) = 1 + δ · m para algum δ ∈ N.
158

Por outro lado, existe θ ∈ N tal que

an1 −n2 = aβ·φ(m) = (aφ(m) )β = (1 + δ · m)β = 1 + θ · m ⇒

an1 −n2 ∼
= 1 mod m ⇒ an1 ∼
= an2 mod m

Portanto, an1 ∼
= an2 mod m.

Problema 20.10.
Mostre que 2730|n13 − n, para todo n ∈ N.
Sugestão: Note que 2730 = 13 × 7 × 5 × 2, e que 13|n13 − n e 2|n13 − n.
Para provar que 7 e 5 dividem n13 − n, use o Problema 7.
Solução.

É imediato que 13|n13 − n e 2|n13 − n então n13 ∼


= n mod 13 e n13 ∼
= n mod2
Por outro lado

n13 − n = n(n12 − 1) = n(n6 − 1)(n6 + 1) ⇒ n13 − n ∼


= 0 mod 7

pois n6 − 1 ∼
= 0 mod 7. Também

n13 − n = n[(n4 )3 − 1] = n(n4 − 1)[(n4 )2 + (n4 ) + 1] ⇒ n13 − n ∼


= 0 mod 3

pois n2 − 1 ∼
= 0 mod 3. Por último

n13 − n = n[(n4 )3 − 1] = n(n4 − 1)[(n4 )2 + (n4 ) + 1] ⇒ n13 − n ∼


= 0 mod 3

pois n4 − 1 ∼
= 0 mod 5.
Assim temos 2|n13 −n, 3|n13 −n, 5|n13 −n, 7|n13 −n e 13|n13 −n e como (2, 3, 5, 7, 13) =
1 então 2 × 3 × 5 × 7 × 13 = 2730|n13 − n.

Problema 20.11.
Sejam a ∈ N e n; r ∈ N∗ , com (r; n) = 1. Mostre que no conjunto

{ a; a + r; . . . ; a + (n − 1)r }

há exatamente φ(n) números primos com n.


Solução.

Como (r, n) = 1 então o conjunto { a; a + r; . . . ; a + (n − 1)r } forma um sistema


completo de resíduos módulo m.
Desse conjunto retiramos { 1; r; . . . ; (n − 1)r }, este conjunto é um sistema reduzido
de resíduos módulo m e tem φ(n) elementos primos com n.
159

Problema 20.12.
Quais são os possíveis restos da divisão de a100 onde a ∈ Z, quando dividido por 125.
Solução.

20.3 Teorema de Wilson


Problema 20.13.
Mostre que o número primo p é o menor inteiro maior do que 1 que divide o número
(p − 1)! + 1.
Solução. Primeira

Pelo Teorema de Wilson temos que (p − 1)! ∼


= −1 mod p, logo (p − 1)! + 1 ∼
= 0 mod p
Isto é (p − 1)! + 1 = αp, α ∈ N assim, temos que os elementos do conjunto

{ p, 2p, 3p, 4p, . . . , }

dividem a (p − 1)! + 1.
Sendo p primo, temos que o número primo p é o menor inteiro maior do que 1 que
divide o número (p − 1)! + 1.
Solução. Segunda
Suponhamos exista 1 < q < p tal que q|(p − 1)! + 1
Como q < p ⇒ q|(p − 1)!, pois 1 · 2 · · · q · · · (p − 1) = (p − 1)!. Assim temos que
q|1 ⇒ q = 1.
Isto é contradição.
Portanto, o número primo p é o menor inteiro maior do que 1 que divide o número
(p − 1)! + 1.

Problema 20.14.
Mostre que, se p > 2 é um número primo, então
a) p|(p − 2)! − 1 b) p|(p − 3)! − (p − 1)/2
Solução.

a) O Teorema de Wilson diz que se p é primo, então (p−1)! ∼


= −1modp, isto é (p−1)!+1 =
αp, α ∈ Z.
160

Esta última igualdade podemos escrever (p − 1)! + 1 = p + p(α − 1), α ∈ Z. Assim


temos

(p − 1)! ∼
= (p − 1) mod p ⇒ (p − 1)(p − 2)! ∼
= (p − 1) mod p

Aplicando a propriedade do cancelamento na congruência módulo p temos

(p − 2)! ∼
= 1 mod p

Portanto, p|(p − 2)! − 1.

b) Pela parte a) deste exercício temos (p − 2)! ∼


= 1 mod p. então

(p − 2)(p − 3)! ∼
= 1 mod p ⇒ p(p − 3)! − 2(p − 3)! ∼
= 1 mod p

Assim temos −2(p − 3)! ∼


= 1 mod p, pois p|p(p − 3)!. Observe que

−2(p − 3)! ∼
= 1 mod p ⇔ 2(p − 3)! − 1 = βp, β∈Z

isto é 2(p − 3)! − (p − 1) = p(β − 1).


Como p − 1 é par e 2(p − 3)! também é par, logo de p(β − 1 segue que β − 1 é par,
pois p é primo p > 2. Assim temos que

(p − 1) ∼
2(p − 3)! − (p − 1) = p(β − 1) ⇔ (p − 3)! − = 0 mod p
2

Portanto, p|(p − 3)! − (p − 1)/2.

Problema 20.15.
Seja p > 3 um número primo.

a) Mostre que p! e (p − 1)! − 1 são primos entre si.

b) Prove que, se n ∈ N∗ e n ∼= (p − 1)! − 1 modp!, então os p − 2 inteiros que precedem


n e os p inteiros que sucedem n são compostos.

Solução.
161

Problema 20.16.
Seja p um número primo e a ∈ N.
Mostre que a) ap + (p − 1)!a ∼
= 0 mod p b) (p − 1)!ap + a ∼
= 0 mod p
Solução.

a) Pelo PTF sabemos que ap ∼


= a mod p.
Por outro lado, pelo Teorema de Wilson sabemos que (p − 1)! ∼
= −1 mod p então

(p − 1)! · a = −a mod p.
Das propriedades da congruência módulo p segue

ap + (p − 1)! · a ∼
= a + (−a) mod p ⇒ ap + (p − 1)!a ∼
= 0 mod p

Portanto, se p é um número primo e a ∈ N, então ap + (p − 1)!a ∼


= 0 mod p.

b) Do Teorema de Wilson sabemos, (p − 1)! ∼


= −1 mod p então (p − 1)! · ap ∼
= −ap mod p.
Pelo PTF sabemos que ap ∼
= a mod p então a ∼
= ap mod p.
Das propriedades da congruência módulo p segue

(p − 1)! · ap ∼
= −ap mod p ⇒ (p − 1)! · ap + a ∼
= 0 mod p

Problema 20.17.
Seja p um número primo tal que p ∼
= 1 mod 4. Mostre que
[( ]2
p − 1) ∼
! = −1 mod p
2

Solução.

Por hipótese p ∼
= 1 mod 4, logo p = 4α + 1, α ∈ Z. Seja o número par q = 2α, então
p−1
temos p = 2(2α) + 1 = 2q + 1, assim q = . Temos
2

p = 2q + 1 ⇒ q + (q + 1) ∼
= 0 mod p ⇔ q∼
= −(q + 1) mod p

Aplicando reiteradamente propriedades aditivas da congruência módulo p temos

q∼
= −(q + 1) mod p

q−1∼
= −(q + 2) mod p

q−2∼
= −(q + 3) mod p
162

..
.

q − (q − 2) = 2 ∼
= −[q + (q − 1)] mod p

q − (q − 1) = 1 ∼
= −[q + (q − 0)] mod p

Aplicando a propriedade multiplicativa da congruência

1 · 2 · 3 · 4 · · · (q − 1)q ∼
= (−1)q (q + 1)(q + 2) · · · (2q − 1)(2q) mod p

Como q é par q! ∼
= (q + 1)(q + 2) · · · (2q − 1)(2q) mod p.
Multiplicando por q! temos

q! × q! ∼
= (q!)(q + 1)(q + 2) · · · (2q − 1)(2q) mod p ⇔

(q!)2 ∼
= (2q)! mod p (20.20)

O Teorema de Wilson diz que se p-primo, então (p − 1)! ∼


= −1 mod p e como
p − 1 = 2q segue
(2q)! ∼
= −1 mod p

Da congruência (20.20) e como p − 1 = 2q temos


[( p − 1 ) ]2
(q!)2 ∼
= −1 mod p ⇔ ! ∼ = −1 mod p
2
[( ]2
p − 1) ∼
Portanto, ! = −1 mod p sempre que p ∼
= 1 mod 4.
2
Problema 20.18.
Seja p um número primo ímpar e seja N = 1 · 3 · ·5 · · · (p − 2). Mostre que N ∼
= 1 mod p
ou N + 1 ∼= 0 mod p.
Sugestão: Suponha que p = 2n + 1 e note que (p − 1)! = 2n n!1 · 3 · ·5 · · · (p − 2). Use
os Problemas 15, 16 para calcular n! e o fato de que 22n ∼
= 1 mod p.
Solução.

Sabemos que (p − 1)! = 1 × 2 × 3 · · · × (p − 2) × (p − 1). Também sabemos que

p−1∼
= −1 mod p, p − 3 ∼
= −3 mod p, p − 5 ∼
= −5 mod p, · · ·

··· 6 ∼
= p − (p − 6) ∼
= −(p − 6) mod p, 4 ∼
= −(p − 4) mod p, 2 ∼
= −(p − 2) mod p

Multiplicando estas congruências módulo p

2 · 4 · 6 · · · (p − 3)(p − 1) ∼
= (−1)(−3)(−5) · · · [−(p − 4)][−(p − 2)] mod p
163

2 · 4 · 6 · · · (p − 3)(p − 1) ∼
p−1
= (−1) 2 1 · 3 · 5 · · · (p − 4)(p − 2) mod p

substituindo N
2 · 4 · 6 · · · (p − 3)(p − 1) ∼
p−1
= (−1) 2 N mod p

Multiplicando por N

N · 2 · 4 · 6 · · · (p − 3)(p − 1) ∼
p−1
= (−1) 2 N 2 mod p

isto é (p − 1)! ∼
p−1
= (−1) 2 N 2 mod p.
O Teorema de Wilson diz que (p − 1)! ∼
= −1 mod p para p-primo. Logo temos das
propriedades da congruência que

N2 ∼
p−1
(−1) 2 = −1 mod p

p−1
Quando = 2k + 1 logo p-primo da forma p = 4k + 3 temos
2

−N 2 ∼
= −1 mod p ⇒ N2 − 1 ∼
= 0 mod p

Portanto, N ∼= 1 mod p ou N + 1 ∼
= 0 mod p.
Contra-exemplo: Suponha p = 13 então N = 1 · 3 · 5 · 7 · 9 · 11 = 10395
Temos 10395  1 mod 13 ou 10395 + 1  0 mod 13.
Assim, a restrição p = 4k + 3 é importante.

Problema 20.19.
Seja p um número primo ímpar. Mostre que

a) 12 32 · · · (p − 2)2 ∼
= 22 42 · · · (p − 1)2 mod p

b) Se p ∼
= 1 mod 4, então 22 42 · · · (p − 1)2 + 1 ∼
= 0 mod p.

c) Se p ∼
= 3 mod 4, então 22 42 · · · (p − 1)2 ∼
= 1 mod p.

Solução.

a) Tinhamos do exercício anterior que

2 · 4 · 6 · · · (p − 3)(p − 1) ∼
p−1
= (−1) 2 · 3 · 5 · · · (p − 4)(p − 2) mod p (20.21)

Multiplicando em (20.21) por 2 · 4 · 6 · · · (p − 3)(p − 1) ambos os lados da congruência

22 · 42 · 62 · · · (p − 3)2 (p − 1)2 ∼
p−1
= (−1) 2 · (p − 1)! mod p
164

Como p é ímpar e da propriedade comutativa da congruência

(p − 1)! ∼
= 22 · 42 · 62 · · · (p − 3)2 (p − 1)2 mod p

De modo análogo, multiplicando em (20.21) por 1 · 3 · 5 · · · (p − 4)(p − 2) ambos os


lados da congruência

(p − 1)! ∼
p−1
= (−1) 2 · 32 · 52 · · · (p − 4)2 (p − 2)2 mod p

Como p é ímpar e da propriedade comutativa da congruência

12 · 32 · 52 · · · (p − 4)2 (p − 2)2 ∼
= (p − 1)! mod p

Portanto, 12 32 · · · (p − 2)2 ∼
= 22 42 · · · (p − 1)2 mod p.

b) Multiplicando em (20.21) por 2 · 4 · 6 · · · (p − 3)(p − 1) ambos os lados da congruência

22 · 42 · 62 · · · (p − 3)2 (p − 1)2 ∼
p−1
= (−1) 2 · (p − 1)! mod p

Por hipótese p ∼
p−1
= 1 mod 4 então p − 1 = 4β, β ∈ Z, de onde (−1) 2 = 1. Logo
pela propriedade aditiva da congruência

22 · 42 · 62 · · · (p − 3)2 (p − 1)2 + 1 ∼
= (p − 1)! + 1 mod p

Do Teorema de Wilson (p − 1)! ∼


= −1 mod p ⇒ (p − 1)! + 1 ∼
= 0 mod p.
Portanto, temos 22 · 42 · 62 · · · (p − 3)2 (p − 1)2 + 1 ∼
= 0 mod p quando p ∼
= 1 mod 4.

c) Multiplicando em (20.21) por 2 · 4 · 6 · · · (p − 3)(p − 1) ambos os lados da congruência

22 · 42 · 62 · · · (p − 3)2 (p − 1)2 ∼
p−1
= (−1) 2 · (p − 1)! mod p

Por hipótese p ∼
p−1
= 3 mod 4 então p − 3 = 4β, β ∈ Z, de onde (−1) 2 = −1.

Do Teorema de Wilson (p − 1)! ∼


= −1 mod p ⇒ −(p − 1)! ∼
= 1 mod p.
Logo temos

22 · 42 · 62 · · · (p − 3)2 (p − 1)2 ∼
= −(p − 1)! ∼
= 1 mod p

Portanto, temos 22 42 · · · (p − 1)2 ∼


= 1 mod p quando p ∼
= 3 mod 4.
165

20.4 Problemas suplementares

Problema 20.20.
Seja p um número primo tal que p ∼
= 3 mod 4.
[( ]2
p − 1) ∼
Mostre que ! = 1 mod p
2
Solução.

Por hipótese p ∼
= 3 mod 4, logo p = 4α + 3, α ∈ Z. Seja o número ímpar q = 2α + 1,
p−1
então temos p = 2(2α + 1) + 1 = 2q + 1, assim q = . Temos
2

p = 2q + 1 ⇒ q + (q + 1) ∼
= 0 mod p ⇔ q∼
= −(q + 1) mod p

Aplicando reiteradamente propriedades aditivas da congruência módulo p temos

q∼
= −(q + 1) mod p

q−1∼
= −(q + 2) mod p

q−2∼
= −(q + 3) mod p
..
.

q − (q − 2) = 2 ∼
= −[q + (q − 1)] mod p

q − (q − 1) = 1 ∼
= −[q + (q − 0)] mod p

Aplicando a propriedade multiplicativa da congruência

1 · 2 · 3 · 4 · · · (q − 1)q ∼
= (−1)q (q + 1)(q + 2) · · · (2q − 1)(2q) mod p

Como q é ímpar q! ∼
= −(q + 1)(q + 2) · · · (2q − 1)(2q) mod p.
Multiplicando por q! temos

q! × q! ∼
= −(q!)(q + 1)(q + 2) · · · (2q − 1)(2q) mod p ⇔

(q!)2 ∼
= −(2q)! mod p (20.22)

O Teorema de Wilson diz que se p-primo, então (p − 1)! ∼


= −1 mod p isto é

(2q)! ∼
= −1 mod p ⇔ −(2q)! ∼
= 1 mod p
121

Da congruência (20.22) e como p − 1 = 2q temos


[( p − 1 ) ]2
(q!) ∼
2
= 1 mod p ⇔ ! ∼ = 1 mod p
2
[( ]2
p − 1) ∼
Portanto, ! = 1 mod p sempre que p ∼
= 3 mod 4.
2
Suplementar 20.1.
Se n ∈ N, então φ(n)|n se, e somente se, n é da forma 1; 2a ; 2a 3b , onde a; b ∈ N.
Solução.

Suplementar 20.2.
Sejam m; n ∈ N e d = (m; n). Mostre que

dφ(m)φ(n)
φ(mn) =
φ(d)

Solução.

Suplementar 20.3.
Mostre que φ(m2 ) = mφ(m) para todo m ∈ N.
Solução.

Suplementar 20.4.
Mostre que, se d|n, então φ(d)|φ(n).
Solução.

Suplementar 20.5.
Mostre que, se r1 ; . . . ; rs e r1′ ; . . . rt′ são sistemas reduzidos de resíduos respectivamente
módulo m e módulo m′ , então os números ri m′ +rj′ m, onde 1 ≤ i ≤ s e 1 ≤ j ≤ t, formam
um sistema reduzido de resíduos módulo mm′ .
Solução.

Suplementar 20.6.
Utilize o problema anterior para dar uma outra prova da Proposição 3.
Solução.
122
MA-14 - Aula 10

Semana 11/11 a 17/11

Unidade 21
Congruências Lineares

21.1 Congruências Lineares


Esta unidade é devotada à resolução de sistemas de congruências lineares, ou seja
sistemas do seguinte tipo:

ai X ∼
= bi mod mi ; i = 1; . . . ; r

onde ai ; bi; m ∈ Z; mi > 1.


O problema consiste em determinar, se existirem, os números inteiros x tais que

ai x ∼
= bi mod mi , para todo i = 1; 2, . . . ; r

Inicialmente, damos um critério para decidir quando uma dessas congruências

ax ∼
= b mod m

admite solução.
Propriedade 21.13. Dados a; b; m ∈ Z, com m > 1, a congruência aX ∼
= bmodm possui
solução se, e somente se, (a; m) | b.
O próximo resultado descreve as soluções, quando existem, da congruência aX ∼
=
b mod m.
Teorema 21.12. Sejam a; b; m ∈ Z, com m > 1 e (a; m) | b. Se x0 é uma solução da
congruência aX ∼
= b mod m, então

m m m
x0 ; x0 + ; x0 + 2 ;··· ; x0 + (d − 1) ;
d d d
123
124

onde d = (a; m), formam um sistema completo de soluções duas a duas incongruentes,
módulo m da congruência.

Corolario 21.1.1. Se (a; m) = 1, então a congruência aX ∼


= b mod m possui uma única
solução módulo m.

Exemplo 21.12. Resolvamos a congruência 8X ∼


= 4 mod 12.
Como d = (8; 12) = 4 divide 4, temos que a congruência tem d = 4 soluções módulo
12.
Por tentativa e erro, obtemos a solução x0 = 2.
12 12 12
Portanto, as soluções módulo 12 são 2; 2 + ; 2+2× ; 2+3× .
4 4 4

21.1.1 Redução de Congruências


Note que, se uma congruência

aX ∼
= b mod m

possui solução, então d = (a; m) divide b.


a b m
Pondo a′ = , b′ = e n = , temos que a congruência acima é equivalente
d d d
a aX ∼

= b mod n; que, por sua vez, é equivalente à congruência X ∼

= b′′ mod n; onde
b′′ = b′ a′′ e a′′ um inverso multiplicativo de a módulo m.

21.2 Problemas
Exercício 21.2.1.
Pode o dobro de um número natural deixar resto igual a 9 quando dividido por 26? E
quando dividido por 25?
Solução.

• Suponhamos que existe x ∈ N tal que 2x ∼ = 9 mod 26, então pela Proposição 11.1.1
do Livro texto, teriamos que (2, 26) = 2 | 9.
Porém como em verdade (2, 26) = 2 - 9, então não existe solução para a equação
2x ∼
= 9 mod 26.
Portanto, o dobro de um número natural nunca poderá deixar resto igual a 9 quando
dividido por 26.
• Para o caso de dividir por 25.
Suponhamos que existe x ∈ N tal que 2x ∼ = 9 mod 25, então pela Proposição 11.1.1 do
Livro texto, teriamos que (2, 25) = 1 | 9. Porém como isto é verdade, então a equação
125

2x ∼= 9 mod 25 sim tem solução. Ainda mais, pelo Corolário 1 tem solução única módulo
25.
Com efeito, 2x ∼
= 9 mod 25 ⇒ (2, 25) = 1 ⇒ 2(13) − 25(1) = 1 de onde
2(117) − 25(9) = 9, logo

2(117 − 25t) − 25(9 − 2t) = 9 ⇒ X = 117 − 25t, Y = 9 − 2t

Quando t = 4 segue que x0 = 17 é a solução minimal.


Portanto a solução de 2x ∼
= 9 mod 25 é x0 = 17 (única solução módulo 25).

Exercício 21.2.2.
Resolva, quando possível, as congruências: a) 3X ∼
= 5 mod 7
b) 6X ∼= 21 mod 18 c) 12X ∼= 36 mod 28 d) 12X + 36 ∼
= 0 mod 28 e)

151X = 11 mod 245
Solução.

a) 3X ∼
= 5 mod 7. Como (3, 7) = 1 e divide 5 a equação tem solução.
Seja 3X − 7Y = 5, então

3(5) − 7(2) = 1 ⇒ 3(25) − 7(10) = 5 ⇒ 3(25 − 7t) − 7(10 − 3t) = 5 ⇒

X = 25 − 7t e Y = 10 − 3t quando t = 3 segue que x0 = 4 é a solução minimal.


Por exemplo quando t = 2 então X = 11 ∼
= 4 mod 7, assim temos 3(4) ∼
= 5 mod 7
Portanto a única solução de 3X ∼
= 5 mod 7 é x0 = 4.
Solução. Segunda
Como (3, 7) = 1 e divide 5 a equação tem solução. Sabemos que 3φ(7) ∼
= 1 mod 7
então 3 − 1 = m(7). Logo
6

5 · 36 − 5 = m(7) ⇒ (3 × 5)(32 )2 − 5 = m(7) ⇒ (1)(3)(22 ) − 5 = m(7)

Isto é, 3(4) ∼
= 5 mod 7.
Portanto a solução de 3X ∼
= 5 mod 7 é x0 = 4 (única solução módulo 7).

b) 6X ∼= 21 mod 18. Como (6, 18) = 6 - 21, pela Proposição 11.1.1 a equação não tem
solução.

c) 12X ∼= 36mod28. Como (12, 28) = 4 e divide 36 a equação tem solução (4 soluções).
12 ∼ 36 28
Sabemos que 12X ∼ = 36 mod 28 ⇔ X = mod , logo temos que
12 12 (12, 28)
resolver X ∼
= 3 mod 7
126

Seja X − 7Y = 3, então 1(8) − 7(1) = 1 ⇒

1(24) − 7(3) = 3 ⇒ 1(24 − 7t) − 7(3 − t) = 3 ⇒

X = 24 − 7t e Y = 3 − t, quando t = 0 segue que x0 = 24. Quando t = 1 segue que


x0 = 27, quando t = 2 segue que x0 = 10; quando t = 3 segue que x0 = 3.
Portanto uma solução x0 é qualquer elemento do conjunto {3, 10, 17, 24}
Solução. Segunda
Resolver 12X ∼
= 36 mod 28 é o mesmo que resolver X ∼
= 3 mod 7.
Logo X = 3 + 7t, t ∈ Z. Quando t = 0 temos x0 = 3 solução minimal.
28 28 28
Assim, a solução da equação é 3, 3 + , 3 + 2 × , 3 + 3 ×
4 4 4
Portanto uma solução x0 é qualquer elemento do conjunto {3, 10, 17, 24}

d) 12X + 36 ∼
= 0 mod 28.
Como (12, 28) = 4 e divide 36 a equação tem solução (4 soluções). Logo

12 36 ∼ 28
12X + 36 ∼
= 0 mod 28 ⇔ X+ = 0 mod
12 12 (12, 28)

Temos que resolver X + 3 ∼


= 0 mod 7 ⇔ 7Y − X = 3. De (7, 1) = 1 segue
7(1) − 1(6) = 1 ⇒ 7(3) − 1(18) = 3, então 7(3 − t) − 1(18 − 7t) = 3
Assim temos, X = 18 − 7t e Y = 3 − t. Devemos escolher valores inteiros de t de
modo que 0 < X < 28, quando t = −1 segue que x0 = 25. Quando t = 0 segue que
x0 = 18, quando t = 1 segue que x0 = 11; quando t = 2 segue que x0 = 4.
Portanto uma solução x0 é qualquer elemento do conjunto {4, 11, 18, 25}.
Solução. Segunda
Resolver 12X + 36 ∼
= 0 mod 28 é o mesmo que X + 3 ∼
= 0 mod 7.
Logo X = −3 + 7t, t ∈ Z. Quando t = 1 temos x0 = 4 solução minimal.
28 28 28
Assim, a solução da equação é 4, 4 + , 4 + 2 × , 4 + 3 ×
4 4 4
Portanto uma solução x0 é qualquer elemento do conjunto {4, 11, 18, 25}.

e)
127

Exercício 21.2.3.
Seja p um número primo e seja a um número inteiro tal que p - a. Mostre que a única
solução módulo p da congruência aX ∼= b mod p é x = ap−2 b.
Solução.

Sem perda de generalidade podemos supor a > p.


Como a - p então (a, p) = 1
Temos a equação aX ∼ = b mod p, e como (a, p) = 1|b o problema admite solução única
(pelo fato (a, p) = 1).
Pelas hipóteses, do pequeno Teorema de Fermat sabemos que, ap−1 ∼
= 1 mod p, isto é
a · ap−2 ∼
= 1 mod p.
Pelas propriedades da congruência podemos escrever na forma a · ap−2 b ∼
= b mod p, isto
é aβ ∼
= b mod p onde β = ap−2 b.
Portanto, como a solução é única, x0 = bap−2 é solução da equação aX ∼
= b mod p.
Solução. Segunda
Temos que resolver aX ∼
= b mod p
Sabemos pelo PTF que ap−1 ∼
= i mod p, pois p - a e p-primo.
Na equação original multiplico por ap−2 isto é

ap−2 · aX ∼
= b · ap−2 mod p ⇔ ap−1 ·X ∼
|{z} = b · ap−2 mod p

=1

isto é X∼
= b · ap−2 mod p.
Portanto, x0 = b · ap−2 é a única solução da congruência.

Exercício 21.2.4.
Sejam a; m ∈ Z, com m > 2 e (a; m) = 1. Mostre que a única solução módulo m da
congruência aX ∼
= bmodm é x = aφ(m)−1 b.
Solução.

Exercício 21.2.5.
Mostre que a congruência X 2 +1 ∼
= 0mod7 não possui soluções. Conclua que a equação
X 2 − 7Y 2 − 14X + 7Y − 6 = 0 não admite soluções inteiras.
Solução.
128

21.3 Teorema Chinês dos Restos


Um problema importante, conforme veremos nas Unidades 23 e 24 sobre criptografia,
é a resolução de sistemas de congruências da forma:

 ∼


a1 X = b1 mod m1

 a2 X ∼
= b2 mod m2
 .. .. ..

 . . .


ar X ∼
= br mod mr

Para que tal sistema possua solução, é necessário que (ai ; mi )|bi , para todo i =
1; 2, · · · , r.
Neste caso, pela discussão feita anteriormente, o sistema acima é equivalente a um da
forma 

 X ∼ = c1 mod n1


 X ∼ = c2 mod n2
. .. .. (21.23)

 .. . .



X ∼ = cr mod nr
As soluções de tal sistema são dadas pelo seguinte

Teorema 21.13. (Teorema Chinês dos Restos)


O sistema (21.23), onde (ni ; nj ) = 1, para todo par ni ; nj com i ̸= j, possui uma
única solução módulo N = n1 n2 · · · nr . Tal solução pode ser obtida como se segue:

x |= N1 y1 c1 + · · · Nr yr cr

onde Ni = N/ni e yi é solução de Ni Y ∼


= 1 mod ni , i = 1, 2, · · · , r

21.3.1 O Problema de Sun-Tsu


Qual é o número que deixa restos 2, 3 e 2 quando dividido, respectivamente, por 3, 5
e 7?
Traduzido em linguagem matemática, o problema de Sun-Tsu equivale a procurar as
soluções do seguinte sistema de congruências:

 ∼
 X = 2 mod 3
X ∼
= 3 mod 5


X ∼
= 2 mod 7

Neste caso, temos que N = 3 × 5 × 7 = 105, N1 = 35, N2 = 21 e N3 = 15.


129

Por outro lado, y1 = 2, y2 = 21 e y3 = 1 são soluções, respectivamente, das congruên-


cias 35Y ∼
= 1 mod 3, 21Y ∼ = 1 mod 5 e 15Y ∼ = 1 mod 7.
Portanto, uma solução módulo N = 105 é dada por

x = N1 y1 c1 + N2 y2 c2 + N3 y3 c3 = 233

Como 233 ∼= 23mod105, segue-se que 23 é uma solução, única módulo 105, do Problema
de Sun-Tsu e qualquer outra solução é da forma 23 + 105t; com t ∈ Z

21.4 Problemas
Exercício 21.4.1.
Ache todos os números naturais que deixam restos 2, 3 e 4 quando divididos por 3, 4
e 5, respectivamente.
Solução.

Temos que resolver o sistema



 ∼
 X = 2 mod 3
X ∼
= 3 mod 4


X ∼
= 4 mod 5

Como (1, 3) = (1, 4) = (1, 5) = 1 o sistema tem solução.


Neste caso, temos que N = 3×4×5 = 60, N1 = 20, N2 = 15 e N3 = 12. Consideremos
o sistema de equações

 ∼ ⇔ ∼
 (4)(5)Y = 20Y = 1 mod 3 2Y = 1 mod 3
(3)(5)Y = 15Y ∼
= 1 mod 4 ⇔ 3Y ∼
= 1 mod 4


(3)(4)Y = 12Y ∼
= 1 mod 5 ⇔ 2Y ∼
= 1 mod 5

Tem-se que y1 = 2, y2 = 3 e y3 = 3 são soluções, respectivamente, do sistema das


congruências
Portanto, uma solução módulo N = 60 é dada por

x = (20)(2)(2) + (15)(3)(3) + (12)(3)(4) = 359 ⇒

Tinhamos N = 60, x = 359 = (5)(60) + 59 assim, seguimos.


Como 359 ∼= 59 mod 60, segue-se que x0 = 59 é a solução minimal única, módulo 60,
do Problema de Sun-Tsu e qualquer outra solução é da forma 59 + 60λ, com λ ∈ N.

Exercício 21.4.2.
130

Ache o menor número natural que deixa restos 1, 3 e 5 quando dividido por 5, 7 e 9,
respectivamente.
Solução.

Temos que resolver o sistema



 ∼
 X = 1 mod 5
X ∼
= 3 mod 7


X ∼
= 5 mod 9

Como (1, 5) = (1, 7) = (1, 9) = 1 o sistema tem solução.


Neste caso, temos que N = 5×7×9 = 315, N1 = 63, N2 = 45 e N3 = 35. Consideremos
o sistema de equações

 ∼ ⇔ ∼
 63Y = 1 mod 5 3Y = 1 mod 5
45Y ∼
= 1 mod 7 ⇔ 3Y ∼
= 1 mod 7


35Y ∼
= 1 mod 9 ⇔ 8Y ∼
= 1 mod 9

Tem-se que y1 = 2, y2 = 5 e y3 = 8 são soluções, respectivamente, do sistema das


congruências.
Tinhamos N = 315, x = 2201 = (54)(315) + 311 assim, seguimos.
Portanto, uma solução módulo N = 315 é dada por

x = (63)(2)(1) + (45)(5)(3) + (35)(8)(5) = 2201 ⇒

Como 2201 ∼ = 311 mod 315 a solução minimal única, módulo 315, do Problema de
Sun-Tsu é 311 e qualquer outra solução é da forma 311 + 315λ, com λ ∈ N.

Exercício 21.4.3.
Dispomos de uma quantia de x reais menor do que 3000. Se distribuirmos essa quantia
entre 11 pessoas, sobra R$1; 00; se a distribuirmos entre 12 pessoas, sobram R$2; 00 e se
a distribuirmos entre 13 pessoas, sobram R$3; 00. De quantos reais dispomos?
Solução.

Exercício 21.4.4.
Um macaco, ao subir uma escada de dois em dois degraus, deixa de sobra um degrau;
ao subir de três em três degraus, sobram dois degraus; e ao subir de cinco em cinco
degraus, sobram três degraus. Quantos degraus possui a escada, sabendo que o número de
degraus está entre 150 e 200 ?
Solução.

Exercício 21.4.5.
131

Resolva o sistema: 
 ∼
 3X = 1 mod 7
5X ∼
= 2 mod 11


4X ∼
= 3 mod 13
Solução.

Primeiramente, (3, 7) = 1 | 7, (5, 11) = 1 | 2, (4, 13) = 1 | 3 logo o sistema tem


solução
Observe que
3X ∼= 1 mod 7 | ⇔ ∼
{z } X = 5 mod 7
×5

5X ∼
= 2 mod 11 | ⇔
{z } X∼
= 7 mod 11
×9

4X ∼
= 3 mod 13 | ⇔
{z } X∼
= 4 mod 13
× 10

Neste caso, temos que N = 7 × 11 × 13 = 1001, N1 = 143, N2 = 91 e N3 = 77.


Consideremos o sistema de equações

 ∼
= 1 mod 7 ⇔ 3Y ∼
 143Y = 1 mod 7
91Y ∼
= 1 mod 11 ⇔ 3Y ∼
= 1 mod 11


77Y ∼
= 1 mod 13 ⇔ 12Y ∼
= 1 mod 13

Tem-se que y1 = 5, y2 = 4 e y3 = 12 são soluções, respectivamente, do sistema das


congruências
Portanto, uma solução módulo N = 1001 é dada por

x = (143)(5)(5) + (91)(4)(7) + (77)(12)(4) = 9819 ⇒

Como 9819 ∼
= 810 mod 1001, segue-se que x0 = 810 é a solução minimal, módulo 1001,
do Problema de Sun-Tsu e qualquer outra solução é da forma 810 + 1001λ, com λ ∈ N.

Exercício 21.4.6.
Levando em consideração que 2275 = 25 × 13 × 7, resolva a congruência 3X ∼
=
11 mod 2275.
Solução.

Temos que (25, 13, 7) = 1 | 3, logo a equação tem solução.


Como 2575 = [25, 13, 7] temos a equivalência

 ∼
 3X = 11 mod 7
3X ∼
= 11 mod 2275 ⇔ 3X ∼
= 11 mod 13


3X ∼
= 11 mod 25
132

Observe que
3X ∼
= 11 mod 7 | ⇔
{z } X∼
= 6 mod 7
× 12

3X ∼
= 11 mod 13 | ⇔
{z } X∼
= 8 mod 13
×9

3X ∼
= 11 mod 25 | ⇔
{z } X∼
= 12 mod 25
× 17

Neste caso, temos que N = 7 × 13 × 25 = 2275, N1 = 325, N2 = 175 e N3 = 91.


Consideremos o sistema de equações

 ∼
= 1 mod 7 ⇔ 3Y ∼
 325Y = 1 mod 7
175Y ∼
= 1 mod 13 ⇔ 6Y ∼
= 1 mod 13


91Y ∼
= 1 mod 25 ⇔ 16Y ∼
= 1 mod 25

Tem-se que y1 = 5, y2 = 11 e y3 = 11 são soluções, respectivamente, do sistema das


congruências
Portanto, uma solução módulo N = 1001 é dada por

x = (325)(5)(6) + (175)(11)(8) + (91)(11)(12) = 37162 ⇒

Como 37162 ∼= 762mod2275, segue-se que x0 = 762 é a solução minimal, módulo 2275,
do Problema de Sun-Tsu e qualquer outra solução é da forma 762 + 2275λ, com λ ∈ N.

Exercício 21.4.7.
Resolva o sistema:

X∼
= 2 mod 3, X∼
= 3 mod 4, X∼
= 4 mod 5, X∼
= 5 mod 6

Sugestão: Resolva, inicialmente, o sistema formado pelas três primeiras congruên-


cias.
Solução.

Resolvendo as três primeiras congruências



 ∼
 X = 2 mod 3
X ∼
= 3 mod 4


X ∼
= 4 mod 5

Como (3, 4) = 1, (3, 5) = 1 e (4, 5) = 1 pelo Teorema Chines do Restos o sistema


tem solução,
Temos que N = 3 × 4 × 5 = 60, N1 = 20, N2 = 15 e N3 = 12. Consideremos o sistema
133

de equações 
 ∼ ⇔ ∼
 20Y = 1 mod 3 2Y = 1 mod 3
15Y ∼
= 1 mod 4 ⇔ 3Y ∼
= 1 mod 4


12Y ∼
= 1 mod 5 ⇔ 2Y ∼
= 1 mod 5
Tem-se que y1 = 2, y2 = 3 e y3 = 3 são soluções, respectivamente, do sistema das
congruências
Portanto, uma solução módulo N = 60 é dada por

x = (20)(2)(2) + (15)(3)(3) + (12)(3)(4) = 359 ⇒

Como 359 ∼
= 59 mod 60 e x0 = 59 é a solução minimal, módulo 60 desta equação
Por outro lado, devemos resolver o sistema, pegando a última equação inicial
{
X ∼
= 59 mod 60
X ∼
= 5 mod 6

Como (60, 6) = 6 e 6|59 − 5 então este último sistema tem solução.


Por outro lado, [60, 6] = 60, logo a sistema se reduz à equação X ∼
= 59 mod 60 cuja
solução é 59 + 60t, t ∈ N.
Portanto, a solução do sistema proposto é 59 + 60t, t ∈ N.

Exercício 21.4.8.
Resolva o sistema:

X∼
= 2 mod 3, X∼
= 3 mod 4, X∼
= 4 mod 5, X∼
= 2 mod 6

Solução.

Pelo Exercício (21.4.7) a solução do sistema



 ∼
 X = 2 mod 3
X ∼
= 3 mod 4


X ∼
= 4 mod 5

nos induce à equação X ∼


= 59 mod 60. Temos que resolver o sistema
{
X ∼
= 59 mod 60
X ∼
= 2 mod 6

Como (60, 6) = 6 e 6 - 59 − 2 então este sistema não tem solução.


Portanto, o sistema proposto não tem solução.
134

Exercício 21.4.9.
(Yi Shing, aprox. 700d.C.) Ache os inteiros que deixam restos 1; 2; 5 e 5 quando
divididos respectivamente por 2; 3; 6 e 12.
Solução.

Exercício 21.4.10.
Sejam F1 , . . . , Fn os n primeiros números de Fermat. Mostre que existe um número
natural N tal que Fi divide N + i − 1 para i = 1, . . . , n.
Solução.

São números de Fermat

1 2 3 n
F1 = 22 + 1 = 5, , F2 = 22 + 1 = 17, , F3 = 22 + 1 = 257, , . . . , Fn = 22 + 1

sempre (Fi ; Fj ) = 1, se i ̸= j.
Temos que mostrar que o sistema


 N ∼
= 0 mod F1



 ∼

 N +1 = 0 mod F2


 ..
. ∼ .
= ..

 N + (i − 1) ∼
= 0 mod Fi



 .. ∼ ..

 . = .


 N + (n − 1) ∼
= 0 mod Fn

possui solução
Do fato mdc{ N + (i − 1), Fi } | 0 então todas as equações têm solução.
Do fato (Fi , Fj ) = 1, i ̸= j = 1, 2, . . . , n ; o sistema tem solução.
Portanto, existe um número natural N tal que Fi divide N + (i − 1) para i = 1, . . . , n.

Exercício 21.4.11.
Sejam a; b; n; m ∈ N, com n; m > 1. Mostre que o sistema

X∼
= a mod n, X∼
= b mod m

possui solução se, e somente se, a ∼


= b mod (n; m). Além disso, se (m; n) = 1, então a
solução é única módulo mn.
Solução.

Condição necessária (⇒)


Suponhamos que o sistema X ∼ = a mod n, X ∼ = b mod m tenha solução, então é óbvio
que (1, m) | a e (1, n) | b isto é cada um das equações tem solução; além disso n | X −a
e m | X − b.
135

Seja d = (m, n), como d | m e d | n então d | X − a e d | X − b logo

X∼
= a mod d e X∼
= b mod d ⇔ X −X ∼
= a − b mod d

então a ∼= b mod (m, n).


Condição suficiente (⇐)
Sejam a, b ∈ N tais que a ∼
= b mod d onde d = (n; m) então existe X ∈ N tal que
{
X ∼
= a mod d
a−b∼
= X − X mod d ⇔
X ∼
= b mod d

Como d = (m, n) então existem β1 , β2 ∈ N tais que n = β1 d e m = β2 d. Dados os


números X − a, X − b, n, m ∈ N pelo algoritmo de Euclides temos que

X − a = nq1 + r1 = (β1 d)q1 + r1 , 0 ≤ r1 < n − 1

também
X − b = mq2 + r2 = (β2 d)q2 + r2 , 0 ≤ r2 < n − 1

De onde pelas congruências temos r1 = r2 = 0.


Logo concluímos que X − a = nq1 também X − b = mq2 .
Portanto, X ∼ = a mod n, X∼= b mod m.
É imediato que quando (m, n) = 1 pelo Teorema Chinese do Resto a sistema tenha
uma única solução.

Exercício 21.4.12.
Seja {a1 ; . . . ; am } um sistema completo de resíduos módulo m.

a) Mostre que se a é um inteiro, então {a1 + a; . . . ; am + a} é um sistema completo de


resíduos módulo m.

b) Se (a; m) = 1, então {a · a1 ; . . . ; a · am } é um sistema completo de resíduos módulo


m. Mostre que vale a recíproca.

c) Se p é primo e a um natural que não é múltiplo de p, mostre que ap−1 ∼


= 1 mod p
(Pequeno Teorema de Fermat).
Sugestão Considere os dois sistemas completos de resíduos mod p: {0; 1; . . . ; p−1}
e {0; a · 1; . . . ; a(p − 1)} e note que 1 · · · (p − 1) ∼
= ap−1 · 1 · · · (p − 1) mod p:

d) Mostre que se (r; m) = 1, então { a, a + r, . . . ; a + (m − 1)r } é um sistema completo


de resíduos módulo m.

Solução.
136

a)

b)

c)

d)

21.5 Exercícios complementares


Exercício 21.5.1.
Resolva o sistema: 
 ∼
 X = 2 mod 11
X ∼
= 4 mod 12


X ∼
= 5 mod 13
Solução.

Como (1, 11) = (1, 12) = (1, 13) = 1 o sistema tem solução.
Neste caso, temos que N = 11 × 12 × 13 = 1716, N1 = 156, N2 = 143 e N3 = 132.
Consideremos o sistema de equações

 ∼ ⇔ 2Y ∼
 156Y = 1 mod 11 = 1 mod 11
143Y ∼
= 1 mod 12 ⇔ 11Y ∼
= 1 mod 12


132Y ∼
= 1 mod 13 ⇔ 2Y ∼
= 1 mod 13

Tem-se que y1 = 6, y2 = 11 e y3 = 7 são soluções, respectivamente, do sistema das


congruências
Portanto, uma solução módulo N = 60 é dada por

x = (156)(6)(2) + (143)(11)(4) + (132)(7)(5) = 12784 ⇒

Tinhamos N = 1716, x = 12784 = (7)(1716) + 772 assim, seguimos.


Como 12784 ∼
= 772mod1716, segue-se que x0 = 772 é a solução minimal, módulo 1716,
do Problema de Sun-Tsu e qualquer outra solução é da forma 772 + 1716λ, com λ ∈ N.

Exercício 21.5.2.
137

Mostre que, se (ni ; nj ) = 1, para todos os i; j = 1; . . . ; r com i ̸= j, então o sistema

X + c1 ∼
= 0 mod n1 , X + c2 ∼
= 0 mod n2 , . . . X + cr ∼
= 0 mod nr

admite solução. Descreva todas as soluções do sistema.


Solução.

As equações da forma X +cr ∼ = 0modnr , podemos escrever na forma 1·X ∼ = −cr modnr ,
estas admitem solução, se (1, ni ) | ci .
Suponhamos que todas as equações do sistema admitam solução.
Como (ni , nj ) = 1, i ̸= j = 1, 2, · · · , r então o sistema admite solução única,
Esta solução única do sistema é módulo N = n1 n2 · · · nr . A solução é da forma

x = N1 y1 c1 + N2 y2 c2 + · · · + nr yr cr

onde Ni = N/ni e yi é solução da equação Ni Y ∼


= i mod ni , i = 1, 2, · · · , r.
138

Unidade 22
Aritmética das Classes Residuais

22.1 Classes Residuais


As congruências módulo um número natural m > 1 permitem definir novas aritméticas.
Atualmente, essas aritméticas são a base de quase todos os procedimentos de cálculo dos
computadores e possuem muitas aplicações na própria matemática e na tecnologia.
Dado um inteiro m > 1, vamos repartir o conjunto Z dos números inteiros em sub-
conjuntos, onde cada um deles é formado por todos os números inteiros que possuem o
mesmo resto quando divididos por m. Isto nos dá a seguinte partição de Z :

[0] = { x ∈ Z; x∼
= 0 mod m },

[1] = { x ∈ Z; x∼
= 1 mod m },
.. ..
. = .

[m − 1] = { x ∈ Z; x∼
= m − 1 mod m }

Paramos em [m − 1], pois tem-se que [m] = [0], [m + 1] = [1], etc.


O conjunto
[a] = { x ∈ Z; x∼
= a mod m }

é chamado de classe residual módulo m do elemento a de Z.


Definimos
Zm = { [0]; [1]; . . . ; [m − 1] }

Note que Zm é um conjunto de conjuntos. Por mais estranho que isto possa parecer,
o conjunto Zm tem uma aritmética própria e tem a vantagem de ser finito, algo muito
desejável em computação.

22.1.1 Exemplos
Exemplo 22.13. Seja m = 2. Então,
[0] = { x ∈ Z; x ∼
= 0 .mod .2 } = { x ∈ Z; x é par }, e
[1] = { x ∈ Z; x∼
= 1 .mod .2 } = { x ∈ Z; x é ímpar }.
Temos também que [a] = [0] se, e somente se, a é par e [a] = [1] se, e somente se, a
é ímpar.

Exemplo 22.14. Seja n = 3. Então


139

[0] = { 3t; t∈Z}


[1] = { 3t + 1; t∈Z}
[2] = { 3t + 2; t∈Z}
Tem-se que


 [0] se a é múltiplo de 3
a∈ ⌈1] se a tem resto 1 quando dividido por 3


⌊2] se a tem resto 2 quando dividido por 3

Uma vantagem das classes residuais é que transformam a congruência a ∼ = b mod m na


igualdade [a] = [b], tornando mais mecânica a manipulação deste conceito.
Em Zm definimos as seguintes operações:
Adição: [a] + [b] = [a + b]
Multiplicação: [a] · [b] = [a · b]
Note que, tendo sido definidas estas operações usando os representantes a e b para
as classes residuais [a] e [b], respectivamente, temos que verificar que ao mudarmos os
representantes das classes [a] e [b], não mudam os valores de [a + b] e de [a · b].
Isto decorre imediatamente das seguintes propriedades das congruências que conhece-
mos:
Se a ∼
= a′ mod m e b ∼= b′ mod m, entao [a + b] = [a′ + b′ ] e [a · b] = [a′ · b′ ].

22.1.2 Propriedades da adição


Essas operações gozam das seguintes propriedades:
Propriedades da Adição
Para todos [a]; [b]; [c] ∈ Zm , temos
A1) Associatividade ([a] + [b]) + [c] = [a] + ([b] + [c]);
A2) Comutatividade [a] + [b] = [b] + [a];
A3) Existência de zero [a] + [0] = [a] para todo [a] ∈ Zm ;
A4) Existência de simétrico [a] + [−a] = [0].

22.1.3 Propriedades da multiplicação


Para todos [a]; [b]; [c] ∈ Zm , temos
M1) Associatividade ([a] · [b]) · [c] = [a] · ([b] · [c]);
M2) Comutatividade [a] · [b] = [b] · [a];
M3) Existência de unidade [a] · [1] = [a].
AM) Distributividade [a] · ([b] + [c]) = [a] · [b] + [a] · [c].
140

Um conjunto munido de uma operação de “adição” e de uma operação de “multiplica-


ção”, com as propriedades acima, será chamado de anel.
Portanto, Zm , com as operações acima, é um anel, chamado anel das classes residuais
módulo m.
Um elemento [a] ∈ Zm será dito invertível, quando existir [b] ∈ Zm tal que [a][b] = 1.
Neste caso, diremos que [b] é o inverso de [a].
As tabelas da adição e da multiplicação em Z2 = { [0]; [1] } são

+ [0] [1] · [0] [1]


[0] [0] [1] [0] [0] [0]
⌊1] [1] [0] ⌈1] [0] [1]

As tabelas da adição e da multiplicação em Z3 = { [0]; [1]; [2] } são

+ [0] [1] [2] · [0] [1] [2]


[0] [0] [1] [2] [0] [0] [0] [0]
⌊1] [1] [2] [0] ⌈1] [0] [1] [2]
⌊2] [2] [0] [1] ⌈2] [0] [2] [1]

Em Z4 = { [0]; [1]; [2], [3] } temos

+ [0] [1] [2] [3] · [0] [1] [2] [3]


[0] [0] [1] [2] [3] [0] [0] [0] [0] [0]
⌈1] [1] [2] [3] [0] ⌈1] [0] [1] [2] [3]
⌈2] [2] [3] [0] [1] ⌈2] [0] [2] [0] [2]
⌈3] [3] [0] [1] [2] ⌈3] [0] [3] [2] [1]

É interessante notar que em Z4 existem dois elementos não nulos cujo produto é nulo:
[2] ̸= [0] e, no entanto, [2] · [2] = [0].
Em Z5 = { [0]; [1]; [2], [3], [4] } temos

+ [0] [1] [2] [3] [4] · [0] [1] [2] [3] [4]
[0] [0] [1] [2] [3] [4] [0] [0] [0] [0] [0] [0]
⌈1] [1] [2] [3] [4] [0] ⌈1] [0] [1] [2] [3] [4]
⌈2] [2] [3] [4] [0] [1] ⌈2] [0] [2] [4] [1] [3]
⌈3] [3] [4] [0] [1] [2] ⌈3] [0] [3] [1] [4] [2]
⌈4] [4] [0] [1] [2] [3] ⌈4] [0] [4] [3] [2] [1]

Note que em Z2 , : Z3 e Z5 , todo elemento distinto de [0] é invertível. Mas isto não
ocorre em todos os Zm . Por exemplo, em Z4 temos que [2] não é invertível.
141

Um anel onde todo elemento não nulo possui um inverso multiplicativo é chamado de
corpo. Portanto, Z2 , Z3 e Z5 , com as operações acima definidas, são corpos; mas Z4 não
é um corpo.
As classe residuais permitem resolver as congruências de seguinte modo: Resolver uma
congruência aX ∼ = b mod m se reduz a resolver em Zm a seguinte equação:

[a]Z = [b]

22.1.4 Exemplo

Exemplo 22.15. Resolver a congruência 4X ∼ = 3 mod 5 equivale a resolver em Z5 a


equação
[4]Z = [3] (22.24)

Ollhando a tabela da multiplicação de Z5 , vemos que [4] · [4] = 1. Logo, 4 é invertível


em Z5 com inverso [4].
Portanto, multiplicando ambos os membros da equação (22.24) por [4] obtemos

[1]Z = [4][4]Z = [4][3] = [2]

Portanto, Z = [2], o que nos diz que as soluções de (22.24) são x = 2 + t4, onde t ∈ Z.
Vemos portanto a importância de saber se um determinado elemento de Zm é invertível.
Esses elementos são caracterizados abaixo.

Propriedade 22.14. [a] ∈ Zm é invertível se, e somente se, (a; m) = 1.

Corolario 22.1.1. Zm é um corpo se, e somente se, m é primo.

22.2 Problemas

Exercício 22.2.1.
Construa as tabelas da adição e da multiplicação para Z6 e Z7
Solução.
142

Para Z6

+ [0] [1] [2] [3] [4] [5] · [0] [1] [2] [3] [4] [5]
[0] [0] [1] [2] [3] [4] [5] [0] [0] [0] [0] [0] [0] [0]
⌈1] [1] [2] [3] [4] [5] [0] ⌈1] [0] [1] [2] [3] [4] [5]
⌈2] [2] [3] [4] [5] [0] [1] ⌈2] [0] [2] [4] [0] [2] [4]
⌈3] [3] [4] [5] [0] [1] [2] ⌈3] [0] [3] [0] [3] [0] [3]
⌈4] [4] [5] [0] [1] [2] [3] ⌈4] [0] [4] [2] [0] [4] [2]
⌈5] [5] [0] [1] [2] [3] [4] ⌈5] [0] [5] [4] [3] [2] [1]

Para Z7

+ [0] [1] [2] [3] [4] [5] [6] · [0] [1] [2] [3] [4] [5] [6]
[0] [0] [1] [2] [3] [4] [5] [6] [0] [0] [0] [0] [0] [0] [0] [0]
⌈1] [1] [2] [3] [4] [5] [6] [0] ⌈1] [0] [1] [2] [3] [4] [5] [6]
⌈2] [2] [3] [4] [5] [6] [0] [1] ⌈2] [0] [2] [4] [6] [1] [3] [5]
⌈3] [3] [4] [5] [6] [0] [1] [2] ⌈3] [0] [3] [6] [5] [3] [1] [4]
⌈4] [4] [5] [6] [0] [1] [2] [3] ⌈4] [0] [4] [2] [5] [2] [6] [3]
⌈5] [5] [6] [0] [1] [2] [3] [4] ⌈5] [0] [5] [3] [1] [6] [4] [2]
⌈6] [6] [0] [1] [2] [3] [4] [5] ⌈6] [0] [6] [5] [4] [3] [2] [1]

Exercício 22.2.2.
Ache os elementos invertíveis de Z6 , Z7 , Z8 e Z9 .
Solução.

• Para Z6 , como (5, 6) = 1, então [5] é o único elemento invertível.

• Para Z7 , como 7 é primo, todos seus elementos não nulos são invertíveis.
O elemento inverso de [2] é [4], de [3] é [5], de [6] é [6].

• Para Z8 , como (3, 8) = 1, (5, 8) = 1, (7, 8) = 1, então o elemento inverso de [3] é


[3], de [5] é [5], de [7] é [7].

• Para Z9 , como (2, 9) = 1, (4, 9) = 1, (5, 9) = 1, (7, 9) = 1, então o elemento inverso


de [2] é [5], de [4] é [7], de [5] é [2], de [7] é [4].

Exercício 22.2.3.
Ache os inversos de:
Solução.

a) [5] em Z6 Resposta: [5]


143

b) [3]; [4] e [5] em Z7 . O elemento inverso de [3] é [5], de [4] é [2], de [5] é [3].

c) [3]; [5], e [7] em Z8 . O elemento inverso de [3] é [3], de [5] é [5], de [7] é [7].

d) [5]; [4] e [8] em Z9 . O elemento inverso de [5] é [2], de [4] é [7], de [8] é [8].

e) [1951] em Z2431

f) [3]; [5], e [7] em Z8 . O elemento inverso de [3] é [3], de [5] é [5], de [7] é [7].

Exercício 22.2.4.
a) Seja {a1 ; . . . ; aφ(m) } um sistema reduzido de resíduos módulo m. Mostre que se
(a; m) = 1, então {a · a1 ; . . . ; a · aφ(m) } é um sistema reduzido de resíduos módulo m.
b) Mostre o Teorema de Euler: Se (a, m) = 1 então aΦ(m) ∼ = 1mod m
Solução.

Exercício 22.2.5.
Considere Zm para m > 2. Mostre que
( a) Zm tem um número par de elementos invertíveis; ( b) se [a] é invertível, então
−[a] é invertível e [a] ̸= −[a]. ( c) Mostre que a soma dos elementos invertíveis de Zm é
igual a [0]. ( d) Mostre que a soma de todos os elementos de um sistema reduzido qualquer
de resíduos módulo m é sempre múltiplo de m.
Solução.

Exercício 22.2.6.
( Enade 2008) No anel dos inteiros módulo 12, R = Z1 2, ( A) não há divisores de
zero. (B) todo elemento não nulo é invertível. ( C) o subconjunto dos elementos invertíveis
forma um subanel de R. ( D) a multiplicação não é comutativa. ( E) há exatamente quatro
elementos invertíveis.
Solução.
144

22.3 Exercícios complementares


Exercício 22.3.1.
Mostre a seguinte generalização do Pequeno Teorema de Fermat, devida a Euler. Se
(a; m) = 1, então aφ(m) ∼
= 1 mod m.
Solução.

Exercício 22.3.2.

a) Mostre que se n não é primo e n > 4, então (n − 1)! ∼


= 0 mod n.

b) E se n = 4, o que acontece?

c) Mostre a recíproca do Teorema de Wilson: Se (n − 1)! + 1 ∼


= 0 mod n, então n é primo.

Solução.
m
A Proposição 9.1.5 diz que a ∼
= b mod ⇒ ac ∼
= bc mod m
(c, m)
a) Por indução sobre n > 4.
Se n = 5 ⇒ (5 − 1)! + 1 = 4! + 1 = 24 + 1 ∼
= 0 mod 5 é verdade.
Suponhamos que seja verdade para qualquer h > 4, isto é que seja verdade

(h − 1)! ∼
=0 mod h ⇒ (h − 1)! = 0 + βh, β∈N

Multiplicando por h + 1 esta última expressão

(h + 1)(h − 1)! = 0 + βh · (h + 1) ⇒

⇒ h(h − 1)! + (h − 1)! = βh · (h + 1)

substituindo a hipótese de indução

⇒ h! + [0 + βh] = βh · (h + 1)


145

b) se n = 4 tem-se (4 − 1)! ∼
= 2 mod 4.

c)

Exercício 22.3.3.
Seja p um número primo, calcule: a) (p!; (p − 1)! − 1) b) (p!; (p − 1)! + 1)
Sugestão Use o Teorema de Wilson.
Solução.

O Teorema de Wilson diz: O número p é primo, é primo se, e somente se (p − 1)! ∼


=
p − 1 mod p.

a) Logo, se p é primo, então p|(p − 1)! − (p − 1) ⇒ p|((p − 1)! − 1) − (p − 2) ⇒


(p − 1)! − 1 ∼
= p − 2 mod p. Sendo sempre p! ∼
= 0 mod p ⇒ p! ∼ = p mod p
Concluimos: (p!; (p − 1)! − 1) ∼
= (p; p − 2) mod p ⇒ (p : p − 2) = 1.
Portanto, (p!; (p − 1)! − 1) = 1.

b) Logo, se p é primo, então p|(p − 1)! − (p − 1) ⇒ (p − 1)! = p − 1 + αp, α ∈ N,


assim (p − 1)! + 1 = (α + 1)p também multiplicando esta última igualdade por p
segue p! + p = p(α + 1)p.
Concluimos: (p − 1)! + 1 ∼
= 0 mod p e p! ∼
= 0 mod p ⇒ (p − 1)! + 1 ∼
= p! mod p.
Portanto, (p!; (p − 1)! + 1) = p.
146
MA-14 - Aula 13

Semana 12/11 a 18/11

Unidade 23
Introdução à Criptografia I

23.1 Criptografia
Exercício 23.1.1.
Será que você notou que o parágrafo acima foi codificado? Use o método de contagem de
frequência para quebrar o código e poder decodificar e ler o parágrafo. Para não simplificar
as coisas, foram eliminados espaços, acentos e pontuação.
Solução.

Exercício 23.1.2.
Discuta as seguintes questões com seus colegas:
(a) Por que a contagem de frequência não funciona quando usamos códigos em bloco?
(b) Por que escolhemos acrescentar exatamente a letra A quando a mensagem tem
quantidade ímpar de letras, em vez de usar, por exemplo, X ou Y ?
Solução.

Exercício 23.1.3.
Usando os números primos 5, 11, 17, 23, 29, 41, 47, 53 e 59, construa uma chave pública
para você utilizar na codificação de mensagens RSA para seus colegas.
Solução.

Exercício 23.1.4.
Use a chave pública que você construiu no exercício 3 para codificar seu nome. Escreva
a chave e a mensagem em um papel. Os papéis deverão ser reunidos, embaralhados e
sorteados entre os alunos para o próximo exercício.
Solução.

147
148

Exercício 23.1.5.
Decodifique os demais blocos da mensagem

34 − 129 − 228 − 37 − 105 − 44 − 386 − 125

usando o procedimento acima.


Solução.

Exercício 23.1.6.
Fatore a chave pública que você recebeu quando fez o exercício 4, calcule d e decodifique
a mensagem para saber de quem ela veio.
Solução.
149

Unidade 24
Introdução à Criptografia II

24.1 Criptografia
Exercício 24.1.1.
Discuta em grupo os seguintes problemas relativos à segurança do RSA:
(a) se as chaves públicas de duas pessoas diferentes têm um primo em comum, então
é fácil quebrar o RSA destas duas pessoas;
(b) se usamos o RSA, mas codificamos a mensagem partindo-a em blocos que consis-
tem de uma única letra, então é fácil decodificar a mensagem, embora o código não seja
quebrado.
Solução.

Você também pode gostar